INVISTA NO SEU SUCESSO:
Preparation for Step 1 relies heavily on a mix of resources like NBME practice exams, UWorld, First Aid, and Mehlman PDFs, as evidenced by the diverse experiences across the posts. Users like those in Post 1 and Post 202 show that NBME scores (ranging from 40s to 80s) are pivotal for gauging readiness, with a consensus that consistent scores above 65% on recent forms signal a strong chance of passing. The journey varies—some, like in Post 212, took two years with extensive resources, while others, like Post 220, passed in weeks with focused NBME review. Key methods include thorough NBME analysis, understanding UWorld explanations, and targeting weak areas with tools like Sketchy or ChatGPT, reflecting a shift from passive reading to active learning.
Test-day experiences reveal a challenging yet manageable exam, often with long question stems and unexpected topics (Post 219), contrasting with NBME’s predictability. Despite this, trusting practice scores over post-exam feelings is a recurring theme—Post 228’s user passed with 72%–74% NBMEs despite feeling “like trash,” and Post 191’s 75%+ scorer felt they “guessed everything” yet likely passed. Time management and energy maintenance (via breaks and snacks) are critical, as stamina wanes over eight hours (Post 178). The posts highlight that while the exam tests familiar content, its presentation demands adaptability, making simulation and resilience key.
Final strategies for the last stretch emphasize high-yield focus (Mehlman PDFs, ethics/biostats) and avoiding burnout (Post 224, Post 245). Opportunities like postponing exist for those below 65%, but many succeed by sticking to a plan—Post 175’s “golden rule” of 70% NBME and 65% Free 120 holds true across successes. Neuroticism post-exam is normal (Post 228, Post 240), yet consistency trumps perfection. Whether cramming (Post 178) or pacing (Post 203), the posts affirm that with targeted effort, passing is achievable, even if the process feels daunting.
Resources
First Aid (FA)
Explanation: Frequently cited as a foundational resource for Step 1 preparation across multiple posts (e.g., Post 7, Post 16, Post 20). It’s used for content review, high-yield facts, and as a reference for weak areas. Users emphasize its importance for holistic knowledge and retention (e.g., Post 6, Post 33).
UWorld
Explanation: A primary question bank used by nearly all posters (e.g., Post 1, Post 5, Post 15). It’s praised for simulating exam conditions, learning through explanations, and identifying weak areas. Completion percentages vary (e.g., 35% in Post 7, 90% in Post 33), with scores ranging from 40% to 78% (e.g., Post 190).
NBME Practice Exams
Free 120
Mehlman PDFs and Videos
Sketchy (Micro and Pharm)
Pathoma
Boards and Beyond (BnB)
Bootcamp
Dirty Medicine
AMBOSS
Anki
Scores
NBME Scores
Range: 31% (Post 55) to 89% (Post 169).
Examples: 56% on NBME 28 (Post 15), 62% on NBME 26/27 (your query), 75% on NBME 28 (Post 25).
Explanation: Scores reflect readiness, with 65%+ often cited as a safe passing threshold (e.g., Post 59, Post 74). Drops like yours from 62% to 56% (Post 15) prompt strategy adjustments.
UWorld Scores
Range: 40% (Post 7) to 78% (Post 190).
Examples: 57% at 90% complete (Post 33), 60% at 70% complete (Post 170), 70% at 35% complete (your query).
Explanation: Indicates learning progress, with higher percentages suggesting better retention and application (Post 190).
Free 120 Scores
Range: 60% (Post 51) to 87% (Post 60).
Examples: 68% (Post 4), 63% (Post 59), 67% (Post 6).
Explanation: Used as a final benchmark, with 65%+ considered reassuring (Post 202).
Methods
Targeted Review of Weak Areas
Explanation: Analyzing NBME performance reports to focus on low-scoring subjects (e.g., Post 1, Post 15). Users adjust study plans based on patterns (Post 5).
URL: e.g., Post 1.
Timed Practice Blocks
Explanation: Simulating exam conditions with UWorld or NBME blocks to build stamina and time management (e.g., Post 15, Post 38).
URL: e.g., Post 15.
Active Recall
Explanation: Techniques like teaching concepts aloud or using flashcards to reinforce memory (e.g., Post 7, Post 60). Preferred over passive reading (Post 52).
URL: e.g., Post 7.
Reviewing Incorrect Answers
Explanation: Thorough analysis of mistakes from practice exams and Qbanks to prevent repetition (e.g., Post 1, Post 19). Key for score improvement (Post 52).
URL: e.g., Post 1.
Tips and Tricks
Prioritize Quality Over Quantity
Explanation: Deep understanding trumps superficial coverage (e.g., Post 15, Post 202). Focus on high-yield material (Post 52).
URL: e.g., Post 15.
Simulate Exam Conditions
Explanation: Practice with timed, full-length exams to build endurance (e.g., Post 15, Post 49). Helps with time management (Post 219).
URL: e.g., Post 15.
Stay Balanced
Trust Your Prep
Explanation: Rely on practice scores over post-exam feelings (e.g., Post 202, Post 220). Avoid overanalyzing (Post 228).
URL: e.g., Post 202.
Key Information
Time Constraints
Explanation: Many face tight schedules (e.g., 1 month in your query, 3 weeks in Post 248). Strategies adapt to urgency (Post 15).
URL: Your query, Post 248.
Score Fluctuations
Explanation: Drops like yours (62% to 56%) are common and can signal fatigue or gaps (e.g., Post 15, Post 41). Addressable with focus (Post 52).
URL: Your query, Post 41.
Exam Difficulty
Mehlman Medical YouTube - Dirty Medicine videos (Post 32).
Thermacare Menstrual Heatwrap - Suggested for comfort during testing (Post 165).
USMLE Step 1 Content Outline - Official breakdown of exam topics (Post 86).
Endodermal Derivatives Video - Visual aid for embryology (Post 53).
Antibiotics Video - Micro review resource (Post 171).
Motivational Video - Anxiety management support (Post 112).
Step 1 Prep Video - CBSE and Step 1 strategy (Post 116).
Sketchy Pharm PDF - Shared resource (Post 36).
High-Yield Images PDF - Visual prep aid (Post 51).
FA 2024 and AMBOSS Topics - Study guide (Post 212).
Rapid Review FA2025 - Condensed review (Post 230).
Post 1: What is wrong with me
https://www.reddit.com/r/step1/comments/1kof9v3/what_is_wrong_with_me/
16/05/2025, 20:42:41
5 comments
Dude, I’ve been getting the same score for a month. My NBME’s were
Form 28 43%
Form 26 49%
Form 27 59%
Form 29 59%
Form 30 61%
Form 31 62%
Each about a week apart I really wanted to take the exam next week and I’m sick of this.
Comments:
User:What’s the gap in between these nbme and also what are u doing in between these for review and all
User:I don't know your study schedule but if you aren't already review the heck out of your NBMEs (go over all of the options choices and know exactly why you got that answer wrong or right). If you are noticing you are getting the same concepts wrong, watch dirty medicine. Review the Mehlmans PDFs and keep doing UWorld. You got this!
User:That's really risky... What I would do is analyse those graphs after each nbme and figure out patterns of what you're getting wrong... If there is no pattern the you're cooked and need to go over entire course material once more and then take nbmes to recheck your readiness... If u identify a pattern fix it by reading the first aid for holistic knowledge maybe watch a video on difficult topic... Step 1 is really more difficult compared to nbmes and I would not test untill a consistent 70 plus on 2 or 3
User:At this point I’ve taken all the available NBME’s, and Im just wondering if I retake them wouldn’t it be inflated? I got a 65.5 on the offline form 25 but I think that’s inflated too
User:sorry ok
Post 2: Parental consent Q in Mehlman Ethics
https://www.reddit.com/r/step1/comments/1kof0ml/parental_consent_q_in_mehlman_ethics/
16/05/2025, 20:29:59
7 comments
Confused about the highlighted sentence in 2nd picture.
I was under the impression that you only need consent from 1 parent for a given treatment?
Comments:
User:If the other parent is unavailable, then you only need 1 parent. But if both are available and are custodial partents and have reasonable differences in opinion about a treatment plan, then it’s better to make them reach a consenus.
User:Sure, I agree with the answer in that its better to make them reach a consensus. But I was more referring to what happens after; if they still do not agree. How do you know which parent's opinion to go with?
User:Then it would be court order because it’s not an emergency procedure but something has to be done medically in the near term to prevent further harm to the child.
User:I see. Thanks for your help.
User:general tip with ethics, most of the time the answer is the one that is the most open and nice. if your ever stuck don't think just pick the one that sounds the nicest
User:Fairs
User:You don’t need any consent. Children have the right to seek medical treatment on their own. On top of that, parents cannot refuse life saving treatment. If both parents refuse life saving treatment the answer will be “receive a court order.”
Post 3: Like Never Before - Medial Medullary Syndrome
https://www.reddit.com/r/step1/comments/1koef3t/like_never_before_medial_medullary_syndrome/
16/05/2025, 20:01:15
1 comment
No content found.
Comments:
User:okay that cool
Post 4: 68% free 120 push exam?
https://www.reddit.com/r/step1/comments/1koccd5/68_free_120_push_exam/
16/05/2025, 18:26:49
12 comments
I got a 70 on form 31 1 week before exam. Took free 120 today and got 68%. 2 days out. Should I be concerned?
Comments:
User:N=1 but i got a 65% and freaked out 2 days before. If i pushed I would have REALLY regretted it. I was prepared and had no doubt that I passed walking out (I did pass). trust your preparation . if u passed nbmes, go for it
User:Do you mind sharing your other nbme scores? I just got a 65% and 66% on forms 29 and 31. Taking the free120 tomorrow and realistically aiming for around 65%. Do you think I’ll be fine with those scores?
User:I wouldn’t. 68 remains still a good score with wiggle room . Review it well though and see if there are things you rushed or just didn’t know
User:Dramatic, you will do fine
User:I got a 65 on the free 120 and passed. You should be good to go
User:I got 70 on free120 and passed 3 days later. 71 on 31 1 week before testing
User:I think you’re fine if your other NBMEs are safe
User:Hey, no, 68 is still good. I'd say go for the exam
User:U can go for it .. got similar scores and Passed
User:You’re good to go , just go throughly over your incorrects
User:Take it
User:You are on a bubble with those scores. Given no other information, you should certainly postpone it. You will not regret taking a little more time to prepare more. You will ABSOLUTELY REGRET taking it and failing, literally lowering your future prospects of residency choice.
Post 5: Step 1 NBMEs
https://www.reddit.com/r/step1/comments/1kobq6i/step_1_nbmes/
16/05/2025, 18:00:33
7 comments
Hi guys, Im 5 weeks out from my exam and 2 weeks into dedicated and so far I got
Form 28: 54
A week later Form 29: 59
Going to do form 30 next week
Im just feeling so discouraged. Im so scared of this exam. Ive never done bad on my med school exams (they were in house so Im sure that makes a difference) but still getting a failing score is so jarring especially since Ive been working so hard :/
Comments:
User:Hey! I am in almost the same situation!
User:dm me, lets create a whatsapp group to help each other
User:Hey i can help you if you like. I DMed you
User:Help me out as well !
User:Dmed you .kindly check
User:Hey ! First key to pass this exam is leave is scaring type shit from this exam , just take this as test where u have only two paths pass or fail .. do your 100% nd after that let the god do the justices many Of my Mates Lose this exam becoz they were fearing it Lot That on the exam day they Panicked a lot that they failed by 2-3 points … so confidence + hardwork+ faith is the key … get through ur nbmes revise ur Weakness do as much as possible but stop fearing it … Even things doesn’t as plan it will not be end … becoz u are much more than a test !! Good luck honey !!!
Post 6: Do I have chance to pass???
https://www.reddit.com/r/step1/comments/1kob4fu/do_i_have_chance_to_pass/
16/05/2025, 17:34:31
7 comments
Last nmbe 30 64% 31 65% new 120 67%
Exam next week 💔😢 can’t extend
Comments:
User:I’d say you’re more likely to pass than not. Just go in confident that you’ve put in the work. Try to focus on some of your deficits between now and your exam!
User:Thank you
User:Its ok
User:Well i hope you make it...i hv heard of ppl passing with worse... But I'll be honest the exam is tough... Just review first aid starting chapters - Immunology gen path and ethics biostats along with psych those r high scoring and take a day to review max... If you're good at this then just go through graphs of your nbmes and see worst performers and review few of those... Don't panic and trust your prep... Step exam tests your overall prep some qs you wouldn't have even heard of but would know from your studies as a med student and few r research qs which aren't graded so don't lose composure if u see tough ones... Godspeed
User:For the millionth time, nbmes mean jackshit. And to answer ur question, YEAH you have a chance to pass it.
User:Same here let’s hope we pass
User:Try revising fA as much as you can Just stick to it for a week You’ll get a P
Post 7: Please help! I need advice
https://www.reddit.com/r/step1/comments/1ko7fx9/please_help_i_need_advice/
16/05/2025, 15:00:22
4 comments
I have 2 months left to study for step 1.
I’ve read FA once but I feel like I’ve forgotten it. I went through all of sketchy micro and most of sketchy pharm but I always forget it. I’ve added anki to help but I don’t know if I’m using it right.
My overall UWorld score is 40% and I’ve done 35%. I know UWorld is a learning tool but I feel like crashing out every time I get a low score.
What can I do to improve my scores/studying? How should I approach the next two months?
Please help me, I’m going crazy here.
Comments:
User:Hi, I'd recommend getting on the NBMEs and reviewing them thoroughly. The concepts repeat throughout the exams. Create a memory tool or watch a video for each to make the material stick, then an anki to apply spaced repetition. You have to make sure to really review the questions - whats your thought process? was it lack of knowledge or incorrect knowledge? was it doubting yourself? Understand how you reason through questions and target that, as well. Hope this helps! If you are looking for a Step 1 tutor, you can DM or email me at [resilientmdtutoring@gmail.com](mailto:resilientmdtutoring@gmail.com) . I provide premium tutoring services and have been doing this for 2 years now. I'm also a matched IM resident at T20 program.
User:sounds like you need to strengthen your content base. i personally used bootcamp bc i liked the teaching style. best rec is find a video resource that you can watch and which is best for your learning style and watch it. if i relied on FA (personally) i would’ve never remembered anything tbh.
User:also use the mnemonics tag on AnKing. truly game changing for certain topics
User:Ask Chat GPT to create a daily schedule for you, doing at least 6 systems daily. Try doing something like 10 Micro, 10 Biochem, 10 Neuroanatomy, 10 GI, and aim to cover at least 6 systems daily to help with content retention. Also, consider using Mehlman's PDF and his YouTube playlist to target your weak areas. Then include FA/Mnemosyne Anki too.
Post 8: Genital ulcers (NBME favorite repeating micro Q)
https://www.reddit.com/r/step1/comments/1ko5szm/genital_ulcers_nbme_favorite_repeating_micro_q/
16/05/2025, 13:54:17
13 comments
**I made this image based on NBMEs Qs (no copyright violation)**
Check out my older posts for more like that.
Comments:
User:Bro I wish I had this, I love it
User:Cool as fuck, I’m in derm unit so this is nice
User:I hope u don’t see genital ulcers 😂😂
User:Haha yeah we did touch upon it a little but i guess it’s not too much of the unit
User:LN?
User:Lymph nodes
User:Where was this when I needed it a week ago huh
User:Nice work, Op. Thank you🙏
User:Thanks ,very useful!
User:Bro I love it
User:so helpful omg!
User:Love it ! Thanks
User:wonderful
Post 9: Study mate at Riyadh
https://www.reddit.com/r/step1/comments/1ko53qu/study_mate_at_riyadh/
16/05/2025, 13:25:09
1 comment
Hello, everyone
I'm looking for study mate at Riyadh city
If anyone interested send me on my DM
Comments:
User:I was looking for a study partner too, lemme know if u are still up!
Post 10: Retaking NBMEs again after5, to 6 months pause
https://www.reddit.com/r/step1/comments/1ko3nq7/retaking_nbmes_again_after5_to_6_months_pause/
16/05/2025, 12:26:24
1 comment
Hi, everyone
If someone retaking NBMEs after 6 months, ant there's is very little bias like 2, 3 percentage will it be good, I mean is it okey to use them for assessments or what to day.
I am on a second attempt for step 1
Kindly need help
Comments:
User:There may be some questions you’ll subconsciously remember so there will be a chance your score will be inflated
Post 11: I'll take "things that never happened" for $500, Alex
https://www.reddit.com/r/step1/comments/1ko37h9/ill_take_things_that_never_happened_for_500_alex/
16/05/2025, 12:07:51
20 comments
No content found.
Comments:
User:Yes, my MS patient just asked me this the other day at the vending machine.
User:Patients and doctors use the same vending machines at your institution? You’re very egalitarian.
User:Idk why but when I read the patient was “being followed” by her neurologist my brain thought she was being stalked 😂
User:I was thinking “well this is an ethics question for sure.”
User:This is the bullshit that we deal with, like why can't they ask that directly instead of wasting our time?
User:I think question makers are bored and just wanna mess with us
User:You know a topic is clinically useless when they can’t even come up with a relevant clinical scenario for a practice question
User:Oh my gosh 😂 I actually have MS but what the heck Anyway - a nice easy question
User:you have the opportunity to do the funniest thing at your next appointment
User:Uk healthcare so its like in a yrs time if I'm lucky but then yeah 😁 we can have some fun Also, I don't think my neuro would know the answer to this haha
User:Only two sentences of that question stem were actually needed, save me the exposition 😫
User:They’re running out of scenarios
User:omg i literally had this question last night
User:Where
User:uworld
User:I had it today! 😆
User:This was a funny question 😆😆
User:What test is this?
User:It’s just a UW question
User:Everyone complaining about the stem length but from what I hear the actual exam stems are long as hell, so it’s probs good we get used to this now. Or so I tell myself
Post 12: Which high-yield drugs are in Pixorize but missing from Sketchy?
https://www.reddit.com/r/step1/comments/1ko1js9/which_highyield_drugs_are_in_pixorize_but_missing/
16/05/2025, 10:59:03
4 comments
If y’all have any suggestions on must watch drugs from Pixorize, please do let me know. Thank you!
Comments:
User:Danazol maybe, that one doesn't have a sketchy to my knowledge.
User:Propofol is one drug
User:That’s in the anesthesia section, no?
User:Yeah it’s in sketchy
Post 13: Help IMG Choose: BnB vs. Bootcamp for Step 1
https://www.reddit.com/r/step1/comments/1ko1884/help_img_choose_bnb_vs_bootcamp_for_step_1/
16/05/2025, 10:44:48
4 comments
Hi everyone,
I'm a 5th-year medical student from Chile planning to take the USMLE Step 1 in approximately 6 months. I'll be studying for it alongside my regular medical school coursework, which means I'll likely only have about 2 **hours per day** for dedicated Step 1 prep.
I've done some research and spoken to friends, and my current plan for core resources includes First Aid, UWorld, and Pathoma
Given the differences between my local syllabus and the USMLE content, I definitely want to add a comprehensive video lecture resource ("classes") to build my foundation. I was initially recommended Boards and Beyond (BnB), but I've also been reading a lot about Med School Bootcamp recently and it looks promising.
Considering my very limited daily study time (\~2 hour), could you please help me decide between **Boards and Beyond and Med School Bootcamp?**
I'd be grateful for any advice on which might be more efficient or better suited for someone in my situation, especially in terms of covering foundational knowledge effectively with such a tight schedule. Pros and cons for either in this context would be incredibly helpful!
Thanks so much for your help!
Comments:
User:Bootcamp is better for building foundation and comprehensiveness imo. BnB put me to sleep. You said you'll have 2h/day but then you said 1h/day. Either way, you'll want to start asap and maybe even watch 1.25 speed at least. Also, spend the bare minimum time on your med school coursework and put the extra time into USMLE studying.
User:Thanks! Already edited the post, it was 2hrs. One month before academic year ends here, so it will be fully dedicated to the preparation of the USMLE exam
User:Bootcamp no contest, esp if you only have 2 hours. Bootcamp is meant for bite sized, chunked learning and it will shine here.
User:[https://www.reddit.com/r/step1/comments/1iea7aw/4\_months\_to\_crack\_the\_boards\_a\_detailedlong/#chat-image#lightbox](https://www.reddit.com/r/step1/comments/1iea7aw/4_months_to_crack_the_boards_a_detailedlong/#chat-image#lightbox) this post is a great guide, really helped me get oriented
Post 14: Reschedule or cancel at prometric?
https://www.reddit.com/r/step1/comments/1ko0x1d/reschedule_or_cancel_at_prometric/
16/05/2025, 10:31:05
5 comments
I had to extend my eligibility period and I have filled out the online application but the paper form 183-E is yet to reach ECFMG.
I only have around 9 days to my scheduled appointment.
How can I cancel and then reinstate my eligibility at the prometric?
Will it be done over the phone or should I cancel it via the website now itself?
Comments:
User:Cancel your appointment in the PROMETRIC first , if your college has EWMPS or mail , your eligibility gets extended then after call to prometric and pay the 100$ fee for late cancellation and then they will send you a mail you need to forward that mail to usmlereg@nbme.org then they will activate your scheduling number, you can book your next appointment after that
User:So as an IMG, do I have to send it to this email address or does ecfmg has its own version?
User:Yes as an IMG you need to do it . As you have cancelled your appointment within 30 days, you will not be allowed to schedule another appointment unless you call prometric and pay 100$. Then they will mail you with new confirmation number and ask you to forward that mail to the mentioned email address.
User:Thank you so much!!
User:hi; I'm new to this subreddit and I've been trying to post a question but it keeps getting removed by reddit filter I have no idea why I have questions regarding "step application" if you could answer me I'm an IMG , I have an ECFMG account ready and want to apply for step1 1) regarding the exam fees if for whatever reason my application was denied are the fees refundable, or is it nonrefundable and would I have lost the 1000+ 2) if I scheduled an eligibility period but changed my mind and no longer want to pursue USMLEs are the fees refundable then to be honest I think it's a no on both accounts but I wanna know for sure
Post 15: Help me up my nbme scores
https://www.reddit.com/r/step1/comments/1kn9873/help_me_up_my_nbme_scores/
15/05/2025, 11:12:34
1 comment
I'll be short with it, i have given nbme 23-28, but my scores have been stagnant for a long time even though I've been reviewing to the best of my capabilities. What are somethings you would advice me to up my scores? Any sources I could be using? 2nd June is my date of the exam
My lowest scores: nbme 26 - 65% and highest nbme 28 - 72%
Comments:
User:I mean they’re not bad scores and definitely within passing range? If there’s no glaring below average systems/ subjects then just keep up the pace and you got this!
Post 16: Advice needed!!
https://www.reddit.com/r/step1/comments/1kny25h/advice_needed/
16/05/2025, 08:04:04
9 comments
Hey everyone,
I used Mehlman Medical along with Bootcamp and UWorld while preparing for the CBSE, and I passed. Now I’m getting ready to takeStep 1 in about three months, I am aiming to take it sooner even though the school gave me 3 months.
I’ve already completed all the NBMEs (21–31) and averaged in the 80s on the most recent ones. I was wondering — has anyone here used just Mehlman (PDFs/videos) and UWorld QBank + the self-assessments (UWSA1/2, Free 120) to prep forStep 1? If so, did you feel like it was enough?
Also open to any advice on what I must focus on during the final few weeks. Thanks in advance and good luck to everyone studying!
Comments:
User:If you were honestly averaging 80s(under testing conditions, no repeats) in especially 29-31, I would say take free120/review and full send Definitely don’t wait 2-3months, I think that would do more harm than good
User:Besides the repeated questions here and there, and the repeated HY images there was nothing I have seen before. I took all the forms under timed testing conditions. And no I would not wait that long, I think I will forget so much of what I know Lol, I just don't know why no matter what I do something inside me tells me I might not be ready.. But thank you!
User:80s on NBMEs? test asap and get it out of the way
User:Just take it tomorrow with those scores 😅
User:Lol, apparently the NBMEs are easier than the actual Step, and now my imposter syndrome is having a full on identity crisis.
User:Stay off this sub haha. Is very toxic. Take it, NBMEs are predictable, thats why they exist. It predicts 4 points over the score you got or 4 points lower… so you are in a safe spot.
User:Thank you so much for the reassurance, I appreciate it!
User:It is harder than NBME but most ppl go into it knowing a lot less than you and do fine(me for example) lol nothing else you can really do will prepare you for it, with your great scores
User:I really appreciate that, it honestly makes me feel a lot better. I’m sure you know how med students can be sometimes, always acting like nothing is ever enough, which just fuels unnecessary anxiety. Thanks again for the reassurance!
Post 17: Neuro step 1
https://www.reddit.com/r/step1/comments/1ko063q/neuro_step_1/
16/05/2025, 09:56:31
17 comments
Guys I test in a month and I’ve pushed off neuro for way too long. I really need someone to help me with where to study neuro from. Consider my knowledge at it is at an all time low. Do you think just mehlman pdf audio and Uworld should do the job? If not any suggestions I’ll be grateful ❤️🩹
Comments:
User:Bootcamp + Physeo worked wonders for me. There are so many different resources. If you've time go through the videos and read FA and then do uworld. The score will improve eventually. If you don't have time, I suggest going through mehlman and reading FA and then uworld.
User:Yeah the issue now is I’ve pushed it too far so I’m on a time crunch
User:BnB would do the trick for neuro
User:Sadly for me it didn’t work out ;(
User:Bootcamp saved my ass in neuro. There was so much of it on my exam. It’s dense but useful.
User:Oh really cuz usually it isn’t tested much… did you test recently ?
User:5/9
User:Oh that’s amazing, hoping you pass
User:I’m doing neuro now. I found bnb to be the best resource. Bnb + qbank + Mehlman neuroanatomy and you should be good to go. Also dirty medicine has a few good neurology videos as well
User:I think BnB would be a far reach for me but I will do mehlman and qbank
User:I did mehlman neuro anatomy + the mehlman YouTube Neuro playlist . That was enough
User:This what I’m gonna be relying on now
User:Mehlman doc and his YouTube playlist
User:Thank you ❤️🩹
User:Bootcamp + mehlman neuroanatomy 🤌🏻🤌🏻🤌🏻❤️❤️❤️❤️
User:I will try some videos of bootcamp
User:Firstly why r u testing in a month if you don't know an entire organ system... I pray you know rest of medicine properly or I would Def delay that exam... Anyway your best bet is watch any video resource and read first aid side by side...should take you 5or 6 days to finish neuro... I would skip Mehlman as I used it as an addon and having written step1 and passed it I can tell you Mehlman didn't help.. my knowledge of the system from medschool and first aid did
Post 18: Which among them are high yeild?
https://www.reddit.com/r/step1/comments/1knzgo0/which_among_them_are_high_yeild/
16/05/2025, 09:21:41
8 comments
https://preview.redd.it/xkmjh3riz41f1.png?width=817&format=png&auto=webp&s=7790b1b2854deb2de1f2577d7a8348282ac33c7c
Exceot for maybe imatinib, which rest ones are high yield?
Comments:
User:bortezomib is the highest yeild here , showed up twice on the NBMEs , read the rest if u could but as a tutor i had zero student reported monoclonal antibody on the exam.
User:thanks!
User:Imo , imatinib , bortezomib,palbociclib
User:Thanks
User:I had Bortezomib show up on one of my NBMEs, but that’s really been it tbh.
User:Just these two: Bortezomib and Imatinib
User:Same opinion
User:Erlotinib, Imatinib, zomibs, palbociclib
Post 19: What to revise exactly before sitting for an NBME ? can anyone also suggest me how to do that please !?
https://www.reddit.com/r/step1/comments/1knz03p/what_to_revise_exactly_before_sitting_for_an_nbme/
16/05/2025, 08:57:38
7 comments
I am seeinga lot of wrongs on my nbmes due to forgetting things ! Not that I didn't know them before ? So. how and what do you guys suggest me to do !
Comments:
User:DIrty Medicine videos and Melman PDFs/Youtube playlist of things you know you lack in (ie. pharm/micro/biochem).. Systems are more of an understanding game so go through your previous nbme takes and dissect your incorrects; you'll see the pattern of the focus of NBME forms and with time it will get better.
User:my names are 120-130 corrects on 200 so far I have done 25,27,29 usaw1 I got 228 uswa3 I got 228 are these bad?
User:this is my 3rd [attempt.im](http://attempt.im) jus feeling so low
User:are the 25,27,29 first takes? uwsa1 scores seem decent
User:hey no 3rd,but I don't remember any question .my memory sucks.
User:i will review receptors, because it is very hard to remember them, and if you know it is free points. Also, review lymph nodes, to guarantee 1 q, diabetes drugs, autonomic drugs, ego mechanisms, anatomy of upper and lower limb neuros. There is maybe more, but this is what comes to mind when I compare what was asked in NBME 30 and 31
User:Genetics mode of inheritance, percent of inheritance , risk factors ,
Post 20: Decent NBMEs but scared
https://www.reddit.com/r/step1/comments/1kny5yw/decent_nbmes_but_scared/
16/05/2025, 08:10:32
23 comments
Hey eveyone, I'm testing on June 13th.
UW: 45% with cummulative average of 60%
NBME 26: 78% (3/5/25)
NBME 27: 79% (10/5/25)
I've never done a formal content review from FA or BNB. Only been doing Uworld for last 3 or so months. But I have been doing Anki for Uworld incorrects the whole while.
I've been told to stop NBMEs and get my Uworld to atleast 70% completion but that seems rather impossible. Would appreciate any advice on how to optimize these last 20odd days. Only have 12 days for dedicated, rotations until then.
Comments:
User:Your nbme scores are pretty good without it
User:Should I just focus on NBME content review in greater detail then?
User:Yes like don’t go to uworld now maybe FA but no need for that much uworld
User:Really appreciate the advice, thank you so much!
User:Like u have 12 days so max nbme in these days and go ahead with the exam
User:Actually have 27 days or so, 15 with rotation rest dedicated. Starting to feel the burnout creeping now
User:Have u done FA before
User:Fairly inconsistently through my basic science years, never since I started preparing for step 1 though
User:Okay try to do two three chap a day along with nbme review and focus solely on nbme I spent so much time on uworld and my nbme were in 60s so I don’t suggest going through uworld if your score is already good
User:Understood, thank you again for the help! Appreciate it
User:Try to prepone the exam give it early
User:I would try to get at least one newer NBME and the free 120 in before taking the exam. On dedicated days, doing 100-160 UWorld questions with review is definitely doable. On days you have rotations, just do what you can. you have good scores right now, so just getting some updated practice is going to help you for the big day!
User:I do plan on taking atleast 3 more NBME forms and the Free 120. Would you say doing those 120 UWorld questions on dedicated days takes precedence over NBME and dedicated content review?
User:I can’t say that there is a one-size-fits-all approach to it. Some people like reading first aid but I personally find doing questions to be much more helpful
User:Just to add it seems like what you’re doing has been working well so don’t feel like you need a crazy change
User:Thank you so much for the help, really appreciate it!
User:similar nbmes to u test start June. have nbme 30 31 and free 120 left. currently doing a FA reread , Anki and like 10 Mehlman videos a day. stopped uworld a while ago. dont really know what else to do
User:What percent and average score did you stop UWorld at? Also what cards are you studying on Anki?
User:Why are you worrying? Just keep doing what you are doing. You got almost 80% on NBME
User:Not a rational fear no doubt about that
Post 21: Anki
https://www.reddit.com/r/step1/comments/1knxc4p/anki/
16/05/2025, 07:19:24
12 comments
Hey everyone I wanna high yield Anki decks only as Anking is very big for me and I prefer something more high yield to focus on it like UW FA pathoma sketchy
Should I go for zanki or high yield in Anking only ?
Can I remove other cards in Anking and keep the high yield tag only?
Comments:
User:Try pepper deck for skethcy pharm and micro. Here's the link: https://www.reddit.com/r/medicalschoolanki/s/8Dmrfoobt4
User:And UW?
User:Do you have it?
User:check dm
User:Could you please send it to me also??
User:can i get the pepper deck for micro and pharm 🥹
User:Sure
User:Can i please get the deck as well? I need to ace micro and pharm in a few days pls
User:Can I please get it as well?
User:pepper deck for micro and pharm. pneumosyne for rest of the FA.
User:And UW?
User:Do you have it?
Post 22: Step 1 Harder in Certain Regions?
https://www.reddit.com/r/step1/comments/1knvz7r/step_1_harder_in_certain_regions/
16/05/2025, 05:43:04
5 comments
Guys, I keep on hearing from people that if you do the exam in India or Asia compared to the middle east or Europe, you have a higher chance of failing as it's harder. Is this true? They say that they've had friends who are really smart that have failed in India and people not as smart pass in Europe. I'm doing my exam in Sri Lanka and I'm slightly worried about this. Please let me know
Comments:
User:No that’s not true
User:As long as you're prepared there's nothing to worry about.
User:Not true, probably more correlated with English as a first language or increased familiarity with English.
User:No. Don’t be too naive with stuff like this
User:That’s a BS. i live in Florida and took the exam yesterday. It was the same as people are saying i mean since april.
Post 23: Examn result
https://www.reddit.com/r/step1/comments/1knppm0/examn_result/
15/05/2025, 23:09:40
6 comments
Did my exams 05/09 do you know when im gonna receive de results?
Comments:
User:Most likely the 21st! I took it on 05/02 and they didn’t release our results 🥺 neither for test takers 05/01
User:Oof. Also tested 5/9 and if you guys didn’t get scores yesterday then we will most likely get our scores the 28th. But hoping for best case scenario and that we get our scores back on the 21st. Fingers crossed.
User:first of all good job on your efforts 😁 , in my case i did it in 23.4.2025 and the results were out in 7.5.2025 , as far as i know , since the day of your test , you must go through 2 Wednesdays to get the results , it means most probably it will be out in 21.5.2025 , i wish you the best if luck , fingers crossed 🤞🏻🤞🏻
User:Very likely the 28th. End of week takers (I also took 5/9) are about 2.5 weeks.
User:any idea for people who test on Wednesdays?
User:2 wednesdays from when you took it.
Post 24: First aid cases
https://www.reddit.com/r/step1/comments/1knngn5/first_aid_cases/
15/05/2025, 21:12:20
1 comment
Can someone help me find google drive or pdf of First aid cases book?
Comments:
User:Try Libgen
Post 25: im ready?
https://www.reddit.com/r/step1/comments/1knmw0c/im_ready/
15/05/2025, 20:43:40
6 comments
My total scores are here:
UWSA1 - 190 (54%) 11/15/2024
NBME 26 - raw 65% 02/26/2025
NBME 27 - raw 68% 03/11/2025
UWorld first pass - 53% 03/10/2025
NBME 28 - raw 75% 03/28/2025
UWSA2 - 220 (64%-70 EPC) 04/16/2025
Old free120 (2021) - raw 78% 04/16/2025
NBME 29 - raw 66% 05/06/2025
NBME 30- raw 73.5% 05/13/2025
all nbmes offline
My test is in may 27th
Do you think that Im ready to take it? Be honest
I'm sorry if I offend anyone, I know that many people don't like to ask these questions and mention their scores, I only do it to relieve my anxiety and I'm open to any kind of advice... also I don't know what to do with the days I have left to study, I would like to make the most of them.
Comments:
User:Hey, you're more than ready. I had lower scores than yours and got my Pass yesterday. Just believe in yourself, you'll do great!
User:I think you are ready
User:Looks like it. Just try and get >65 on the free 120 and you should be good to go
User:Looks like you're ready now, do a light review and you've got this
User:You look ready. Just do 1 nbme ie 31 online or at prometric centre to have a feel of the exam. Real deal has longer stems hence a timed simulation helps preparation.
User:Simple truth: YES. Free 120 and NBME scores honestly don’t mean much. The actual exam feels different, even if the content is the same. What truly matters is doing UWorld properly and building a strong, deep understanding of the material. That’s what gets you through this exam ,not practice test scores.
Post 26: Anxiety management technique
https://www.reddit.com/r/step1/comments/1knmez6/anxiety_management_technique/
15/05/2025, 20:20:29
5 comments
Hello guys,
I'm suffering a lot by doing usmle.Most of the times of day I'm suffering anxiety.How do you manage yourself & keep focus on study?
Comments:
User:Go check out Healthy gamer videos on YouTube. But basically- practice detachment. Next time you’re anxious, say to yourself “My Brain is telling me I’m anxious.” Also CBT and an SSRI maybe
User:Thanks for your nice advice!
User:First of all, make sure you're eating and sleeping well. Look up breathing techniques and practice them. As much as you can focus on the tasks you need to do daily instead of thinking of the exam, this will all pass andkeep up the good work!
User:Thank you! Yeah I practice breathing technique.It works.though it gives temporary relief, I do it when I become more anxious.
User:Im having the same issue. Started taking bus-irons took an NBME and scored Lowe, from 65% on 26 to 61% on 27 . feeling like shit
Post 27: Hello everyone! I need some help. I’ve been doing UWorld seriously, but my scores are stuck around 50–60%, and for Cardiovascular System, I’m getting 40–50%. I’ve already completed First Aid and watched all the Bootcamp lectures, but I still feel weak in CVS. Any help would be appreciated!
https://www.reddit.com/r/step1/comments/1knllwi/hello_everyone_i_need_some_help_ive_been_doing/
15/05/2025, 19:42:25
2 comments
Uworld scores
Comments:
User:In CVS you need to make sure you actually understand the pathophysiology of all the major topics, it really makes a difference. Try switching to Amboss or NBMEs to further test your knowledge too
User:Mostly, i know the pathophysiology of the diseases, but when i solve the questions from uworld at that it does not click on my mind And 1 more thing, after completion of systems what should i do, wrong uworld or complete 2nd pass of uworld?
Post 28: immunology
https://www.reddit.com/r/step1/comments/1knir9g/immunology/
15/05/2025, 17:39:47
1 comment
Are the complicated topics in immunology really important for Step 1?
Do I need to know all the details thoroughly, or is it enough to just understand the basic concepts?
Comments:
User:Must know ILs, hypersensitivity reactions, TNFs, name of macrophages for various sites, B cell / T Cell + combined deficiencies, digeorge, HIV impact on immuno lines, common immunosuppression drugs for transplant
Post 29: date close dont know what to do please guide
https://www.reddit.com/r/step1/comments/1knilh5/date_close_dont_know_what_to_do_please_guide/
15/05/2025, 17:33:13
6 comments
This is my second attempt ive failed once
I got 47% in nbme 31
Got 55% in nbme 30
Got 62% Free 120old
And a week away from my exam date got 47% in new free120
Im burnt out really wanna give it people are telling me to extend it
And what should i do ive studied almost everything havent watched all mehlmans videos
But have done his immuno arrows endo and some other docs
81 percent done with Uworld second pass
Average is 49 percent
Im thinking of extending it for 3 weeks
can i manage in 3 weeks and since ive done everything what exam will i do access myself. What sources should i use ? If i should extend how many days should i extend im tired of spending money and time
Comments:
User:Before adding another resource hoping some external source will be your saving grace I’d do a self-analysis of what your error types and patterns are and see if you could make progress addressing those systematically. As a random example, If you find that you’re missing questions consistently because you fixate on a buzzword and tunnel vision your way into wrong answers, more content review won’t help in that. I found the most valuable question mistakes are the ones where I knew the information but reasoned my way to another answer- those require intentional training with self awareness to undo habits, not more learning. Don’t know if this applies to you, but if I were a betting man I’d say your problem’s not likely to be that you haven’t seen the material.
User:Thats the issue i know the material just dont know how to tackle the questions i end up doing some of the very hard ones and end up screwing over the easy ones
User:tbh then if content isn’t the issue, you don’t need to watch/read more content. i used UW and AMBOSS qbanks. AMBOSS imo helped me a ton bc the explanations for each answer showed WHY it was the wrong answer way better than UW. maybe you could try out AMBOSS (they have a free trial i think)?
User:Okay i can look into it
User:it sounds like ur biggest issue may be approach, it might also be worth seeing if u can find any resources on how to approach a question and what info to look for etc! good luck!
User:I think first of all… postpone. If you have time, dont think about a date right now. Take a week off, completely off. You are burnt out. Relax Then: make a strategy; review which are your weak areas and mistakes and tackle that For example: if your problem is lack of content: take a resource that would help with that (i recommend bootcamp) If you problem is pattern recognition of qs, then make more qs If the problem is reasoning thru the qs, then watch mehlman videos Dont stress if you have done what everyone says and you are not seeing results, this is trial and error unfortunately. Find what works FOR YOU
Post 30: Fastest Way to Get Through Sketchy Micro and Pharm?
https://www.reddit.com/r/step1/comments/1knhuf3/fastest_way_to_get_through_sketchy_micro_and_pharm/
15/05/2025, 17:02:17
14 comments
One month left till my exam and I am still extremely weak in micro and pharm. I’ve only finished bacteria videos and Anki. Is it possible to study all of micro and pharm in a week? Should I just do UWorld?
What would you guys recommend?
Comments:
User:Just gotta bang out the videos back to back and become a sketchy fiend. If you want to finish in a week, you more than likely will just have to send all day everyday doing just sketchy because you also have to review as you go. Reviewing frequently is crucial because it’s very easy to get lost in the sauce and forget stuff easily. If you can afford the time to do it, it’s not impossible but you have to be very dedicated to it.
User:Sketchy anything is overkill in todays age
User:Nothing is overkill, better to be over prepared than under prepared. It’s no joke that we had 73% passing rate last year, and it’s only going to fall given the forms people have experienced in these recent months. It’s still doable, but surely tougher than what we had before. Edit: passing rate for IMGs* Even for US MDs, it has gone from 95% -> 87%.
User:Wrong. studying drugs/bugs/disease that arent tested isnt being over prepared its called wasting time
User:but here's the thing, we don't know what drugs/bugs/disease aren't tested. we only know what is high-yield and what is low-yield. sure, if on a time crunch, one could focus only on the high-yield stuff to pass, but if not, there's nothing wrong with trying to focus on both. it's not really a waste of time if it's helping you cement your knowledge base + if you think long-term. and sometimes, the low-yield stuff comes on the exam as well.
User:Sketchy’s the single most valuable thing I did preclins. It’s constantly paying off and it’s stuck in my memory still
User:Not talking about preclinical, this is just for step 1 prep. Sketchy will cover the bugs yes but there is WAYY more stuff on there that u dont need for a P/F exam
User:But the foundation carries over into clinicals and step 2. I kept up with it for IM and got a ton of free points because of it. It’s pretty well condensed into relevant info. Especially useful getting pimped cuz you reach the answer way quicker with the visual recall. Sketchy is goated. If it works for you you should absolutely do it
User:If sketchy is overkill, what’s the best way to study micro or pharm in your opinion? I haven’t started dedicated but I was planning to set two weeks aside just to bang out sketchy pharm and micro videos.
User:Mehlman micro + pharm
User:Hmm ok I can look into it!
User:Screenshot the full sketch at the end, copy and paste into notability, and annotate the stuff they're saying and put a red box around AEs and green box around indications. Thank me later.
User:Sketchy is most useful when you do a little bit a day. Brute force watching all the videos in a week would not help me tbh, but I think picking some high yield sections could be useful. Like the sympathomimetics
Post 31: NBME 28???
https://www.reddit.com/r/step1/comments/1knhndw/nbme_28/
15/05/2025, 16:54:17
5 comments
I had a 10% drop from form 27 to 28
This was my second nbme and i am devastated !
The exam felt soooooooo vague
Are the forms 29-31 also similar ??
Comments:
User:i heard that nbme 25 and 28 are the worse so idk try doing 25 and 26
User:Actually it just depends on your strengths and weaknesses.
User:What was your score
User:Thoughts on going through 20-31 (taking them and reviewing them well) except 28 and then taking it ?? would there still be a drop expected or you get used to the pattern at that point? it's my last untouched nbme saving it for the final 2 weeks with new free 120. Can't push back exam any further. Worth to still take uwsa2? or would it just bring down the momentum
Post 32: Flashcard Friday Alert
https://www.reddit.com/r/step1/comments/1knhj5f/flashcard_friday_alert/
15/05/2025, 16:49:24
2 comments
Thank you for the wonderful response on my flashcards. I don't have decks yet but I'm working on creating them!! Comment below - which system you'd want a deck in and I can work around it 🤍
Tomorrow marks the beginning of Flashcard Friday : where I share a secret flashcard with my subscribers on fridays through newsletters 🤍
If you wish to receive my newsletters, you can subscribe here - https://sendfox.com/usmlelittles
Select subscribe on the top right corner on the portal and confirm the subscription on email - to make sure you're on my email list.
If you enjoy what I create - You could find more of my work on my [youtube](https://youtube.com/@usmle_littless?si=XzCToklLTj7CqqNX) and [instagram](https://www.instagram.com/usmle.littles?igsh=bGNuYTJtaTIzb2pk)!
Lastly, I'd love to invite you to join the usmle littles reddit community where I'll post updates going forward - https://www.reddit.com/r/usmlelittles/s/NficL5nhFi
Thanks and welcome!!🤍
Comments:
User:Neuro msk
User:Okay!
Post 33: PASSED STEP 1!!! (IMG)
https://www.reddit.com/r/step1/comments/1knh97x/passed_step_1_img/
15/05/2025, 16:37:54
24 comments
I got my pass yesterday (may 14th 2025) and decided to write this reddit because literally without them I wouldn't have made it, all the countless reddit threads that I read of people who were in the same situation as me were my comfort and helped me keep hope so I thought the least I could do was write one myself of my own experience for whoever it may help. I had been religiously studying for step 1 for about 2 years, there were lots of obstacles along the way so I didn't end up taking it sooner like I wanted to but oh well. my biggest obstacle was definitely myself, mentally I wasn't prepared, I suffered from anxiety and was terrified of taking the test but with some external help and a support system I managed to push through and finally and actually believed that I could do it. that plus the severe burnout I was going through which is why I also decided not to push back anymore because I knew I was unable progress further or do better than I already was, I had officially reached a plateau and knew that if I had even months more of studying it wouldn't make a difference. I took all the nbmes but my last ones were **nbme 29: 57% nbme 30: 58% nbme 31: 64%** then I retook nbme 26 since I had taken it 10 months prior and didn't remember a single question (maybe subconsciously) and there were actually a few questions that were not in the nbme 26 the first time I took it because apparently nbme updates the forms or something like that I don't really know, anyway got a 69% (48% the first time I took it) also retook 28 and got a 75% (got a 51% the first time I took it which was like 7-8 months prior) since I had ran out of nbmes and needed more reassurance that I was ready I took nbme 25 which I had read online that despite it not being on the nbme website anymore it is still a valid nbme because it's one of the newer ones that was made after it became pass/fail so I took that one and got a 72%. took uwsa 1 and got a 61% which is a 222 in 3 digit score and 196 is the passing score threshold so that was very good for me. got a 51% on uwsa 2 and 50% on uwsa 3 which are way harder. It was definitely a bummer but didn't take it as a predictor because honestly it isn't, uworld doesn't even make the actual exam so if you have nbmes telling you that you pass, why should you listen to the scores of exams that are known to be harder and are not even predictors which nbme themselves have said to use nbmes solely as predictors. So that's why I did, I also knew a few people who failed their uwsa right before the actual exam and all passed. but I do definitely recommend to take them all for PRACTICE because at the end of the day uworld is the best learning tool and who knows maybe I got questions right because of hard questions I saw on the self assessment. Anyway, I wanted to confirm that I did bad because uwsa are harder and if it were an nbme I would've done better so I took an old nbme (form 24) to confirm my theory and I got a 74% or 72% so I carried on with my plan to take my exam in a week and a half. I took the old free 120 to continue building confidence and reassurance and got a 78% . took the new free 120 2 days after and got a 63%. I wanted a 65 or more to go into the test confident and relaxed so when I got the 63 I was a little like fck but I didn't even consider changing my exam because 1) between 63 and 65 there isn't a big difference, 2) I was getting consistent scores in all self assessments I had been taking new or old so I felt if I passed with a 63% that meant I was able to get that score on the actual exam and pass 3) when I reviewed the new 120 I had gotten almost all ethics and biostats questions wrong if not all so without that I would've gotten like a 68%-72% so I obviously wasn't gonna move my date just for biostats and ethics when that's a small amount of the exam, I could just focus on that until my test day and the actually high yield stuff I was solid. so my test was in 2 days, the day after I focused on ethics and biostats only and the day before the exam I just did rapid review all day with a friend asking me questions and I was solid. so I was confident and knew that I couldn't do more, if I were to fail I knew that at least I did everything I could to pass. soo I took it, lots of ethics but literally no biostats (only 3 questions my whole exam) came out the test not knowing how to feel, cried in the parking lot because I thought I had failed and was scared, tried to distract myself for the next 2 weeks but it was hard and torture and at last I passed!!!!! lots of reddit comments were true so it was honestly all thanks to reddit, my years of hard work that only God knows what i've been through. I had lots of faith that I was gonna pass because I had been praying for this forever and I knew my blood sweat and tears would pay off. Good luck and hope this helps to anyone who needs it.
Comments:
User:I feel like im reading my story! Congratulations!! My exam on 26th wish me luck
User:I'm also taking the test on the 26th. Good luck with the exam
User:IMG ? Hey , im planning to give exam in next 3 weeks , did u take all nbmes ?
User:Yes IMG,still didn’t do 30/31
User:Can i dm you ?
User:Sure
User:Im done with 27,28, Im in need of accountability partner, for motivation
User:Same, mine on 31th
User:Mine too on the 31st
User:Much congrats 🥳 Dod u use Mehlman’s PDFs at all?
User:Congratulations! Hard work pays! Are there a lot of histo slides on the test? Can you plz let me know?
User:tip to improve nbme score please mine is not improving and its really bad like 50%
User:congratsss
User:I needed this thank you lol. I just got a 59% on my uwsa2 and I was like wtfff. Reading this made me feel so much better
User:Congratulations !!
User:Not yet… Still remaining 30 and 31
User:Was the exam based on nbme concepts?
User:There is definitely a lot of repeated concepts tested from practice exams
User:Congratulations!!
User:When did you take NBME 31? Congratulations!
User:Congratulations… god with us
User:Congrats! Testing next week, where do I find the free 120? I’ve heard the USMLE website but I couldn’t find it. Is it called something else?
User:You've finally passed the first step.
User:How does it feel to have passed step I and not know how to write up a proper post
Post 34: How to encounter uworld
https://www.reddit.com/r/step1/comments/1knh8hx/how_to_encounter_uworld/
15/05/2025, 16:37:03
1 comment
Hi,
I finished Sketchy micro and the according Anki deck. I feel very confident about the topics and never skipped a day with my reviews.
Now I did 2 micro blocks and only scored 45%. I am so disappointed.
Do you have any advice how to encounter uworld questions? I can’t believe I scored so bad.
Test day is in 2 months.
Thank you guys
Comments:
User:Don’t worry, there is always a difference between learning a resource and applying it as your clinical knowledge. After a few block, you’ll see improvement for sure
Post 35: Ethics?
https://www.reddit.com/r/step1/comments/1knh46w/ethics/
15/05/2025, 16:32:01
5 comments
Please someone help i got exam in 8 weeks
And dont know how to even solve a single ethics question, and hearing from recent test takers it is a very important part of the exam now.
Help?
Comments:
User:Dont waste your time watching videos. Just jump to qs. The qbank you have. Either uworld, amboss, bootcamp. Also read mehlmans ethic
User:Yeahh youre right I watched dirty med like every says on reddit and only got just the basic overview Still get half questions wrong Ill do uwolrd and amboss and mehlman pdf Do you think this will do? Are these enough?
User:Yep. They are enough for ethics. U world ethics is gold! I haven’t done amboss, but everyone says they are good as well so i think that would do it. Videos for ethics are a huge waste of time, even from resources like B&B or bootcamp 🥲 mehlman is good too since he gives scenarios as well soo… the key is to recognize patterns, and you do that with qs.
User:Imo going through uworld and/or mehlman pdf doesn't necessarily guarantee correct answers on the real deal, but just prepares you to not be fazed by wonky ethics questions...and there's nothing else that you can do besides that Because it can be very objective
Post 36: Sketchy pharm,fuck it
https://www.reddit.com/r/step1/comments/1knfkcc/sketchy_pharmfuck_it/
15/05/2025, 15:28:51
35 comments
I can't stand her voice anymore ,just going high and low every second .The moment she tries to make a joke ,I lose my temper,How can I complete sketchy pharm without even listening to her voice?
Comments:
User:Read the pdf
User:I just do the Anki cards, the videos aren’t necessarily a requirement, they just make the cards easier the first few times. Also, the pdf is good.
User:Which pdf?
User:https://drive.google.com/file/d/1B-DIOFhk8pH46VO804ymmaVd-c102od0/view
User:Do you have the pdf for the bugs by any chance?
User:I am sincerely jealous of people who are able to learn from Sketchy. The only one that was okayish for me was Micro.
User:How did you study pharm then?
User:The way that people did before sketchy existed. Just somehow look at it enough times for it to stick
User:It’s too high yield, took mine today and was a shit load of pharm
User:Pharm is HY not sketchy
User:So you’re saying sketchy doesn’t cover everything you need to know?
User:Did you find sketchy pharm good enough to answer most of them?
User:yes
User:This is all I needed to hear. I will suck it up and just watch it all
User:andrew is the only narrator I like
User:she’s so triggering. cannot stand her. i just do the anki cards whenever it’s her video
User:I got the MSB pharm flashcards if you're interested!
User:yess please dm
User:What’s MSB? Bootcamp?
User:medschoolbro
User:Ooh ok thanks!
User:please send me
User:Interested as well
User:Plz send me too! Thanks a lot 🙏
User:Me too please 🙏
User:same omg 😭😭😭😭😭
User:Lol same problem with all sketchy shitty content
User:Never worked for me. BnB pharm is pretty good you can try that along with FA
User:Word
User:Dirty med + Mehlman was enough for me
User:I used pixorize for pharm.
User:do have the link for pixorize please?
User:Look up medicalstudyzone or Afratafreeh on google, I switch back and forth between the 2 sites
Post 37: Help, med school bootcamp learner here
https://www.reddit.com/r/step1/comments/1knfecn/help_med_school_bootcamp_learner_here/
15/05/2025, 15:22:01
4 comments
Anyone who has been avidly learning from med school bootcamp for step 1, was neurology pdf with videos enough or did you reinforce neuroanatomy from their videos or any other resource, their neuroanatomy is school based not high yield step 1 based so I second using it as I'm short on time.
Also I cant find a section on neurocutaneous disorders ?
Comments:
User:I did just that and b&b for strokes and eye stuff- I don’t think the have neurocutaneous videos. I liked the sketchy path for that bc it was just memorizing the difference between the two and it’s a short video.
User:So did you cover their neuroanatomy too ?
User:We had separate neuroanatomy lectures in school, and they were so bad. I watched bootcamp videos only for what my school was covering, bc neuroanatomy is not needed for step 1 tbh. The Mehlman neuroanatomy deck is enough to get the images right, conceptual neuro is more important. (ps. I'm a huge noted anatomist girly and loved his neuro videos on the brain, circle of wills, CNs, and tracts really underrated)
User:Thankyou so much, I needed this advice
Post 38: What to do before exam
https://www.reddit.com/r/step1/comments/1knemyd/what_to_do_before_exam/
15/05/2025, 14:51:29
3 comments
What do you recommend I do during this period? I’ve reviewed First Aid 3 times and I’m doing Anki for certain subjects like Biochem and Immuno.
Should I solve random UWorld questions every day from now until the exam (I’ve already gone through UWorld once, but system-based, not random)? Or should I just review my weak points from First Aid and focus on them thoroughly and solve questions related to them? I don’t want to overwhelm myself too much or end up losing what I’ve already studied.
Comments:
User:Retention is better through random timed mode questions rather than going through just text. 2nd pass makes huge difference
User:Do i need to finish it again?
User:Yes daily 3 blocks and another q bank like bootcamp. Question are better for retention than just reading the book. From now on always do it timed and random mode.
Post 39: Recommended NBMEs
https://www.reddit.com/r/step1/comments/1knd0go/recommended_nbmes/
15/05/2025, 13:46:51
2 comments
hi, which NBMEs are recommended for step1?
Comments:
User:20-31 but if you’re short for time 25-31 is enough
User:Thank you!
Post 40: Hyguru nbme videos in the beginning of prep?
https://www.reddit.com/r/step1/comments/1kncxvq/hyguru_nbme_videos_in_the_beginning_of_prep/
15/05/2025, 13:43:52
1 comment
Should I watch Hyguru nbme concepts videos on YouTube before taking any practice tests/NBMEs (in the beginning of preparation)? Is it high yield?
Is it wise to go through Mehlman subject PDFs, arrows and risk factors as well in the beginning of preparation and before taking any practice tests?
Thanks.
Comments:
User:Why would you? The self assessments are meant to test your readiness for the exam
Post 41: Help
https://www.reddit.com/r/step1/comments/1knb0fc/help/
15/05/2025, 12:25:50
12 comments
My nbme scores are in 70s nbme 28 and 29 i took nbme 30 today and i scored 55 percent dont know whyy
Comments:
User:How did you study? I got 58% in NBME 30, and after 70% on NBME 26
User:I don’t know what resource you are using, but maybe it inflated your score on the last NBMEs
User:Right after reviewing 29 i did 30
User:But did you see you have any lack in knowledge or were you tired?
User:But NBME 30 is harder. Vague questions, some non classic presentations
User:Is it similar with real exam ?
User:I am crying literally 😭
User:When i open first aid for the topic i know that Topic and details … physically I dont feel that i am tored may be my stress hormones are high this time
User:I dont know if this applies to you but several people in my med school including me all had significant drops in our nbme 30 score, the ethics/biostats is super hard, but that was our experience. Take another nbme soon and i bet you score goes back up.
User:me too. NBME 30 and 31 are really hard. They deviate a little from the tradicional exam
User:Yeah I did the worst ever on the ethics/biostats on 30
User:Another factor could be burn out - you mentioned you took 30 right after reviewing 29? You might want to try putting some time between your NBMEs to not only give yourself some time to learn the material but also to prevent burn out
Post 42: Applying for 2nd attempt.
https://www.reddit.com/r/step1/comments/1kn9yt7/applying_for_2nd_attempt/
15/05/2025, 11:43:31
8 comments
Hi, is there any thing else that we should mention in the form or the same as 1st attempt if someone applying for 2nd attempt.
Please let me know if any .
Comments:
User:How were yous SELF ASSESMENTS in your first attempt? Would help me a lot
User:62 to 72 for 25 to 31 NBMEs
User:I was not feeling well at test day that was the reason otherwise, exam is very much doeble
User:Oh, sorry to hear that man, you have great scores!
User:Good luck next time, and thanks for the info
User:Wow, 72 on NBME 31? Was there any problem on test day?
User:Lack of sleep so mental exhaustion and brain fug at test day
User:J's got F by one mark unfortunately,and that's literally crushing me
Post 43: Need help to categorise where to start
https://www.reddit.com/r/step1/comments/1kn98p7/need_help_to_categorise_where_to_start/
15/05/2025, 11:13:12
1 comment
Hi , so I just started to prepare for step1 but seeing the syllabus I have no clue whi h units to pic up first . Can someone pls guide . What would you do assuming you being a med student in 3rd prof who doesn't know anything and have to start from basics, do the q bank and take the test within 5 months time
Post 44: Mehlman Risk factors PDF
https://www.reddit.com/r/step1/comments/1kn8g3a/mehlman_risk_factors_pdf/
15/05/2025, 10:38:52
4 comments
Is it really HY? I cant wrap my head around it. Theyr all so confusing n everyone over here has recommended doing them. Those who took the exam, was it heavy on risk factors??
Comments:
User:ABSOLUTELY
User:MI, Stroke you name it
User:Yes, risk factors are HY for step. However, never used mehlman. Imo the NBMEs do a good job of letting you know which ones are most important. The answer is usually smoking, alcohol or age…
User:Gave exam yesterday. It felt like an exam of risk factors
Post 45: immunology?
https://www.reddit.com/r/step1/comments/1kn7xj7/immunology/
15/05/2025, 10:15:15
2 comments
how to do immunology? whats high yield? whats the best resource?
Comments:
User:Mehlman immuno playlist video. Go check out my post
User:Indeed, Mehlman by a longg shotttttttttttt
Post 46: HELP!
https://www.reddit.com/r/step1/comments/1kn7uu4/help/
15/05/2025, 10:11:52
5 comments
I took the exam before couple of days and i cant help myself. I am so stressed worrying about the possibility of failing especially after i remembered 10-15 easy questions i got wrong!!!
My imposter syndrome is killing me
Everybody told me that i will go out of the exam and say that but idk i can help my self.
I studied from FA for 11 months and i revised it 3 times .
Did 3 nbmes and free 120 (new).
NBME 28: 79
NBME 29: 83
NBME 31 before 3 days: 79
Free 120: 78 2 days before the real deal
My scores were good so I didn’t panic while taking the exam but i cant remember how i did i flagged 10 Qs every block and i think there were more that didn’t flag em.
The stems were so long and didnt have much time to review my flags
I want to trust the process and my scores but basically i am worried that the NBMEs are not representative because it was not the same.
Comments:
User:Those scores are incredibly high
User:But apparently FA is very important
User:Yeah its very important but time consuming
User:I am in the same situation as you , i scored 87% on nbme 31 but i made multiple mistakes in the exam some of them are silly and i am anxious now
User:Hope we both pass and leave all of this behind 🤞
Post 47: For cell organelles is dirty enough
https://www.reddit.com/r/step1/comments/1kn6zj4/for_cell_organelles_is_dirty_enough/
15/05/2025, 09:30:31
3 comments
Will I be able to solve questions with that 1 Vedio or any other suggestions
Comments:
User:Depends on your background knowledge. How about do some uworld biochem questions and see how it goes, and if not can supplement with another resource, such as boards and beyond and first aid
User:Ok
User:No , read it from FA
Post 48: Passed
https://www.reddit.com/r/step1/comments/1kn5dk5/passed/
15/05/2025, 08:01:59
25 comments
No content found.
Comments:
User:can somebody gimme some karma? need advice on a fail but cant post due to it getting filtered out? :/
User:Much congrats 🥳 What resources did u use? How many questions a day on average? What was ur NBME score range if u don’t mind sharing? TIA
User:Thank you Nbme 26-69% 27-71% 28-72% 29-74% 30-72% 31-73%
User:Hey congrats man! I’m also waiting for my result .I really hope I passed
User:Congrats bro!!
User:Congrats !!
User:Congrats!!!
User:How long did you study for ?
User:It took me roughly around 7 months bro
User:Tell me plz how long does it take ..
User:Around 7-8 months
User:What is your score range on the NBME exam, if you don't mind sharing?
User:Nbme 26-69% 27-71% 28-72% 29-74% 30-72% 31-73%
User:How were your NBME?
User:Nbme 26-69% 27-71% 28-72% 29-74% 30-72% 31-73%
User:NBMEs ??
User:Nbme 26-69% 27-71% 28-72% 29-74% 30-72% 31-73%
User:Congratulations ! Great triumph ! With more to come for sure !
User:Thank you buddy
User:Which area is more asked? Is Biochemistry a lot on the paper or is pathology more than the rest?
User:Pathology is the most asked subject
User:Thanks
User:Congrats! Was your test heavy on ethics? Do you recommend amboss ethics or Mehlman ethics? Exam in 2 weeks
User:I strongly recommend amboss and mehalman
Post 49: Got the P - ?another generic write up with maybe some nuggets of wisdom
https://www.reddit.com/r/step1/comments/1kn47me/got_the_p_another_generic_write_up_with_maybe/
15/05/2025, 06:47:11
12 comments
I was fairly sure I'd passed after the exam since it felt exactly like a regular NBME and was gonna post as exam impressions but didn't wanna count eggs before they hatched with mediocre scores. But had most of this in mind with little change, before, during, and after results. Result release took about 12 days for me.
Anyway, I wanted to say that the exam felt *exactly* like a regular NBME practice test. If you spun me around quickly and told me this is an old NBME I would've believed you. No joke, no difference in stem length, no difference in reasoning, questions asked in exactly the same manner bar one field*. They don't change the concepts at all, it's still the same things asked in a different way. There's some things that they love to test on an will pop up every exam, e.g. like something about vWF.
I think just going through NBMEs and making a note of what pops up most frequently is the way to go. If not just doing like 70% of NBMEs in the last 2 weeks is a good strat imo to intuitively recognize it. So I seriously don't understand wtf is up with all the "omg my form was so different posts" maybe I just got lucky, maybe people exaggerate, take everything you read with a grain of salt.
Anyway, enough meaningless post exam insight cause that shit is useless. In fact I had in mind to make this post during studying just about the things that I thought were worthwhile and what else wasn't and so a lot of these things were jotted down by me as I was studying and now can reflect back a bit on the actual exam and link some of it.
First I'll give a shout out to what I think is the best step 1 post out there:
https://www.reddit.com/r/step1/comments/ub7lk4/from_55_to_84_nbme_with_3_weeks_dedicated_no/
All of that advice is gold and holds. Ill add some quick things that are just my personal experience and think highly of/worked for me. As well as some quick tips I found useful. YMMV. I will dispense this advice now:
Quick trix:
1. Read the damn question stem properly. No, really, if you think it doesn't make sense, skip and get back to it.
Addit: skipping is a key skill, you need to know when to use it (often times quick) and it also lets you try to solve the q subconciously in the background. It's like sleeping on something overnight but in real time; it oftentimes give you a new perspective and lets you figure it out/see a critical point missed on second pass. Doesn't always work but I would say skipping gave me 4-5 aha moments per block in both practice and the real deal that I would've fallen for NBME gotchas or by reading the q like a spazz and missing a key detail on first pass. I would say a good 10-15 question skip per block should be standard and lets you rack up some easy qs/confidence boosts in the meanwhile.
Worst part is to dwell on a question forever, not come up with the answer and then you tank a whole section and maybe more because you were too hard headed to skip and ran into time trouble. Seriously go back and see how much of a statistical difference it makes whether you tank for 20mins or 2mins on a question you don't know, you'll see there isn't any.
2. The two similar answers trick. Guys this is a NBME pattern on questions I noticed on the medium tough to tough questions. It doesn't hold water 100% of the time but it should raise your spidey senses.
Often times you will have some random question where it had two similar findings but neither are correct.
Uearth example:
Thought patient would have wet status from the stem prior to answer choices. Two answer choices I have are bilateral crackles at lungs, elevated JVP but this raises red flags to me so I reread the question and the third answer that I wouldn't have picked makes more sense now. Normal appearing volume status. SIADH with transient subclinical hypervolemia.
You might say pft, yeah right. But exact same scenario popped for me on the very first goddamn question I got on the real deal. I managed to do a second take and fight knee jerk reaction on this.
PVD diabetic patient, answer choices on clinical findings, weak femoral pulse, weak popliteal pulse (answer selected instantaneously but hmm kinda similar) but then I see hairlessness of lower limb. Well shucks, made me rethink, obv this is a much more common finding.
You can go through the qbanks and see several examples of this, when two answers are very similar maybe re-read stem again or look out for gotchas.
3. * The communications questions. This is the different section y'all. Only place where I felt what all the fearmongering that happens on this board was relevant. I did not heed the warnings y'all. Do not do this mistake, there was legit easily 6 comms questions per section (they throw in some ethics intermeshed w them so legit 15% of your exam is this).
I thought, hell I'm scoring 90%+ on uworld and NBMEs on this (this, biostats, and psych were easily my best sections). I thought it was just some more fearmongering with the wtf comms questions. It was not. This is some fk'd up shit now, I'm not even sure if I scored above 60%+ on these gun to my head.
So this is the easiest score booster you should work on imo. They ask less than 1/3 of questions in a relatively straight forward manner like they do in NBME forms/uworld. Now they ask what you do in step 2-3, now they ask to integrate to ethical principles, it's some crazy shit. Like they'll give you the stem, say patient has been consented properly, his feelings have been validated etc., all the easy free points you could've got before are gone and they hit you with what do you do next. And you have 3 reasonable answer choices that you would do in probably no particular order. But you better make goddamn sure you know what order you need to say things in and also integrate the ethical concepts for this exam (mainly the big 4).
I still have no clue what some of these were, hell if you told me I tanked and I got <50% and missed all my 50-50s from what was a good 90%ish baseline on them, I'd believe you. Shit's completely revamped, the fearmongering on this was right!
4. This exam is very much akin to CARS MCAT section with the biology data reading and psych section questions. I think that's why a lot of non native English speaking people struggle, and they have went even harder on this angle the last 4-5 years. I think questions are badly worded, or gotcha type on some on purpose.
Resources:
Disclaimer, IMG, mid year 4. So this may not be relevant to most starting from scratch.
Yeah yeah, pathoma 1-3 gold standard for most things. But I don't think it will necessarily net you many free points, just give you a good baseline.
Other pathoma goodies, 4-5 (imo on par with 1-3 prob even better for free points since I think 1 is fairly basic, 2-3 are the main ones).
Cardiac also amazing and short, don't get why it doesn't get much love. Endocrine (particularly thyroid) also great, skin and breast also good and very short.
B&B - don't really like them, CNS one is great tho.
Prob the top resource out there is Goljan fluids - prob a good free 5+ questions in this one. Single best resource out there for 2hrs imo. Cardio one is good too (I think that's the one where he explains the shock forms, but has an hour or so of spam within it, still great but longer at 4hrs). Endo, another 2hrs of magic, hell the ~15min of Daddy Goljan going over PTH is worth more than all of Mehlman arrows imo.
Sketchy micro - def worthwhile to do, legit as free points as it gets if you know them well.
Sketchy pharm also good but way more dense and less bang for your buck. You should know the HY ones tho but the effort to learn this properly is 2-3x that of sketchy micro.
Mehlman PDFs. Don't understand the hype, arrows and immuno seem ok. Neuro if you wanna rote learn and not understand I guess. Skimmed through them but found it meh.
FA - good for a quick review and last minute short term maxxxing but don't see it adding much if you do or don't do it. You should know all this and if you find yourself unable to skim and not able to speed read and nod knowing the majority of what's on the page - content is weak. Stop FA and go back to content.
Final thoughts, I think the % required to pass on this is pretty low esp if they count experimental questions as bonus points (saw some say they do, i.e. give you the point if you get it right but otherwise don't count). But in any case, the % to pass is is prob in the <55% range so easily doable. I don't see how some fkn droolers in here say it's 68-72% or some insane shit that doesn't make any sense. Mfker NBME gives you a 98% chance to pass with a 68% how in the world would you need 68-72% on the real deal lmfao.
Anyway, if 98% is the P likelihood at 68%, that would put it at 2 standard deviations which I would venture to say would be about 4-5% (imo, but I'm sure there's some insights out there that would let you approximate this more accurately) and it would put P at around 53%ish (if we go 3.5std below for a fail). A 58% gives you like a 80% chance to pass on NBME so I reckon the standard deviation is likely not too far off from that. I'm pretty sure the 98% and 80% rates are right since I got 68% and 58% on 2 of my forms (this was weighted average tho, so might be 70 and 60% raw).
Oh, and final point. Got 0 of the free images PDF on my form (or maybe 1 out of like 40-50 or however many pics they put but had little to no bearing on the answer). So wouldn't spend much time memorizing old images religiously. One pass night before suffices. But again, YMMV.
ETA:
Can't believe I forgot RN for biostats, obv gold standard for content and then everything from Uworld/FA flows much better and makes much more sense. I had a decent biostats background and even though I didn't need a lot of it, it was a good refresher and still helped for some mnemonics or just general review. For someone with 0 background, this would be even more HY. Unfortunately I barely got any biostats on my exam, and all of it was straightforward, had a single calc question the whole exam and it was the most basic shit on incidence.
DM vids on comms/ethics and how to approach the next steps for the comms questions prob up there now. I saw his one video on it long ago, scoffed,thought I was hot shit w my practice %s then felt like Mr. Bean exam day reading some of these on the real deal.
Finally, use the consensus agreed upon gold standard resources. E.g. uworld, sketchy micro, pathoma 1-3, RN biostats, etc. for most things. But don't dwell on some resources if it doesn't work for you. Eg, I almost never do anki (only instance was sketchy for a month or so during earlier med school, and even then it was the pepper deck with whole scenario recall, I have no faith in single cloze anking deletions for long term learning) as I find less benefit in it than doing other stuff. Mehlman that everyone's high up, I don't see much use in doing personally. B&B same deal on most topics since it's more FA narration (altho he can explain more in some concepts, but some sections are just straight facts).
Still, some of these may work for you, imo just use the universal gold standards on some topics (e.g. sketchy micro) and see which resource best suits you in others.
Finally, work on your weak areas. Don't altogether neglect your strong areas but do way more work/questions/try different resources if it doesn't make sense until it makes sense/go back to basics etc. on your weak content than the stuff you know well.
Comments:
User:Congratulations on the p! I’d like to ask, did you get a lot of Biochem and Immunology? Also were your Neuro questions >your Nephro or what? Big ups to you on a nice detailed post!
User:Thx, very little immuno molecular type shit, e.g. particular cytokine receptors, mechanisms and the micro details. I got most immuno in terms of more basic what disease is this questions (obv you'll get the timeless NADPH oxidase vs. MPO type question in some form) but it felt less than most forms. Some biochem, about average to what you'll find on the forms. Nothing too crazy/out of left field/nitty gritty. Know the classic glycogen storage diseases they always love to ask those. Again, asked in a similar manner to immuno, w some enzymes and co-factors thrown in there, but the common ones, nothing crazy.
User:Congratulations!! 🎊 can you list out the best resource system wise please 🙏
User:Too subjective, and don't even really know tbh. For some topics a 5-10min vid from some random youtuber might be more worthwhile than a 1hr+ vid explanation elsewhere. DM for biochem is good for some rote learning. B&B is also pretty good for biochem from memory but also did this really long ago in year 2, and didn't revise since. Immuno prob pathoma + FA worked for me. The rest have kinda touched upon somewhat. I hear onlinemeded has some good vids on some topics but never used them. I'd just target weak areas with specific videos that work for you, and supplement with doing all uworld qbanks on said topic + proper revision.
User:Thankyou 💙
User:How were your NBMEs?
User:Pretty average and high variance. 56 baseline 3 months outs (27) 62 2 months out (26) 65 5 weeks out (28) 76 4 weeks out (25, offline) 58 (!) 3 weeks out (free 120) 70 3 weeks out (old 120 back to back days) 78 10 days out (30) 69 5 days out (29) 70 4 days out (31) All of these are raw scores, the weighted average is prob between -1 to -3% less for each test.
User:Congratulations! What are the DM vids for communication/ethics?
User:Dirty Medicine. He has like a broad video on it and some vids on ethics principle. I think Randy Neil has a buncha comms videos too. I can't vouch for any, I only saw the DM main comms overview which was good, and highlighted the order in which you should say things which is going to be key. I would start with that and consolidate w FA comms section or some of those other vids.
User:Oh LOL I should have put that together. Thanks!!
User:Congratulations I didn't get what did you watch for ethics and communication and I am still struggling with it and exam is really soon
Post 50: Should I take Uworld assessment vs Nmbe 25??
https://www.reddit.com/r/step1/comments/1kn24sm/should_i_take_uworld_assessment_vs_nmbe_25/
15/05/2025, 04:18:02
3 comments
I have 2 weeks till my exam and plan to take 32 next week. I haven taken 26-30 so far . Some people have told me that 25 is very high and very biochem heavy and not that representative; While Uworld assessment 1 and 4 is very similar to real deal.
Thoughts?
Comments:
User:brother , when you do nbme 32 and uwsa 4 do let me know. it seems like you'd be the first one ever
User:lol
User:someone hook it up with what this dude's on
Post 51: Passed step 1 with free 120 score of 60%
https://www.reddit.com/r/step1/comments/1kn0ms1/passed_step_1_with_free_120_score_of_60/
15/05/2025, 02:37:15
13 comments
My exam was on April 23 2025, got my
Results last week. (It took two weeks)
Hello everyone ! Iam Non US IMG, YOG 2022. My undergrad had compulsory rural service for 2 years post mbbs so time went in that and couldn’t study much during those two years. I started preparing for step 1 after internship.
Material:
1. I started with BOARDS AND BEYOND
Opinion: it’s quite slow and after a while it gets boring. But everyone advised me to start with it. So I watched all his videos once.
If you are bad at basics then it would be useful to watch it.
2.First Aid.
Opinion: it was extremely useful for step 1 and NBMEs. Everything in First Aid is important I feel. I read it multiple times.
3.Uworld.
Opinion:useful to acclimatise to long stems similar to the step 1 exam. I finished 1 pass with 96 percent completed. But I stopped uworld more than 6 months before exam.
4.NBME and Free 120.
Opinion: these are must do !! And it’s better to start NBMEs early. Because most of the topics in the real deal are from nbme so better to start it early and get better at those topics.
Iam bad at BIOSTATS. But I did some last minute brushing using Randy Neil Biostats videos. They were very helpful.
And high yield images pdf
Pathoma 1 to 3 MUST DO !!!
Step 1 : the REAL DEAL
1.Some questions were super easy, directly out of nbme topics
2.Some had to use a lot of logic and hope it’s correct.
3.I got 2 images from high yield images.
4. 2 exact questions from free 120z
5.ethics were hard (Istill dont know how to make sure we choose the right answer mostly just gotta trust your gut and choose) and a lot !! 5 to 7 questions per block.
6. Some questions I had no idea and was pretty sure I was getting it wrong.
What I found hard was sitting for 7 hours ! By the 5th block it was exhausting for my eyes, brain and spine!!
So better practise to sit for long hours.
And take enough break time coz I had extra 30mins left by the end of the exam.
And take good food (wish I had taken instead of just snicker bars and dried up oranges)
Nbme 25: 61%
Nbme 26: didn’t write
Nbme 27: 67%
Nbme 28: didn’t write
Nbme 30: 68%
Nbme 31: 68%
Free 120
Old: 63 %
Free 120 new: 60% ( 3 days before exam)
By this time I had cried enough and I was so tired of postponing and getting scared of step 1 but somehow I felt confident to just
Write the exam.
And gods grace I passed !!
I don’t have any advise to give..if anything
Put the trust in god,
Pray !
Be truthful to yourself and you would know if you are ready to give the exam.
Trust yourself !
All the best for everyone else ! This too shall pass !!
Comments:
User:Thank you for sharing this. I tested last Wednesday and such posts have been so helpful(as much as anything can be post exam). I completely resonate with the part where you’re not completely satisfied with your scores but you’re too tired to go any further and you just want to get it done with. Congratulations🥂
User:Thank you !
User:Congratulations on the P Can I seek some advice from you over DM
User:Thank you and yes you DM me
User:Congratulations 🎉
User:Thank you!
User:Congratulations! How long was your prep time?
User:Well the two years of rural service, I got maximum of 2-3 hours to study per day between patients,during lunch break and post dinner. So in that I tried to complete boards and beyond and uworld (1 pass). Then after rural service I got 6 months time to prepare.
User:Congratulations!
User:Which high yield images pdf are you referring to?
User:https://drive.google.com/drive/folders/133JAFkx7HR957_Mp_Mu2txhF3aomtTH0
User:Congratulations. Howd u target ur weak points after taking an nbme.
User:Make a list of topics you’ve missed a refer back to first aid or mehlman docs, I usually just had a excel sheet and would control find the concept from pdfs from either resource. And if it’s applicable like a physiology concept, make sure you actually understand it not just memorizing it. I think the big part in all of this is keeping track of your missed questions
Post 52: NBME 55% → Step 1 Pass | Why I Stopped Caring About Practice Scores and Focused on What Actually Helped
https://www.reddit.com/r/step1/comments/1kn0g2k/nbme_55_step_1_pass_why_i_stopped_caring_about/
15/05/2025, 02:25:32
53 comments
Posting this for anyone who’s feeling crushed by NBME scores. Wrong F Answer!
One week before my Step 1 exam, my *highest* NBME score was 58%. The last one I took (NBME 31), I scored **55%** — and honestly, I didn’t even review it. I was over it. Burnt out, frustrated, and convinced that I was doomed.
But I passed. And looking back, the **NBME scores didn’t mean sh\*t**.
After that 55%, I ditched what I was doing (UWorld and NBME back-to-back) and completely shifted gears. I went all-in on **Mehlman videos**. Watched them playlist by playlist, all day. Before he answered any question, I’d pause and try answering it myself. That changed everything for me.
It wasn’t just content — it was learning *how to think*, how to eliminate wrong answers, how to rule things out with confidence. That mindset shift was the game-changer. It trained me to approach questions more calmly, more logically, even when I didn’t know the answer outright.
The test itself? Honestly, it felt harder than UWorld and nothing like the NBMEs. But I was ready for that. I went in expecting to be unsure about 90% of the questions. I didn’t panic. I just treated it like solving a puzzle: find the best answer, not the perfect one. That shift saved me.
Three days before the test, I took both the old and new Free 120s.
* Old: 70%
* New: 62% (first block <60%, second and third around 65%)I did better once I calmed down. First block nerves definitely hit hard.
In the last stretch, I also watched all Mehlman’s micro playlists and a bit of physio. No more practice questions. Just locked in on *understanding* and *strategy*.
If I were to do it again:
I’d run through UWorld *twice*, add Amboss if time allowed, and I’d definitely **watch all of Mehlman** — supplement with the PDFs when needed. But most importantly, I’d train my mindset early. Because high NBME scores don’t guarantee a pass, and low scores don’t mean you’ll fail.
They don’t correlate like you think. They just show you know some stuff — but Step 1 tests *how you think*, *how you manage stress*, and *how you approach uncertainty*.
Don’t go in expecting to recognize answers. Go in knowing you’ll have to *reason through* most of them.
That’s it. You got this. Feel free to ask about playlists I used if it helps.
Comments:
User:How long did it take you to watch all videos? Im exactly 15 days away from my test and tbh i feel that doing qs and reading is just not my thing. Ive watched mehlman before i love his content but have never done it like this. My last NBME was 29 offline and scored 58% (then i repeated 25 and got a 65%) but biased since it was a repeat. Also got a borderline chance of passing in BC SA. From there i havent done any, this saturday im having free 120 (the practice session in prometric), and at this point i feel i dont even know how to study anymore. My uworld expired, bootcamp expires today, meh. Im just trying to do flashcards and re read my notes but I feel im not progressing…
User:So I spent a day on a block. Hopefully that gives you an idea. It took me roughly a week, to watch more than half of all his content. With 2 weeks left, you can honestly binge watch all of them block by block. Once you are done, take the old free 120. Then the new free 120.
User:I do recommend taking old free 120 a week in. Then the new free 120, the week after
User:Thank you so much ! I already have the free 120 scheduled for tomorrow, because i wanted to take it in the center to simulate and to be familiar with the site😅. However i would do old free 120 and i still have 30 and 31 left in the upcoming days, along with your plan i think would be enough. This is very helpful! I needed someone with an experience like this because i really love his videos but everyone kept saying: do more qs do more qs 🙄
User:You got this for sure! A week before my test, form 31 was 55%. So I did mehlman for only a week then saw that jump in free 120. He’s so good! You got this
User:Yeah we’re in very similar boats, testing end of May, UW just ran out, and I’m consistently under 60 but getting closer. Need to just fight the burnout and rip Mehlman plus whatever other free resources we can find! Good luck
User:How are you ripping mehlman? You mean the vids?? Im thinking about taking the amboss free trial, but i dont know if now i should only be targeting Nbme material. Im losttt 🥹
User:I just provided a plan
User:I’m in a similar boat… I’d love to add melhman on but I still have a good bit out UWorld to finish and a fuck load of anki and content to learn everyday… I can’t imaging ripping through blocks of videos on top of what I’m doing now
User:Well, everyone says that if you are close to your exam to drop uworld, (or any qbank you have for that matter). I really liked u world, but to be honest i cant say that i noticed a significant progress while i was doing it. It helped me with pattern recognition and i learned tons of micro and biochem with it, immuno as well, but for physiology and path (which is most of the exam), it did not help me AT ALL… but thats just me. I did not finish it, i dropped it at 55% with 50% score… my relationship with anki is love-hate haha i really dont like it, but I have to say that is the only thing that does not let me forget tiny details. I dont do it everyday though, and I really wish i started doing it sooner. If you ask me i would drop uworld and start reviewing nbme content (Mehlman is GREAT for this) whether is the PDFs or the vids or both. You can keep doing qs if you want, just at this point I think is better if you focus more on HY material, and lower u world qs.
User:would you suggest doing mehlman in initial phase of prep? i am currently doing it along with FA and honestly speaking i've found it quite helpful but im still confused as most people say it will just artificially boost up your nbme scores in future since he only focuses on nbme questions and topics and that you should not touch them until after your nbmes. what are your thoughts on this?
User:definitely! depending on your timeline, you could watch few videos a day. The videos are based on his pdfs and for me it helped me apply the concept better. If you have a good foundation, you could jump straight to the videos. Then right, after do some uworld to lock in the concepts. The videos tramp uworld in my opinion.
User:Mehlman GOAT🎉🎊💯
User:Congrats on passing! I’m 5 weeks out and NBME scores are in low 40s so def need some guidance. For the MM videos do you mean his HY USMLE Q videos?
User:Thank you!! With scores in the 40s just verify if you need to improve your content by doing focused studies. Honestly, I think the videos will help you with content + skill. I went to his you tube channel, clicked on playlist , and watched them system wise instead of random. That drilled the contents into my head. I started with immunology, which to me was perfect.
User:Can you explain more please what do you mean by the test is harder than uworld and not like nbmes ?you mean the main points they focused on or the style of the questions? My exam on 26th and my score is about 62 and now im freaking out
User:I just took it on 30th this is bs, I would say test is what you make of it some questions are really easy and some are hard. If you go in thinking you have done enough and you are at least consistent mid 60s on nbmes and other practice tests you will pass. His shift on how to approach questions is 100% correct though. It is about recognizing patterns in questions and helping yourself out. I do not agree with his mehlman stuff but to each their own there.
User:That was my experience. My test was harder than Uworld. If yours was easier it is your experience. That being said, it’s being harder doesn’t mean you can’t pass. You need the right mindset and approach the questions the right way. It doesn’t matter how hard or easy it is.
User:Im not disagreeing with your mindset but their is a reason u world is a gold standard and its not because the real test is harder in fact it is opposite. Your telling al your questions were harder than u world or that majority of questions are harder than uworld is hard to believe.
User:The style was more difficult to decipher than Uworld. The stems were way longer! It was different than what I was used to and that makes it difficult. Content wise, it was difficult to tell what they were asking cos they throw so many bs on your face, and you are cornered to pick the best answer. Risk factors were even difficult cos most resources don’t hit on them well enough. So many other factors…
User:I’m doing 40 UWorld questions a day, and I‘m not sure if I should add Mehlman PDFs or just watch the videos. Which one would you recommend more?
User:The videos supplement with the pdf when totally lost at a concept
User:how much time you took to watch videos all in total you watched systemwise,??And do you make notes out of them and at what speed u used to listen
User:So from morning to evening I sat through the videos. It was less hectic as I found his videos funny whenever he said wrong fn answer. Again I tried to answer the questions before he did, so it counted as “doing questions” once I got use to his tone, I was able to watch the in 2x speed (1.7x and 1.5x sometimes if it needed to be slowed down) 90% of the time, it was on 2x speed. I made sure I completed a system before moving on to the next system. And no, I didn’t take notes - because he repeated the content over and over again. That drilled the info into my head lol. Since you have enough time, you can always go back to the videos later, and review them (I would just skip through them the second time)
User:Tysm one more thing how much will they take to complete all
User:It took me a week to watch more than half of his content. 2 weeks is enough to finish all
User:And also you also watched Subject wise folders for exampple one for pharma made separately and for pathology made separately
User:I recommend doing pharm last after micro only if you have time. The current forms has been low on pharm for some reason. Micro is a must tho I recommend doing it after everything. Each system covers the pathology so you don’t have to watch the pathology playlist (they are the same). The pathology playlist covers the pathology portion of all the systems so if you are covering each system you are automatically covering path as well
User:Did you just run through the system specific playlists where he goes through questions?
User:Yes system specific. I started with immunology, and ended with physiology the day before my test(I just did a few of the physio videos as I didn’t have enough time). I completed each system in one day.
User:I think this is the main tip for getting into passing range. You need to know the content well enough and then you can focus on question strategy. Depending on your school and their curriculum there may be knowledge gaps or time gaps. By bunkering down and really developing a strong content base, questions naturally become easier
User:Can you tell which playlist
User:How long to watch all the Mehlman’s videos? 17 days away and I pretty much started the same strategy rn
User:2 weeks is enough to finish all the content
User:Can you share the playlist please
User:It’s on YouTube. Mehlman then hit playlist.
User:God I hate the idea of giving Mehlman views (look up why he's a creep) but I really need to get it together so may have to give this a try...maybe I'll start from a HY systems review (like Cardio/GI)?
User:I heard about all that. I give him credit for my pass. I’m here to study and pass lol
User:How much of Uworld did you get through? I’m testing late May and I’ve only done half of Uworld. Got a 63 on Form 28 last week but I just don’t know how much it’s helping
User:I did half of Uworld as well. 17% was pre dedicated
User:I think you are in good position to binge those videos. Let me know how it goes
User:Are the videos you’re referring to the ones that are on his YT channel for free?
User:Yup free
User:Wow congrats!!!!!
User:thank you!
User:One day 40 questions?
User:huh
User:Tysm for helping out and can you plz tell mehlmann pdfs must to do!
User:Which videos are you talking about?
User:Please let us know the playlist of mehlman and it's pdfs that you read...
User:Hello doc doing uworld twice is necessary?
User:I am waiting for my step1 result for next Wednesday.. my experience is the actual step1 Qs were long stemed and a lot more chart Qs than I expected , plus not similar to the NBMEs .. I struggled with time I the 1st 2 blokes.. I hope everything will be okay.
Post 53: What’s the origin of para follicular C cells?
https://www.reddit.com/r/step1/comments/1kn0chm/whats_the_origin_of_para_follicular_c_cells/
15/05/2025, 02:19:14
6 comments
Is it endoderm or neural crest?
Comments:
User:According to FA endoderm phyrengeal pouch 4th one
User:Okay! So this is what we should choose for the exam ? Thanks!!
User:Well u should follow the text obviously it endoderm on FA and also on embryology books I read like langmans medical embryology and also klm I guess Soo I guess it should be right 😅
User:Endoderm. If you think it'd be helpful - I made a silly video to remember endodermal derivatives - https://youtu.be/TG__42RFS_U?si=46yuOIcAIAV9ADQD
User:Neural Crest cells, they are the ones who invade pharyngeal arches.
User:i hate embryo so much
Post 54: Order of NBMEs?
https://www.reddit.com/r/step1/comments/1kmzitc/order_of_nbmes/
15/05/2025, 01:28:52
4 comments
This has prolly been asked a lot but my dedicated just started and I have about 4 weeks. What order should I take them in? I want to do like one per week hopefully? Thanks!
Comments:
User:Personally I did 25 first because I read it was the hardest. But I went in order. I don't think it makes a difference.
User:It really doesn’t make a difference. The only recommendation I got was do 31 last.
User:And then do the free 120 after that?
Post 55: From an NBME of 33 to a pass in 4 months (you guys got this!!!! trust your gut and try your best)
https://www.reddit.com/r/step1/comments/1kmxr1c/from_an_nbme_of_33_to_a_pass_in_4_months_you_guys/
14/05/2025, 23:50:40
37 comments
This post is for all of you who have worked so hard but your NBME scores still did not make the "cut" that ppl think you need to take this exam. I started studying for this exam at the end of December after procrastinating and being a terrible student throughout preclinicals (and I payed for that mistake). At the start of dedicated I took NBME 27 and got a 33 (lol) and literally though I was done for. But I worked hard and grinded like I have never done before and got the pass. My NBME scores in the order I took them (this will make you feel better if you are worried about your scores):
CBSE (through school): 35
NBME 27: 33
NBME 28: 35 (had a breakdown after this)
NBME 26: 45
NBME 25: 48
NBME 29: 50
Free 120 (2021 version): 60
NBME 30: 49 (full on breakdown after this)
NBME 27 retake: 67
NBME 28 retake (never reviewed it 1st time): 58
NBME 31: 60
Free 120 (new version) - week before exam: 58
NBME 30 retake (to boost confidence, remembered a good amount): 75
These were all the exams I took and as you can very obviously see, my scores were not high. This is not to tell you to be delusional and just take the exam, but for those of you who have put in the work and just cant seem to see any progress on NBMEs even though you know deep inside that you have done everything you can possibly do to pass the exam without losing your mind. I had pushed my exam back so many times and got to such a low point that I knew that it was time to take it regardless of what happened because I needed to be finished and done with this exam before I lost my mind completely.
What I used that helped: Uworld, Amboss, Dirty Med (I used this later and wish I used it earlier bc it was very helpful!!!- watch the entire pathology playlist), Sketchy micro and pharm, Mehlman HY arrows
YOU GUYS GOT THIS!!!!! TRUST YOUR GUT, TRUST YOUR HEART, YOU WILL KNOW WHEN IT IS TIME TO TAKE THE EXAM (REGARDLESS OF YOUR SCORES) - BE POSITIVE <3333
wrote this super fast, lmk if yall have any questions :)
Comments:
User:yooo are you me because this is litearlly my set up rn and i just took the exam a week ago. congrats on the p!!
User:good luck!! you definitely got that P!!
User:Yesss!!! Congrats! How did you feel post exam?
User:mostly relief for just being done with it lol, but I honestly did not feel too bad after but the stress of waiting for the score made me second guess myself
User:hey! congratulations! i took the exam a week ago and you have no idea how much of my anxiety you just helped quell.
User:thank you!! and I am so glad! that is exactly why I made this post, you will do great just try not to stress yourself out during this waiting period
User:Congratulations 🎊 👏 best review ever 👏
User:thank you!!
User:Can you give a step by step on what you did? I just got a 38% on form 26 and I am 6 weeks out.
User:to keep it short and simple: I would do 20-60 uworld questions in the morning, then when reviewing if I got an answer wrong I would (in that very moment!!) look up a youtube video (dirty med was helpful) or find a pathoma video that related to that topic and watch it, then after that I would go on amboss and do very targeted questions just on that topic before moving on to the next question (ex, I get a problem on lung cancer wrong on Uworld, find video on youtube or pathoma about lung cancer and watch it, then go on amboss and only do questions on lung cancer and feel comfortable with those questions (I would make little fact sheets about the info) before moving on), this helped me a lot, there were also days where I did not even do UW and just did targeted review of topics I struggled with on amboss (I struggled with hematology/oncology and endocrine, so I would watch videos and do amboss Qs in the format I stated above). Good luck!!!
User:First of all congratulations!! I wanted to ask how did you manage your medical school curriculum and step 1 prep side by side . Like did you drop everything and focused on just step 1 or how did you allocate your time between them?
User:Thanks!! and yeah I am not someone who can multitask so I basically dropped off of the face of the earth just to focus on step 1 studying lol (not fun but it had to be done), I had to move my schedule around so that I could just focus on STEP 1
User:Which prof were you in if you don't mind answering?
User:Congrats!!
User:thank you!!
User:>thank you!! You're welcome!
User:Thank you recently got a 44 and I was devastated. Gives me hope
User:you got this!! so close, almost there!
User:Congrats! Was this 4 months dedicated? Or alongside rotations?
User:thanks!! basically 3 months of full dedicated and 1 month where I was between another class I had to take and STEP 1 prep, but I still focused all my energy on step prep in that month
User:did you do dedicated all through those 4 months?
User:what does dedicated mean everyone here says this
User:as in did you give your time entirely for step prep or did you do other med school work parallely
User:for the majority of it I was fully focused on just step 1 prep which was definitely exhausting but I had to do what I had to do (there was about a month where I was between having to take a class that I was unable to get out of and studying but I still made step1 my priority).
User:Hey man congrats Nothing sweeter than the taste of victory after hard work. I have been prepping for this exam for such a long time because of the frequent interruptions of in-class exams. i have done 74% uworld completion, some dirty medicine, mehlman, sketchy micro, pathoma here and there and 2 NBMEs. Scores are good but I really don’t think i’m gonna finish all uworld before i can take my exam in june. If i don’t take it in june, i know i will never take it after that since im an IMG and starting my internship year soon. So i’m just worried really. I have only enough time to revise incorrects for NBME not every question and ones I got by chance. What would you give as a word of advice?
User:Hey, thanks! I would not worry about finishing uworld bc most ppl do not finish it (it has a lot of questions!) I finished 86% of it but I think that rather than focusing on just finishing Uworld you should start doing blocks on uworld with both unused questions and incorrect questions bc that will help reinforce concepts that you learned but might have forgotten (I did this and it helped me a lot). I would also focus on the concepts on NBME and trying to understand them fully, like if you see that you got a question wrong on lung cancer understand how they presented this lung cancer and why the answer they wanted was the correct answer, and if you have time look at the incorrect answer choices and see what those answers would actually be referring to (like if the answer was about pathology of squamous lung cancer understand the pathology of that but if the answer you picked was pathology of small cell lung cancer also know the pathology/presentation of that). Good luck!!
User:Congratulations!!!! Can I ask what difference in preparation you made in the 4 months?? Like the resources and such
User:thanks!! I switched around my resources a lot during the 4 months but what helped me the most was uworld, amboss, dirty medicine, youtube videos in general on things that confused me, anki (only for sketchy) and I wrote a comment above explaining how I integrated all of this together in a way that helped me. Good luck!!
User:This is the same sort of scores I’m getting and throughly stressed out. Thinking of pushing my exam back a few weeks (since it’s in 2 weeks.) just so I can review properly. Still going through sketchy and dirty medicine.
User:Please I am new in med school what is nbme or cbse if they are the same thing what is it important for?
User:Omggggg congratulations So motivating, really needed to hear this♥️
User:THANK YOU SO MUCH!!! 😭😭😭😭 I NEEDED THIS!! TODAY I WAS ABOUT TO GET A MENTAL BREAK DOWN AND GIVE UP ON LIFE. YOUR POST SAVED ME!! 💗💗💗
User:Congratzz! Have you finished First aid? If yes, then how many times?
User:Where can I find this NBME
User:congrats Op! how did uo mange nerves and pace on exam?
Post 56: PASSED!!
https://www.reddit.com/r/step1/comments/1kmxf7o/passed/
14/05/2025, 23:33:38
16 comments
Soooo, before I took the exam, **I had never used Reddit before.** But the post-exam feeling was so extremely confusing and defeating that I had to find people feeling just like me. It couldn't be possible that only I had this type of feeling. So I got here, started reading everything, and found out I was not alone. My feelings were normal post-exam anxiety.
For the past 2 weeks, I’ve been in total suppression of any thought. Just focusing on working out, pre-clerkship rotation, and keeping my head away from medicine. But still, every 2-3 hours, I would get this horrendous feeling through my whole body with just the thought of failing the exam. This is the hardest part. The wait...
But thank God, everything was completely worth it — I passed. I’ve re-checked a hundred times because my impostor syndrome won't let me be happy. But yes guys, it's normal to feel anxiety and feel lost right after the exam and in the next 2 weeks. You’re not broken. You’re not alone. Pray for the best, pray for your pass.
Hope you all get it. And thanks for being there when I most needed it. Even if you didn’t know you were helping — you were.
[.](https://preview.redd.it/cncrslgnxu0f1.png?width=690&format=png&auto=webp&s=b47c8cea4fc6abfddb9c24eeb0f2294ee3541f1f)
Comments:
User:Please share your NBME!
User:My names are: 25- 67%, 26-69%, 27-72%, 28- 78%, 29- 77%, 30- 85% and 31-86%. UWSA were wild, 1- 84%, 2- 72% and 3-69% Free120 was 80% This is very subjective, as the real thing is not similar to any assesment
User:Bro no shot you were failing with scores like that. I have never seen anyone almost get a A on the nbme
User:Guys please stop making nbme results as a determining factor. IMO the exam is not even close to the nbmes anymore. They need new to make new forms
User:so true
User:Mediocre swimmer on fire
User:Congratulations
User:Thank you!!
User:Congrats!!
User:thank uuu
User:Congratulations!
User:thank uu
User:can you plz tell me the resources and i just started the prep with zero basics and i’ve chosen kaplan that’s why noting the bnb was tough for me can you plz tell me how should i do all the stuff
User:I learned everything through BnB, Sketchy for micro and pharm, pathoma 1-3, amboss and uWorld. I used sam turco from Kaplan for biochemistry. Also used dirty medicine for ethics and biochemistry. And last few days used Mehlmann for ethics and repro. Randy Neil for biostatistics. Hope this helps!
User:For how long you were studying for?
User:Im an IMG, so studied for 1 year and a half, during my pre-clerkship rotation. About 4 months dedicated.
Post 57: STEP 1 FOR DO STUDENT
https://www.reddit.com/r/step1/comments/1kmwda0/step_1_for_do_student/
14/05/2025, 22:39:35
2 comments
my school is requiring a 57 on my CBSE in order to sit for comlex level one. i was not planning on taking step 1, however, i took a cbse through my school last week and i got a 48. On my COMSAE (basically its the predictor for comlex) i got a 460, the pass for that is 450. How can i increase my CBSE score?
Comments:
User:Do more Uworld/Amboss. Highest yield things you can improve on are stats/ethics/communications. Add in some micro/pharm and plenty of pathology questions. Then take an NBME self assessment and see which areas are your weakest in the insights tab.
User:Do the NBMEs and review them thoroughly.
Post 58: Uworld so expensive.
https://www.reddit.com/r/step1/comments/1kmv9ff/uworld_so_expensive/
14/05/2025, 21:42:14
4 comments
Anyone know a way to get uworld step1 for cheaper for 6 months ?
Comments:
User:How?
User:Message me for more info
User:IMD APP
User:Thank me later
Post 59: Got 63% on the free120, exam in 3 days, should I delay?
https://www.reddit.com/r/step1/comments/1kmou3v/got_63_on_the_free120_exam_in_3_days_should_i/
14/05/2025, 16:56:58
12 comments
My nbme scores in order:
Nbme 25: 60% (5 weeks ago)
Nbme 28: 61% (4 weeks ago)
Nbme 29: 58% (3 weeks ago)
Nbme 30: 67% (2 weeks ago)
Nbme 31: 61% (10 days ago)
free120: 63% (today)
Exam in 3 days, should I delay?
Comments:
User:I think you should delay. You need a buffer in the exam.
User:I think I will postpone it till the 3rd of June because this is the only available date in my test cente, what do you think I should study during this period and what self assessment exams should I take
User:It depends how prepared u feel. If you feel like your nerves might derail you, then delay. But if you feel ready, take it. The only concern to me is the point drop between nbme 30 and 31. Don’t let my scores factor into your decision, because I’m just one data point. But, if you decide to take it, maybe it will provide solace. Form 30: 61%, form 31: 67% (1 week out), and free 120: 63% (2 days out). I got the p today. The people on this reddit are a bit scary. Of course getting a 70+ on practice exams is ideal for maximizing your chances, but it’s not possible for all of us to achieve that in the timeline given. Most of my friends were hitting mid to high 60s up until the exam and they all passed. Just keep in mind though I felt like I failed so these past two weeks have been absolutely terrible. Having higher practice scores will not only act as a buffer for maximizing your chance of passing, but will give you better peace of mind.
User:I think I will postpone it till the 3rd of June because this is the only available date in my test cente, what do you think I should study during this period and what self assessment exams should I take
User:Yes, delay it. You are not ready.
User:I think I will postpone it till the 3rd of June because this is the only available date in my test cente, what do you think I should study during this period and what self assessment exams should I take
User:Hi! I think delay. Take more free120s and if you get 67+ on them then test! NBME 30 is really good but your other scores are borderline. You’re on the right track, I think it might be test taking skills.
User:I think I will postpone it till the 3rd of June because this is the only available date in my test cente, what do you think I should study during this period and what self assessment exams should I take
User:I would delay until you can get 65+ on another NBME
User:I think I will postpone it till the 3rd of June because this is the only available date in my test cente, what do you think I should study during this period and what self assessment exams should I take
User:F
User:I think I will postpone it till the 3rd of June because this is the only available date in my test cente, what do you think I should study during this period and what self assessment exams should I take
Post 60: STEP 1- PASS AMENNNNNN
https://www.reddit.com/r/step1/comments/1kmo2ik/step_1_pass_amennnnnn/
14/05/2025, 16:25:52
7 comments
This exam was very doable, no crazy bugs, no crazy drugs, straight forward and everything all the free 120s and NBMEs prepared us for.I felt calm (after initial anxiety) during the exam, there were no curveballs and I felt cautiously optimistic. I thank GOD for this as this journey was SO LONG for me. I had to transfer schools because a 65 CBSE wasn't "good enough". This built so much panic and anxiety but in the end I am right where I needed to be.
Resources:
As a "read/ write" learner: I was on my 2nd pass or third (who knows) of Uworld with a 68% accuracy.
I used Chat GPT to help me with content I did not know or questions I got wrong and asked it to help question me as a form of active recall. I gave AI the content to question me on to prevent errors, this took hours a day.
I used anki for several hours a day, I recommend Mnemosyne its directly from First Aid.
I used Melman pdfs infrequently. Bootcamp was helpful for weak pointsl. I listened to Pathoma chapters (1-3) when i was driving at least 1-2 times a week (not consistently). I could not just sit down and passively watch videos.
Data:
My second pass of NBMEs ranged from 68-73. But I did get a 76.5 on an offline NBME.
My first pass of the OLD 120 (2021) - 76. OLD 120(2022)- 69. NEW 120(prometric center)- 87. The new 120 was so high because there were so many repeats. BUT it helped with anxiety as I saw that I could be successful at a prometric center. 3 days later I took the real deal. If you have any questions please let me know.
Comments:
User:Amen!!! And Congrats! Love reading score updates from ppl who thought the exam was doable. I also felt that way but Reddit has me second guessing like maybe I felt that way because I actually failed lol.
User:lol Same!
User:Congratulations! Couple of questions! Are you an IMG? And how long was your prep time?
User:Yes, US-IMG. I prepped for 4 months.
User:Awesome, I’m also planning a similar timeline! (also an IMG, except I’ve been out of school for years). How were your basics starting out? Did you feel like 4 months dedicated was sufficient?
User:Basics was good, first year better than the second year. Yes, I chose 4 months, I could have elongated it to 6 months if I wanted to. I felt ready around 3.5 months honestly. I did take a year break due to transferring schools so that helped as I studied for a year on my own before doing a school mediated prep course.
User:how did you use AI? please help, i suck at it
Post 61: Feeling lost with dedicated prep
https://www.reddit.com/r/step1/comments/1kmnr7z/feeling_lost_with_dedicated_prep/
14/05/2025, 16:13:03
2 comments
I have read reddit posts and watched a handful of youtube videos and there seems to not be a consistent answer on how to properly structure dedicated prep. I take STEP in the beginning of july and im particularly struggling with how to approach anki with reviewing old material. If im reviewing a system that I struggle with (heme/respiratory since those were a year ago) and cant remember the material should I be resetting the anki cards as I review the system to have them pop up as im prepping for the exam? At this point im doing 40 questions a day, reviewing them, and working through pathoma/sketchy pharm (my cardio pharm is bad), and then doing my anki reviews for the day. I would love any pointers on how to better approach board prep.
Comments:
User:Hey there, I tutor for Step 1 and here are my recommendations. Focus on the NBME practice exams and reviewing them thoroughly. Know those concepts well and find memory tools or helpful videos (dirty medicine, sketchy) to help the material stick. Create anki including those memory tools and keep doing more NBMEs. Hope this helps! And if you're looking for a tutor, you can DM me.
User:All I did was use the “find cards from uworld” add on and pulled cards for every UW question I missed. I also added the tagged cards from every NBME I took. I never reset cards, just add them into the deck
Post 62: Take it/Push back/Cancel??
https://www.reddit.com/r/step1/comments/1kmne9n/take_itpush_backcancel/
14/05/2025, 15:58:53
5 comments
Hello all - DO student here. I need some honest advice on whether or not I need should take STEP or if I should push back or if I should cancel. Signed up to take COMLEX and STEP on 5/20 and 5/23 respectively. I have been preparing with Sketchy Micro/Pharm, B/B, and Pathoma since January. I was doing Anki with Sketchy Micro/Pharm until 3/29. I gave up Anki and started doing 1-3 40 ? UWorld Blocks/day since then. Write down notes on ? I miss to understand why I missed ?. I completed all video review on 4/22. UW average is 61% with 52% completed (1st time through all new questions each time)
Practice Tests include COMSAEs, NBMEs, UWorld SA, Bootcamp SA
COMSAE 107 on 3/29 - 356 (87% chance passing based on data from NBOME)
UW SA 1 on 4/1 - 52%
COMSAE 114 on 4/28 - 426 (97% chance passing)
UW SA 2 on 5/1 - 58%
NBME 28 on 5/3 - 60%
NBME 29 on 5/9 - 54%
COMSAE 111 on 5/12 - 453 (66% overall with a 99% chance passing)
Bootcamp SA on 5/14 - 62%
I know free 120 is the "most accurate predictor" and I am planning on taking that on 5/16, but I do not know what to do. I can't ask any admin at my school because they are "DO focused" and only care about COMLEX. Many of my MD friends think I am fine and to "trust the process" but I honestly have no idea what to think/do. I am 100% confident in my abilities for COMLEX since I have gotten the passing score, but not as convinced for STEP because I am right on that line.
Thoughts on pushing back STEP one week to get more familiarized with questions? Thoughts on cancelling and just taking STEP 2? Any and all advice would be beneficial. Thank you all.
Comments:
User:The only predictive measures you have so far are 28 and 29, all the other stuff you’ve taken is not considered predictive. Free120 isn’t even technically considered predictive. Based on a 54 and a 60 I would push the exam back by 2 weeks at least. You need to be scoring 65+ 1 week before you test. That’s the rule at my school (US MD) I can’t tell you anything about whether or not to take it at all because I’m a US MD. By my school’s standards though you would not be permitted to test until you can get 65+ on at least 2 NBMEs
User:Thank you, I think I will push it back if I do not get a 65 by Saturday on an NMBE.
User:yeah at my school you have to get 65+ on 2 or 70+ on 1 to test, so if you only have 1 left before the real deal I would only proceed if you can crack 70
User:I’d highly suggest taking all nbmes if you can and studying the content on that. All of it is super high yield
User:Thank you, I have been doing this but have not yet seen the results. I need to stop second guessing myself because that is happening on 50% of my wrong questions
Post 63: Which among them is high yield?
https://www.reddit.com/r/step1/comments/1kmmsox/which_among_them_is_high_yield/
14/05/2025, 15:34:53
14 comments
No content found.
Comments:
User:These names are ridiculous 💀
User:ikr!!
User:Actually because who came up with this
User:None 😬 I might have seen them as wrong answers to some qs
User:I've seen Ustekinumab and Secukinumab written the most often. I've never heard / forgot about the last four. But it's just my opinion.
User:same i feel like i have seen those two names only somewhere, thanks
User:Low yield but if you do want to remember one secunamab is il-17 inhibitor
User:LMAO never seen those in my life
User:None of them, tested. 5/7
User:Punt
User:I’ve never heard of these and passed, not on my exam lol
User:They sound like country names to me!
User:None of them 😂
User:Never even seen those before and I got the P lmao
Post 64: PASSED!!!!
https://www.reddit.com/r/step1/comments/1kmmeqx/passed/
14/05/2025, 15:19:27
4 comments
Walked out of that testing center convinced I failed, so hopefully this gives people some hope!! Cried tears of joy seeing that pass. I completed 63% of Uworld with 57% correct. My ISP began March 28th. Up to that point I had been doing psych and neuro UWorld questions to match up with school content, and had finished the entirety of the psych questions before ISP began. I primarily used UWorld and FirstAid, and did Chapters 1-3 of Pathoma. I watched BNB videos on anything that tripped me up content-wise (renal). Prioritize questions!!
4/06 CBSSA 31: 59; 04/13 CBSSA 29: 68; 04/20 CBSSA 30: 73; 04/23 Free120: 67; Test Day 4/26.
Comments:
User:congratulations!!
User:Did u do anki?
User:No, because I figured there was no point starting if I hadn't been using it for medical school thus far. If I was to do anything different, I would have used Anki since M1 or at least began at the start of M2, just to maybe make my life a little easier. But it is NOT a requirement!!
User:congratulations 💥💥💥 Which system did you encounter the most tests from?
Post 65: Passed!
https://www.reddit.com/r/step1/comments/1kmlps6/passed/
14/05/2025, 14:52:02
21 comments
So I passed! For complete transparency, my nbme scores never reached above 70. Highest was a 68. Other scores were 54, 57, 62 and 61. Free 120 was a 65. I scored the 60s all within 3 weeks of the exam. Study the nbmes. Know the concepts on there inside and out. Good riddance to that exam!
Comments:
User:Congrats!
User:Hey, many congrats on your pass. As an IMG I hope I'll pass too. Giving my step 1 in less than 2 weeks' time from now (after postponing it from May 9th).
User:What day did you take the exam? Congratulations!!
User:Took the exam on 4/28. Thank you!
User:How many days did it take for your result?
User:Only took 2 weeks
User:How was NBME 31?
User:That was the 62. Panic took it 4 days before exam in case stuff from it was on the exam and I’m glad I did!
User:wow, so people in reddit are so neurotic, always saying need to get 70+
User:This is the post I was waiting, my day is 22/5 I never reached 70 I am so scared
User:Stay locked in, you know enough and you are enough. That is what I told myself. The adrenaline will push you through
User:Congratulations. What were your post exam thots
User:It was tough but reviewing all available nbme’s is what I think helped out the most.
User:img?
User:No, US MD
User:How many NBMEs did you take & what were your exact scores! Thank you in advance
User:I took all available nbme’s. 27: 47 26:57 28:54 29: 61 30: 68 31: 62 Free 120: 65
User:Congratulations 🎊
User:Congrats!!
User:CONGRATS!!!!
Post 66: Urgent urgent help step 1 biostats
https://www.reddit.com/r/step1/comments/1kmlcxv/urgent_urgent_help_step_1_biostats/
14/05/2025, 14:38:16
1 comment
I dont have time to rewise my uworld question of biostats and i watched rendy neil two times but i forgot his problem solving skills for biostats now i am doing mehelman pdf of biostats .. is this pdf cover my concepts for real exam ?
Comments:
User:On my exam, Randy Niel was all I needed. Just do some practice problems in uworld and you’ll be good
Post 67: Uworld subscription
https://www.reddit.com/r/step1/comments/1kml32y/uworld_subscription/
14/05/2025, 14:27:31
3 comments
Anyone trying to get rid of their Uworld account? Mine expired and I only needed it for like 30 more days and paying 300 for 1 month is crazy. If you have even like just a couple of days left on it and are willing to give it up please let me know
Comments:
User:You can get uworld access for 50$ for 10 monhts on myqbanks com they provide 7 days free trial as well.
Post 68: I have a doubt regarding weightage of systems..?
https://www.reddit.com/r/step1/comments/1kmkc6v/i_have_a_doubt_regarding_weightage_of_systems/
14/05/2025, 13:58:16
1 comment
USMLE content outline gives us an estimated of % share of questions from individual systems and subjects. While reviewing an NBME, I’ve observed that the 200 questions were distributed exactly as the weightage mentioned in USMLE website. But UWSA didn’t follow this pattern, for example, a UWSA had about 15-20% questions from only endocrine.. I obviously bombed it.
From the pass reports posted in this sub, I’ve observed that subject weightage changes between different test-takers but systems weightage is constant across every score card.
My question is, does USMLE stick to the range they publish both it their website and scorecards? Or the real exam can be like UWSA and throw 20% questions from one random system?
Comments:
User:Generally they will stick to the range but obviously their is variance in their range itself
Post 69: Can't report step 2 without step 1 anymore? DO
https://www.reddit.com/r/step1/comments/1kmk5tv/cant_report_step_2_without_step_1_anymore_do/
14/05/2025, 13:51:11
4 comments
2nd year DO student. Have heard from multiple 3rd yr students that you need to take step 1 because you can't report step 2 without taking step 1 anymore. Is this true or are they off their rocker? I've searched all over the web but couldn't anything to support that. It would definitely alter my plans for dedicated time if true.
Also if someone knows the source, please cite it so I can send it to others.
Comments:
User:That’s not true, I’ve mailed the ECFMG and they responded with: “Please note the USMLE® Step 1/ 2CK examination does not need to be taken in order. You must have completed the basic medical science component of the medical school curriculum by the beginning of your eligibility period. Although you may apply for and take the examinations after completing the basic medical science component of your medical school curriculum, it is recommended that you complete your core clinical clerkships, including actual patient contact, before taking Step 2 CK.”
User:While that partially answers the questions, doesn't full. I know you can take them in any order. But can I report Step 2 and never take Step 1? That response doesn't quite answer that question.
User:I think this is a very DO specific question so you might not get an answer from nbme/usmle. I noticed more people took step 2ck than step 1 in our COM’s upper classes. You probably can ask your career counselors if the algorithm at some residencies have changed to let comlex l1 stand in for a step 1 pass after they both went p/f? I don’t know.
User:Fair point I will start with that and see if they have any further guidance.
Post 70: PASSED
https://www.reddit.com/r/step1/comments/1kmj6tg/passed/
14/05/2025, 13:12:54
7 comments
Just got my P! I thought I would write this in case it is any help to anyone else! AMA!
CBSE (2/21): 54%
Form 30 (3/24): 65%
Form 28 (4/6): 66%
Form 29 (4/14): 66%
Form 31 (4/21): 71%
New Free120 (4/25): 70%
Finished 66% of UWorld at 56% correct
Tested on 4/28: Passed
Resources used:
UWorld
Anking
Bootcamp
Dirty Medicine anatomy
HyGuru anatomy
Pathoma 1-3
FA Rapid Review
Randy Neil biostatistics
Listened to all of Daddy Goljan at least once, some multiple times
Comments:
User:Congrats!!!
User:Thank you!
User:Congratssss عقبالي
User:Congratulations 🎉🎉
User:thank you!
User:Congratulations!’
User:thanks!
Post 71: Passed
https://www.reddit.com/r/step1/comments/1kmizfn/passed/
14/05/2025, 13:04:47
7 comments
Hello everyone I just want to say I passed Step One. I had to delay my clerkships and everything but it was worth the wait just to see passed. For those of you are worried trust me you can do it. I started with a 34 in our schools practice Nbme in January and had to grind my way up. For studying I will say do practice nbme forms 28-31 at least not because they are reflective of the exam ( they to me were harder) but you get to see how usmle asks easy questions in odd ways. Just seeing the content in those exams and reviewing ( made my own anki deck for each exam) helped me review a lot.
In terms of q banks, while I do recommend uworld it is fine if you do not use it. I think one thing that really helped me with uworld is that they have the same screen as step 1 so when I took my actual exam I was able to calm down and just think of it as another u world session. I did all of uworld q banks and 600 bootcamp and 500 amboss questions I think getting a mix of questions from different banks regardless if they are too easy or not is the best way to do things because you get to see stuff in different ways. If you are time crunched stick to u world and do bootcamps 10 practice step 1 questions.
In terms of content review I would just pick and choose bits from Anking lkke I did most of the biochem in his deck and chapters 1-3 pathoma but not much else I would more just select different stuff I maybe missed on uworld or other q banks and just make smaller subdecks. Im not a anki god so I say this to people who think you need Anki you really dont but Anki does help you constantly see info in a active form which I think there is value in that.
Lastly for videos I did not watch many videos as to me a question if reviewed well was basically worth a video in fraction of the time. But I did watch lysomals and glycogen storage disorders on bootcamp and heme/onc stuff. Only watch videos for stuff where you are so confused that you know watching videos well clear up all confusion. For me I just could never figure out the above stuff and while doing cards I realized I never knew it ( aka always hitting one minutes) so I would watch videos. I guess what I am saying is watch videos ( whether this means sketchy bootcamp or whatever) if you just do not have a base knowledge at all. One other thing and it sounds dumb is take breaks. I basically studied for 8 weeks straight got burned out and sick and could not study for a week. I then came back and like everything clicked.
Lastly for mehlman docs I recommend reading them only towards end of studying not because they are holy grail but I have seen some people boost their scores through him and then fail the real deal just because there was no real based knowledge. The one I do recommend you go through over a a few weeks is his arrow one as he does good job actually explaining concepts in it and arrows questions can be tricky. I would do about 30 of the arrows a day and it took me about 10 days to go through the doc, and be happy with myself. I also then quick reveiwed for another two days. The review time for these should be at most two hours for 30 arrows. For all his other docs at most you should just quickly glance through write down what you really think is important about a week before exam. The Nbme rocks and dirty medicine high yield images are frankly useless. Not because they are bad but by the time day of exam comes you will have forgotten it. Plus at most there will be three similar images. My exam had no similar images for example.
Im sorry for the long post but I really try to showcase I did my best and succeeded and frankly started a lot lower than most of yall. My scores I only broke 70 once. I took the exam when I had consistent over 65 pluses on nbmes and other practice forms such as blue. I say take the exam if you are getting consistent 64 plus because on like at least 3 exams. You just need to prove to yourself on good or bad exams you can be consistent.
Comments:
User:Congrats for a P I want to ask if you did a single pass of UW or did you incorrects too
User:I did all of UW starting working through incorrect but still had at least 1200 incorrect left. To me working through incorrect is nice but not necessary as it provides good reviews over concepts but you should hopefully have already seen it at some point.
User:I see, thank you for this insight. Also I’ve just started my UW. Can you tell me how much time did you spend doing UW. No of block per day and the time it took you to complete it. I’m sorry if asking too much
User:No you’re fine so I started Uworld in January initially while I was still in preclinical and did maybe between 120-200 a week because most days I was still studying my endo repro block and stats afterwards. I then in dedicated would do between 80-120 everyday all days. I slowed down uworld towards the end of my intial schedule date ( april 4th) as I was already running out and switched to a bit if amboss and bootcamp with maybe one u world section a day. If I were to have just grinded uworld I would have been done in Mid march but I did not feel that was helping as I got to the point where I recognized uworld questions really well based of their wording and other missed qs but did not translate to real knowledge imo. Forgot to add I did all blocks of 40 always tutored never timed but would usually finish most blocks in about 50 minutes ( aka timed bit of uworld) and spend probably a total of 40 minutes reviewing missed qs and making notes etc.
User:I see, thank you so much, wishing you best of luck for your future too, really appreciate.
User:Can u tell me did dng amboss really helped u or could we skip it with doing an other pass of Uworld?
User:If you have done all of uworld i think it is worth switching to something else with doing a few i incorrects from uworld here and there. U world does not make the test so it is worth seeing different perspective’s. Personally I feel the test tested over concepts more than specific stuff like they want you to analyze in terms of clinical aspect for a lot of questions.
Post 72: I got the P! A mind opening write-up!
https://www.reddit.com/r/step1/comments/1kmin4z/i_got_the_p_a_mind_opening_writeup/
14/05/2025, 12:51:33
24 comments
All glory to God
I got the P!
Some bulletproof points regarding preparation for USMLE Step 1:
● Decide on your main video resource early in your preparation. Try different ones, but personally, I believe Pathoma should be a must. Pair it with another source—for me, BnB worked out really well.
● As soon as you finish a system, Finish the same system in Fist Aid, then jump into UWorld—this is where you apply the knowledge you've built.
● UWorld is not a self-assessment tool, but at the same time, try to keep your scores above 60%—except in the beginning, when you're still adapting.
● If your scores are low in UWorld, revisit videos. Continue UWorld, but slow it down and focus on rebuilding your foundation.
● Try to go through First Aid after each system from your video course before UWorld, and then recheck it again during UWorld.
● The more you hit FA, the better you retain the little facts. Don’t read it like a newspaper—real exam questions come from the nit-picky details.
● BnB has some basic videos in renal, biochemistry, and respiratory that are hard to retain and not very high-yield—be wise with your time!
● I never liked this guy Mehlman, uses many foul words, but to be frank, his PDFs are great for building the USMLE mindset.
●Anki is up to you, personally i didn't use it except of some occasions, but i know some people who can't live without it, you decide, but if you don't use it then you gotta to revision everywhere, in the wc, on the Bus, while talking with friends, before falling asleep....etc.
● Check Reddit—I personally learned a lot there. But don’t forget, people have different personalities. Learn to recognize and avoid those who stress out or over-dramatize everything.
● Every NBME concept should be stuck in your mind!
● Accept it: in the end, the real deal doesn't look like anything you've seen before.
● By exam day, make sure you’ve mastered all NBME principles, are efficient with time, and can think outside the box.
● The exam is not fair. They mess with your dignity. I had a friend scoring in the 80s on NBMEs, mine were in the mid-70s, and we both walked out complaining about the same things!
● Enjoy the post-exam 2 weeks!
May God bless you all.
إِنَّ اللَّهَ وَمَلائِكَتَهُ يُصَلُّونَ عَلَى النَّبِيِّ يَا أَيُّهَا الَّذِينَ آمَنُوا صَلُّوا
عَلَيْهِ وَسَلِّمُوا تَسْلِيمًا.
Linkedin: https://www.linkedin.com/in/hakar-mohammed?utm_source=share&utm_campaign=share_via&utm_content=profile&utm_medium=android_app
Comments:
User:Congrats on the P! Did you do Anki or anything to continue keeping that information in your mind?
User:Thanks, good point i will try to add it to the write-up, well i didn't use anki personally it didn't work for me, but i would rather revise through continues practicing of uworld or even during university classes with friends....!
User:Congratulations ,,, which proactive NBME score or new 120 ?
User:Could you please clarify what you mean?
User:Which is more productive NBMEs scores or new free 120 ? Cuz I got 60% in free 120
User:It was more similar to new 120, but don't worry much about it, it might have been due to burn out since free 120 is the last exam we take, i personally dropped to 64% on the new free 120 but i knew it was due to burn out. Check your answers, take enough rest, and use ibuprofen if u have headache....
User:Thank you … pray for me to pass
User:Ameen, all the best 🙌
User:Uworld low average ,uw main learning resource
User:Well if you aim to learn mainly from uworld and u are doung fine with that, then low scores are fine , but i would still Recommend videos....
User:should do second pass random mood?
User:If you have enough time, yeah do it, actually when we reviewed my incorrects, many time i chose the same incorrect answer again, so better to review if u have time
User:Sorry if this is a dumb question but how exactly are you using Mehlman PDF’s? Like do you reference them when you’re wanting to review a topic or do you skim through it passively?
User:It is a good question, i did both to be honest, and some require returning to the same pdfs multiple times or try to discuss it with a studying partner or friends, will help!
User:Please, how long did it take you to prepare and write? I find that the videos take some time, but they are truly helpful. Secondly, did you repeat the same pathologies taught in B and B in pathoma again?
User:It took me around 8 months, no i never checked pathology of bnb but i did the pdf of mehlman of pathology
User:Hi! Congratulations. Are you an IMG? How long was your prep time and how were you basics starting out?
User:Yes I'm an international medical student, 8 months of preparation, my foundation was kinda normal not that good nor bad somewhere between...
User:جزاك الله خيراً برو
User:و جزاكم
User:what is the best time to use mehlman pdfs? and also should we only use the topics portion of his pdfs or the one line questions at the end of every pdf?
User:I used it in the last 1.5m before the exam, most of my content review were from his pdfs, and definitely do not skip anything in the pdfs.
User:Jezakellah kheyran. Very beautiful write up. One thing I would like to comment is if you could just erase one phrase in your write up. You understand what I mean.
User:Thanks mate wa jakazum, which sentence?
Post 73: If passing is 60-65% how getting 60% in NBME almost get you 90% chance of passing?
https://www.reddit.com/r/step1/comments/1kmi9fc/if_passing_is_6065_how_getting_60_in_nbme_almost/
14/05/2025, 12:36:12
16 comments
I have always had this question in mind especially after I sit the exam, everyone agrees that the real exam in many cases a bit harder than the nbme + stress and the longest of the exam despite all of this is still scoring below the passing range get you very good chance of passing???
I FEEL SOMETHING IS MISSING!
Comments:
User:Someone double-check this, but the %correct you see on the report when you finish the NBME is actually lower than your actual %correct. (I personally never check it, but a friend of mine did) Example. lets say you got 124/200 (62%) questions correct on a NBME form. The NBME report says you got 60% correct and 80% chance of passing.
User:Yeah they lower the percentage, its correctef becaude NBME assumes the older NBMES are easier than the real deal (which they are)
User:Which more productive new free 120 or NBMEs?
User:I think both, NBME content wise and there is many form so it covers so many topics and Free 120 similar vibe to the real exam and longer questions so good training for times management.
User:Hi sorry, where can I find the doc you screenshotted?
User:I found it on reddit in old post just type in google nbme chanceing of passing you gonna find it.
User:Thank u!
User:It doesn't mean 90% chance of passing per se. It means this: Of 100 people who took STEP 1 and scored a 60% on this NBME, 90 people passed STEP 1.
User:Means the same lol
User:What about new free 120 I got 60% ?
User:You not guarantee the passing but as we discuss here according to this data you probably gonna pass ( assuming you have similar grade in multiples NBME )
User:That mean free120 is more productive than NBME .. ?
User:No, I think each equally important cos they examine different things. Just do both.
User:Ok thank you and congratulations 🎉🎉
User:No worries, I didn't pass yet :(
User:25% of the questions on the real exam are unscored and used as trials for future exams. A majority of these experimental questions are intentionally hard. And because of that, most people score better on the real thing than they do in practice
Post 74: Why you need to get off this reddit
https://www.reddit.com/r/step1/comments/1kmhzdm/why_you_need_to_get_off_this_reddit/
14/05/2025, 12:24:37
50 comments
I just received my passing score today, and I want to encourage everyone to *please ignore the negativity on this subreddit*.
First of all, this subreddit often makes it seem like passing Step 1 is nearly impossible. It’s not. The national pass rate is around 90%. You’ll also see people here saying you need extremely high NBME scores before sitting for the exam, and that’s simply not true.
A month prior, I scored 60% and 61% on 2 nbmes. then pushed my exam back. My last NBME was at 68% a week before step. It was inflated though because i did several offline nbmes and saw around 8 repeats on this nbme that i previously got wrong. I understand the importance of being cautious, but we also need to be *realistic*. Scoring 60% EPC on an NBME still gives you an 83% chance of passing. I’m not saying you *should* take the exam at 60%, but it shows that a “high” score isn’t required to pass. I do not recommend sitting for the exam if your best is at 60% but statistically, its important to realize that you will still most likely pass.
My Free 120 score was 62%. This subreddit treats the Free 120 like it’s make-or-break. It’s useful for format and familiarity, but it’s *not a predictive exam*. Trust your NBME scores—that’s what truly matters.
I’m sharing this because I felt awful in the days leading up to my exam, mostly because of how toxic and fear-driven this subreddit can be. Whether it’s gunners or just extremely anxious people, their posts can really mess with your confidence.
If you’ve taken more than two NBMEs and scored over 62% EPC on them, I’d say go ahead and take the exam. The NBME's predictive model is statistically sound—it accounts for test-day anxiety, fatigue, and uncertainty.
Have confidence. Don’t let fear-mongering discourage you. You’ve got this.
Comments:
User:Good post, hate to say it but it’s mostly just IMGs fear mongering. Granted they have much less room for error but still.
User:I hate saying this because I’m an IMG myself, but it’s true. Most of the fear mongering here are from IMG’s who were just caught off guard by the exam irrespective of how well they’re prepared and humans do have a tendency to ‘crash out’ when career making exams like these just surprise you despite how much you’ve prepared. So I get the occasional heads up or update, but people just straight up crash out here in a way that’ll demotivate the hell out of others. And this is super evident when you read positive posts by IMG’s who’ve passed the exam because it’s a whole different experience for them and they genuinely motivate you. Or even US med students who admit how they were underprepared or weren’t sure if they prepared properly.
User:I am getting neurotic with so many people in here saying that you need 3 +70
User:This is why i said get off this reddit. I dont know if its because all the IMGs on here or just super cautious people but i know only 1 person in real life who sat for this exam with over a 70% epc. I think its more of a safety thing, unless you have a mental collapse on test day or run out of time, an nbme around 60-62% is passing
User:One can try getting 70+ in nbmes but then people say exam is not even similar to nbmes.makes us wonder is it similar to then ,is it even medicine being asked or some outside stuff where people only know 5/40 questions per block and rest are all blind guesses left right and center
User:People say the exam is not similar to the nbmes because the format is different, the actual thing has extremely long question stems and you have to move much quicker, but read the last sentence first and if you compare the nbmes to the step questions, its literally the same stuff just longer stems.
User:Bless you
User:Oh okay .thanks
User:I never got an NBME above 68 and passed. You got this
User:I disagree. A lot of the failure posts are people with NBME scores of 60-65%, who fail anyway despite the high chance of passing that NBME claims. If you check this very sub, that is almost always the case. While you may have passed with a free 120 of 62, a lot of people posting on this sub failed with much higher scores. The national pass rate of 90% includes people who are well prepared and scoring much higher than 60% on their nbmes. If you take the exam with borderline pass NBMEs of 60%, your chances of passing will not be 90%, it'll be much lower. You must consider the fact that you may be an exception and not the norm, don't encourage others to take the test without being completely prepared. Quite honestly, you'll probably pass with NBMEs of 65% and there genuinely is more fear mongering than there should be - but why not work a bit more and boost your scores to prevent the chance of a failure. Anyone scoring 65% can score 70% with a bit more hard work.
User:I never stated that you should take the exam with an nbme score of 60%, im simply telling the facts of the outcomes of this exam. Yes, going in with a 60% nbme and passing is cutting it close but passing is not the exception, that is still the rule. as the nbme states, 83% of people with that score still pass. I was not ok with that and pushed my exam back and got a higher nbme score before sitting. My point was that this reddit is not an accurate representation of the real world results of step 1 takers, it is filled with fear mongering and inaccuracies about the exam. The exam is like the nbmes, just longer, and this reddit talks about the free 120 like it some exam that is used for readiness. It is not, your nbmes are what tell the story, as the USMLE say, the free 120 is simply 120 questions with the format of test day to get you prepared. People with 75% free 120 probably pass but if their nbmes are not good, i would not take the exam. I was not the exception, I was statistically what the nbme predicts to pass. If you score a 62% epc, that is around 90% pass rate. I understand your point of caution but I made this point because this reddit overhypes the difficulty of this test and overhypes the scores needed to pass. Just wanted to give confidence to others like me that never broke 70% epc and still passed.
User:Reddit doesn’t give an accurate sample of actual test takers. I’d still be more inclined to trust the NBME projected pass rates based on more complete data sets than antidotes on Reddit. While someone with low 60s NBMEs is more likely to fail than someone with 70+, they’re still more likely to pass than to fail, albeit a low pass.
User:>Reddit doesn’t give an accurate sample of actual test takers Agree. >still be more inclined to trust the NBME projected pass rates based on more complete data sets than antidotes on Reddit I think you meant anecdotes. That's the issue. When we see a fail result, the graph they show is from 2023 data. The pass rate has decreased every year since the shift to pass/fail. Is the nbme pass percentage up to date? Even if you advocate taking the test with NBMEs in the 60s, which would be around a 85% chance to pass - that means 15 % would fail. And it's that 15% who'll post here asking for advice and feedback on why they failed. Which y'all will conveniently label fear mongering. If you're aiming for NBMEs in the 60s, and take it with a chance of failing - sure, you do you. You'll probably pass anyway. But why do you feel the need to recommend others do the same? Why are you encouraging other people aim low? A step fail closes so many doors. I mean I understand your POV, but I would personally never recommend anyone take the exam with more than a 10% chance of failing. Which would correspond to NBMEs of more than 65% anyway.
User:No one is recommending people aim low. It’s more so letting people know they’re statistically not doomed if they aim high and don’t make it. Sure I want to get a 70+ on every NBME. Does that mean it’s going to happen? Not necessarily. Does that mean I must be lazy and not trying hard enough? Also not necessarily. But for some (and I’d argue a good number of) people, pushing their exam out time after time chasing that 70+ is more detrimental than going in confident that you’ve put in your best and it’ll statistically pay off.
User:I wish you a successful career
User:Thank you, you as well
User:Appreciate this post heavy. Am I correct in thinking that compared to NBME’s the question stems on the real thing will be longer and there may be more HPI type questions? … But ultimately, there’s only so many ways they can ask about renal stones or stroke localisation etc? And congrats!
User:Yea thats a good way to look at it, they may give you the entire hpi of someone with a kidney stone but there will be like 2 sentences that you actually need to answer the question
User:Agreed. I made the decision to stay off this subreddit for the entirety of my ISP. Otherwise I'm convinced it would have driven me into a mental breakdown. Focus on YOU!! YOU got this!!
User:Yes!!! I commented on one of those posts to say as much and got a bunch of shit for “invalidating” OPs subjective experiences. Posts like that had me sweating over the reschedule button because my forms sat at 65-69 but couldn’t break 70 (until literally day before lol). Meanwhile it was smooth sailing through the actual exam ~
User:THANK YOU for posting this. Exam is tomorrow and this is exactly what I needed to hear right now.
User:we got it bro
User:This was reassuring. I had my free120 at 68 and nbme 31 last day at 66.
User:testing tomorrow and needed to see this ! thank you my friend !!
User:Were the qs inexam similar to nbme and free 120
User:Both, the length is the free 120 but it is nbme concepts all throughout, review the free 120 and use it for more questions and practice but your nbme scores will determine your readiness.
User:Congratulations on the P! 🎉 Can you please tell what does 70% on an offline NBME 28 mean? I'm unable to find any score converter. I'm asking because you mentioned about the 'inflated' thing in your post.
User:I meant inflated only for me because i did several offline nbmes before i did form 26 so i saw like 8 exact repeat questions that were on the old offline nbmes so i lowered my actual epc to account for them because i would most likely not have got them right if i hadnt previously seen them. You took nbme 28 offline and got a 70% raw score, im not 100% correct but from what i saw, form 26-30 raw score is 2 points higher then the epc, so your epc would be probably 68%, which is definitely ready to take the actual test
User:Thank you so much for this. I've been waiting on my score and I got a 69% on the free 120 and only a 66% for the nbmes which was a retake of form 26 since I took it months ago, but the rest were sub 60. Obviosuly not erring on the side of caution here, but everyone here has been freaking me out lol
User:Yea you a probably fine then, please stay off reddit and this toxic subreddit, i know its hard but if you adjust your nbmes for inflation, and you are in passing range and nothing catastrophic happened on test day, im sure you will pass.
User:I know you're not a fortune teller lmfao but that actually made me feel so much better, tysm :') and congrats on the big P!!
User:Of course, good luck, im sure you did fine
User:What do you mean adjust for inflation? I just took NBME 27 and got %60 correct? I’m trying to figure out if I need to push my exam back.
User:inflation meaning i took nbme 26 last 1 week before my test, and i saw around 10 questions that were on the offline nbmes that i studied from prior to this nbme. of those 10, 8, i got wrong previously so i asked chat gpt and they said to subtract 8 from your total raw score, which is around 4-5% epc, so i went into step with the belief that 64-65% epc was where i really was at. If you just go a 60% epc, and your exam is within the next 2 weeks, i would push it back, because that is literally borderlline pass fail, and unless you dont wanna study anymore or believe in the next week it will improve a enough to pass, then definetly push back. I was just saying you definitely do not need over a 65% epc to pass step 1 like many people on this reddit claim.
User:Ohh thank you, that makes sense.
User:Thank you for saying this and sharing your experience! I was freaking out because I got a 65 and a 66 and it seemed like everyone else was getting 70s. I made the decision to take the exam in 5 days. I understand anxiety around step1 but I’m honestly so frustrated by people asking if they’re ready to take it with scores in high 70s.
User:I remember going through this same thing for the mcat, this reddit has a bunch of weird people who want to be reassured of how smart they are "am i good to take step with a 78% epc" yes like why do you even ask these questions. It creates panic and uncertainty in the community and makes people doubt the actual data that the nbme tells us about chance of passing. You have two scores of 65 and 66, the nbme is telling you, that they almost guarantee you will pass, unless you have severe test anxiety that hinders your performance, go take the exam.
User:How does uw compare to the real exam in terms of difficulty and length of stems?
User:Same length as uworld, uworld has the same format for step 1 but is much harder, i found uworld tricky, and it seemed like they try to trick you, so nbmes are most accurate
User:So we don’t need to overthink questions in the real exam (like we do in uworld)?
User:exactly, uworld is filled with trick questions to get you to learn better. you gotta remember the actual examiners are usually not trying to trick you, there is information and facts they want you to know and they give you the clues, you just gotta see the pattern recognition. On the nbmes and real thing, if it feels right, pick it, its probably exactly what they are looking for.
User:Would you mind sharing how you were scoring on uw blocks? I’m mostly hovering in the 60s with the occasional spike and drop.
User:If you hovering in the 60s and have done enough questions, you got Step in the bag
User:never got above low 60s in uworld blocks, by the end, i was scoring from 55-63. Like i said before uworld is a learning tool and is intentionally hard.
User:greed1
User:Congratulations! 😊 im a premed and was wondering if you could give me some tips because i have my eyes on some very competitive fields (nsgy, ortho, cv surg, ic, em)?
User:Just focus on killing the mcat and doing research early and getting publications if you want to do a competitive speciality (when you get to med school)
User:Congrats dude 🥳 did you also took the offline nmbes? If yes? Do you think they can inflate your scores on the newer ones?
User:Yea i took offline 25, and 22 i believe, like 10 of those questions end up on nbme 26 (online exam) so i definitely accounted for how it probably inflated my score.
Post 75: Got the P!
https://www.reddit.com/r/step1/comments/1kmhyzr/got_the_p/
14/05/2025, 12:24:08
4 comments
Only felt OKAY after sitting. However, as time went by the possibility of failing started to settle in and I became anxious/depressed/hopeless. I had accepted that i had failed and even began re-strategizing. I was certain I had bombed a block or two since I was pressed on time all the time. but we saw the light at the end of the tunnel today!
it can be done! Study hard, like your future patients' lives depend on it, and everything else will fall into place. there's really no secret sauce to this :)
Comments:
User:Share your NBME please!
User:Congratulations!! What date did you sit for your exam?
User:Glad I am seeing such posts 😭🤝🏻 congratulations btw
User:Congratulations!!!
Post 76: 5/1 Takers
https://www.reddit.com/r/step1/comments/1kmhey6/51_takers/
14/05/2025, 12:02:00
8 comments
Looks like no results today..
Comments:
User:my guess is that we took it at a time there was a higher volume of test takers... so another week of agony we go...
User:5/2, same, another painful WEEK, WTF!
User:Yeah this is dumb bc 4/30 takers got theirs
User:Sad, waiting another week is gonna be tough
User:So far I have only seen the 4/28 guys, get the results, it is ridiculous
User:bro 5/3 over here, I dont think I can wait another week yall
User:Same broo
User:Took it 5/2-5/3 because of extra time, no results :/
Post 77: Did you pass the USMLE??!
https://www.reddit.com/r/step1/comments/1kmg3ut/did_you_pass_the_usmle/
14/05/2025, 11:08:48
12 comments
Please, anyone who has done USMLE this year, if you could share your NBMEs and the results of your exam.
Mine are:
MBME 29: 63%
NBME 30: 58%
1 month later Free old 120: 68%
2 weeks later NBME 26: 70%
NBME 31: 63%
I am doing this to decide if I should schedule the exam 5 weeks from now
Comments:
User:img here as well. my scores were in the same range as yours and got the P today. just be consistent in your prep and i think you're good to go
User:Hey, can you specify your scores and time before you took the exam?
User:25: 62% (7 weeks before) | 26: 64% (6 weeks before): | 27: 60% (took this first 8 weeks before): | 28: 68% (4 weeks before) | 29: 70% (3 weeks before) | 30: 68% (2 weeks before) | 31: 70% (2 weeks before) | Free 120 new: 72% (1 week before)
User:you are being dramatic, two 65% means youre good to take it next week.
User:But this 63% in the latest one worries me. I am an IMG, so all I know is high yield content. NBME 31 is most alike the actual exam.
User:Do more amboss/uworld, read the 2nd half of mehlman docs for your weakest subjects. You are doing very well
User:[deleted]
User:I already did uworld. Finishing mehlman arrows now, and gonna do the rest of mehlman
User:Take one more exam if you get over a 64 take it. You need to be confident when taking the exam as it does not look like any practice exams out there it is a mix of all. Being consistent should make you feel confident to pass the exam.
User:I have done all NBME. Only have uwsa and free 120 left
User:You could still do nbme 27,28, or 29 but I would recommend just taking free 120 and like I said over 64 then take it
User:27 and 28 I did a long time ago, and 29 i got 63% as i said
User:Have some confidence bro thats whats gonna make you pass. 5 weeks is good. All my NBMEs were 63-68% and I passed on my first try. You'll be fine
Post 78: OFFERING STEP 1 MENTORSHIP
https://www.reddit.com/r/step1/comments/1kmfzgp/offering_step_1_mentorship/
14/05/2025, 11:03:41
1 comment
Hi everyone,
I’m excited to share that I’m offering mentorship for USMLE STEP 1 . I’ve gone through the process myself and scored in the 254 and 258 in UWSAs and a 99% chances of passing in all 3 NBMEs that i gave . My journey, including the resources I used, what worked and what didn’t, is detailed in a STEP 1 experience post I shared here on Reddit – feel free to check it out on my profile!
If you're feeling overwhelmed, unsure how to start, or need accountability, some of the areas I can help with include :
* Personalized study plans
* Daily/weekly check-ins
* UWorld strategies
* FA & Pathoma integration
* Time management and mental prep
* Help choosing the right resources
This is a paid service (reasonable pricing), aimed at students who are serious about improving and want consistent support from someone who's been through it.
If you’re interested or have questions, feel free to message me or comment below. Happy to help however I can!
Wishing you all the best in your prep!
Comments:
User:Hello can i dm you ?
Post 79: step 1 NBMEs scores 26-66%, 27-70% , 31-68% and uwsa 1 64%
https://www.reddit.com/r/step1/comments/1kmfyi9/step_1_nbmes_scores_2666_2770_3168_and_uwsa_1_64/
14/05/2025, 11:02:30
2 comments
Exam on 13 june
Should i go for it ?
If i should delay then how many days ?
Comments:
User:my nbme scores were similar to yours and i got the P today. so i think if you're consistent in your prep and answer the remaining nbme's and free 120, you're good to take it as scheduled.
User:Congratulations And tnx
Post 80: “Unconventional USMLE Journey: Step 2 Before Step 1 – Learn From My Mistake”
https://www.reddit.com/r/step1/comments/1kmfrdn/unconventional_usmle_journey_step_2_before_step_1/
14/05/2025, 10:54:05
2 comments
Hey everyone,
I wanted to share my USMLE journey—it’s a bit different from the usual path, and I hope it helps others avoid the mistake I made.
I came to the U.S. from India in 2022 for a master’s program. During my second semester, I decided to pursue the USMLE. But instead of following the usual order, I took Step 2 first, which was definitely not a wise decision. Without a solid Step 1 foundation, scoring well on Step 2 was incredibly difficult, and my Step 2 score ended up being lower than I’d hoped.
After that, I spent around 9–10 months preparing for Step 1, all while working full time. For both steps, I relied heavily on Mehlman PDFs as my main resource. They helped me understand connections across topics. I used Pathoma extensively for Step 1 and Sketchy for both Step 1 and Step 2. I never went through First Aid cover to cover, though I did refer to it for cell biology and biochemistry.
Step 1 felt more straightforward to me—mostly basics. I’d say:
• 50–60% of the exam felt like Pathoma
• Around 25% was ethics
• The rest felt random and unpredictable
I was super nervous going into the Step 1 exam. One thing that helped me a lot was practicing time management. I strongly recommend:
• Reading the question from the bottom up—70% of the time, the main clue is in the last few lines
• Skimming through the answer choices first before reading the full question stem
Feel free to ask me any questions. I’m more than happy to help—good luck to everyone on their own journey!
Comments:
User:Same experience. Took step 2 before step 1. Before now, I’d have said it doesn’t matter, however I had many step 1 level questions in my exam and they were like WTF. NBMEs were in the 250s, one UWSA was 271. Amboss predicted was 260. Ended in the 230s. Took one of the NBMEs for step 1 after my test and saw at least questions lifted directly from that form alone!
User:What’s your scores like OP?
Post 81: Anyone got their result after 11 am EST?
https://www.reddit.com/r/step1/comments/1kmfra5/anyone_got_their_result_after_11_am_est/
14/05/2025, 10:53:57
2 comments
Has anyone ever got their result after 11 AM on a wednesday? If not ill just quit refreshing my mail
Comments:
User:Wondering the same haha
User:I actually got the email at 1:00 AM (I'm CDT) saying my results would be up by 11:00AM EDT today, so yours probably aren't gonna show today soldier, I'm sorry. The wait is nerve wracking
Post 82: May 2nd Results
https://www.reddit.com/r/step1/comments/1kmfjji/may_2nd_results/
14/05/2025, 10:44:34
11 comments
Anyone who tested May 2nd gotten the email yet? (Not results but the other one)
Comments:
User:got to be kidding me.. tested 1/5 permit disappeared 2d after the exam & still no email
User:nothing. can’t believe we have to wait another week🥹
User:Same here. Took my exam May 02 but still suffering.
User:Same 05/02!
User:No results email , no other email. Tested on may 02
User:ughhh my friend who tested the Friday before us got theirs last week so I was so hopeful that 5/2 would be this week :(
User:Me too🥲
User:Tested may 6th and no email
User:no results here :(
User:No results, tested 5/2
User:End of week takers usually are 2.5 weeks, so 5/21 for yall :(
Post 83: Thought I Failed but Passed Step 1
https://www.reddit.com/r/step1/comments/1kmfiwk/thought_i_failed_but_passed_step_1/
14/05/2025, 10:43:47
3 comments
Hello,
I am writing this message for those who sat the exam and are scared for their results or are scared about sitting it. I am an IMG and I have mediocre grades on my NBMEs and free120 and I sat the exam on April 30 and I found it extremely hard. I thought I failed and stopped flagging questions because I was unsure of more than half of each block. I spent the past two weeks always on reddit, I wrote down every question I was unsure of and the others that I know that I got wrong. I panicked the entire time but somehow I ended up passing. The exam is made to be hard and just keep in mind that the questions are experimental so it's hard to calculate your grade too. My point is, do not lose hope and keep studying because if I can do it, you can do it too!
My scores:
NBME 26: 64 (03/09/2025)
NBME 27: 69 (03/20/2025)
NBME 28: 58 (01/21/2025)
NBME 29: 65 (04/06/2025)
NBME 30: 70 (04/12/2025)
NBME 31: 65 (04/22/2025)
Free 120: 74 (04/26/2025)
Comments:
User:These NBMEs are not mediocre
User:11/05 test taker. I feel kinda bad about my exam, feels like I won’t make it. My nbmes and free 120 were very decent. Couldn’t do well in the exam
User:Congratulations
Post 84: My step 1 story
https://www.reddit.com/r/step1/comments/1kmew8j/my_step_1_story/
14/05/2025, 10:16:18
31 comments
I don’t even know how to process this right now. From November to April, I lived the same day on repeat—study, eat, stress, crash. I built my entire life around one goal. I isolated myself, pushed through mental and physical exhaustion, and kept going even when I didn’t think I could.
There were so many times I thought I wouldn’t make it. I had breakdowns. I questioned everything. I sacrificed more than people around me even realized. But I kept showing up—for months—because I knew what this meant for my future.
And today… I found out I passed.
It’s hard to describe the feeling. Relief, pride, disbelief… all mixed with exhaustion. I’m still trying to accept that it’s real.
To anyone out there in the middle of it—you’re not alone. If you’re grinding, doubting, burning out—I get it. Feel free to ask me anything. I’ll be honest about the highs, the lows, the mistakes, and what helped me keep going.
I just needed to say this somewhere. I’m proud of myself.
Comments:
User:Got results today and found failed feeling depressed i put blood & sweat for prep my nbmes are around 55-60 all 25-31 free 120was 63 %
User:Im so sorry to hear that. You'll pass next time. just try to get a safer margin in your free 120. i think going into the exam with 63% is very risky. I went into mine with 80%
User:Yeah that was a little low my guy. Someone should've advised you to get higher scores before taking.
User:Have my step 1 in 1 month Scoring around 60s on NBMEs ( 3 left ) U world done 90% with avg of 55% Struggling with Reading FA It takes me long to read stuff not because I m a slow reader , it happens coz i read each n every line in FoMo Kindly help me !
User:Skim fast through everything and try to get at least above 70% you should be fine. Best of luck !
User:Congrats 💓💓💓💓💓
User:Thank you 😊
User:Congrats <3
User:Thank you so much
User:Congrats ! How are y’all getting your results earlier
User:Thank you
User:Could you or anyone who passed share their self assessments?
User:Assessments all done online: Uwsa1 232 (November) Uwsa2 211 (Jan) Nbme26 67% (Jan) Nbme31 75% (march) Free120 80% (22nd of april)
User:I will share my soon. Stay tuned
User:Congratulations!
User:Much congrats 🥳 What resources did u use? How many questions a day did u do on average?
User:FA, pathoma, sketchy, b&b.1 block a day since day 1.
User:you put into words exactly how i'm feeling right now. congrats!!!
User:Thank you !! Did you pass too?
User:CONGRATS!!!
User:Thank you 🥰🥰
User:Congrats!
User:Congrats, OP!
User:Congrats 🍾🎈🎉🎊
User:As someone in the middle of it, thank you. Congratulations! I'm proud of you too :)
User:I'm so glad you did , your post felt like how I am dealing with this phase right now I know UWORLD is a learning tool, but getting low scores really puts me to doubt but I keep going thinking tomorrow scores will be better I have my exam after 2.5 months any tips for doing better ?
User:What did you do when you were so burnt out that whatever u did no information went into your brain? I find it so hard to study when I’m mentally very very exhausted. Tips on that please!
User:How did you feel post-exam? I sat for the exam on 11/05 and my anxiety has only got worse
Post 85: Fatigue after NBMEs
https://www.reddit.com/r/step1/comments/1kmeuq6/fatigue_after_nbmes/
14/05/2025, 10:14:21
2 comments
It's been 6 hrs since I got done with another NBME and once again I'm too exhausted to pick myself up and start revising it. I'm just one month out. Ik this post kinda dumb but is there anything that works for you people?
Comments:
User:unmm why not reward yourself for doing an nbme and take the rest of day off! It’s already mentally exhausting and your brain deserves to relax for the rest of the day
User:100% agree
Post 86: Congratulations to all who Passed! 🎉 Could you please help the rest of us testing soon? 🥹
https://www.reddit.com/r/step1/comments/1kmeje1/congratulations_to_all_who_passed_could_you/
14/05/2025, 09:59:38
9 comments
How was your test? I’ve been reading a lot of posts saying it’s harder and first aid isn’t enough. Is it really true? I test in 6 weeks and these posts freak me out
What do you think helped the most to improve your NBME scores?
If you could repeat dedicated again, would you do something differently?
Comments:
User:Step 1 is diff for everyone.for me there was too much micro in my test and a friend of mine got tested a while back and he had a whole ass weird ppr....just keep revising ur stuff,give ur nbmes and pray.i don't think we'll ever be completely prepared for what USMLE is going to test us for....it's another kind of monster.
User:Thank you! Did you find sketchy micro good enough?
User:I didn't do sketchy and there wasn't much pharma in my test thankfully.All I did was FA for pharma.
User:FA + Uworld + practice nbmes are more than enough. Exam is super doable. Don’t let anyone scare u here. Do a practice test every weekend and review it thoroughly. It’ll help u build stamina to focus for a long time. That way, the real deal will feel like you are doing another practice test. People may say they were tested on xyz subjects more, but it could be a bias bc that was one of their weaker subjects. The % of questions tested per section doesn’t change much. My exam had this exact same breakdown of questions tested. You can find the breakdown here: [https://www.usmle.org/exam-resources/step-1-materials/step-1-content-outline-and-specifications](https://www.usmle.org/exam-resources/step-1-materials/step-1-content-outline-and-specifications)
User:I felt like my first 2 blocks were really easy, and last 2 were really hard, maybe just because I was exhausted. What helped the most to improve my scores were just finishing every system, and doing more mixed blocks on UWorld. I saw the biggest jump in my NBMEs by doing those two things. In my opinion, my exam was most similar to a combination of Free120 and AMBOSS-style questions. I used UWorld because everyone does and it's the gold standard, I was too afraid to do anything different, but when I was in the test I felt like it was AMBOSS questions, which I've done to prep for in house exams.
User:Thank you for sharing! Would you recommend to do 4 and 5 hammer questions in amboss too?
User:I’m not familiar with those, but I recommend doing the “200 concepts tested on every Step 1 exam” on AMBOSS, as well as the Biostatistics concepts one. I think they’re under “study plans” on there. I didn’t get through but I think it would have prepared me more for the style and length of questions on the real thing.
User:I truly felt like questions on the actual exam are what you make out of it. Like my half my first section was impossibly hard as their were questions where I did not recognize a few of the answer choices but then mid way through the first section I realized it looked like uworld ( just the screen) and I had the thought of man I can actually common sense a lot of these questions unlike u world and from there on out the exam became really easy. Like dont get me wrong there were still some hard questions here and there but I just never let them bother me as I knew I had done a good 50/50 on most of them based of previous practice exams. After that the time ironically flew by like 8 hours felt like nothing. I went home maybe stressed for a day and just waited till today for results and I passed. If I could do dedicated again I think day one I would read chapters 4-18 of pathoma and try to note down stuff I really didnt know. Im not saying do the anki for it bht I feel like their is such a high focus chapters 1-3 and the anki that people kind of just never go through other chapters but ironically other chapters provide a really good base knowledge for everything you will need pathology wise.
Post 87: Cardio tips
https://www.reddit.com/r/step1/comments/1kme910/cardio_tips/
14/05/2025, 09:45:39
3 comments
Hey everyone! So I’m going to start cardio since it’s super dense and one of my weakest subjects. Can yall let me know what the highest yield topics are, (I’m going to study everything, just want to start off with the highest yield/must knows and then fill in the gaps!)
Comments:
User:Heart failure, murmurs
User:HF, MIs, Arrhthymias. HTN
User:Look up murmur man and never get another question wrong
Post 88: IMG results email
https://www.reddit.com/r/step1/comments/1kme6g1/img_results_email/
14/05/2025, 09:42:09
4 comments
Anyone got the email yet
Comments:
User:Not yet! I dont know why tested on may/02
User:Maybe it’ll be next week
User:US MD tested 5/2-5/3 (1.5x extra time) and didn't get any email yet), permit hasn't disappeared either
User:I did
Post 89: OMG I PASSED
https://www.reddit.com/r/step1/comments/1kmdpee/omg_i_passed/
14/05/2025, 09:18:51
62 comments
Thanks to everyone in this subreddit for being super supportive!!! Wish you all the best and hoping everyone to PASSS!!!!!!
Comments:
User:GUYSSSSS I PAAAAAAAAASSED!!!!!!!!!!!!!!!!!!!
User:OMG CONGRATSSSS!!!!!!
User:congrats to youuuu!!!
User:Hearty Congratulations! Wish you all the best for Step 2.
User:you’re so kind, thank you!!
User:How did you check? I haven’t received any score report yet
User:you should receive an email (IMG here)
User:I am img too. You received your pass report?
User:just an email that my score was up and then I checked on ecfmg
User:Congratulations! You must be so relieved! I have to wait till next week 🥲
User:Congratsssss!!!!
User:EASY PASS LADSSSSSS. I am so happy, congratulations to all of us. We deserve this
User:Thankkss!!!!!
User:Heavy on ethics?
User:Congratulations 🥳, if you don’t mind can I ask what was your nbme and free120 score, i took my exam on 7may, but I don’t feel like i did well enough,
User:my scores were: 61% NBME 28, 61% NBME 29, 68% NBME 30, 65% NBME 31, 64% free 120. Never cracked a high pass. Just got my P today! My mentor said "trust my scores" and she was right -- the one caveat I'll give is that I do not struggle with test day anxiety, so I could reasonably trust that my scores wouldn't take a big hit from that particular aspect on the big day.
User:Yes test day anxiety can be real demon if not handled well, hoping for the best,
User:Congrats, babe!!!!! THE RELIEF YOU FEEL <3 <3 <3
User:Congratulations 🥳🤝🏻
User:Listen, feeling bad after the exam is common. Trust your scores and the hours that you have put in this shit. Don’t think about it now.
User:🤝🏻🤝🏻
User:Sure!! I also barely slept these two weeks waiting my anxiety was making me feel physically ill but i made it and as everyone says although i never believed them, trust your NBME scores!!! Here are my scores NBME 16: 74% NBME 20: 73% NBME 21: 70% NBME 22: 69% NBME 23: 79% NBME 24: 70% NBME 25: 68% (my first nbme as a baseline) NBME 27: 77% NBME 28: 73% NBME 29: 74% NBME 30: 73% NBME 31: 75% Free 120 (New): 70%
User:Thank you for sharing✨🤝🏻
User:Youre welcome!! Waiting for you to claim your PASS!!!!
User:Fingers crossed 👉🏻👈🏻
User:Do you feel like pre form 20 NBMEs are helpful? Never thought to go back that far?
User:Guys I didn’t get any score report yet 😭
User:When did you test??
User:Look at you! Congratulations Rooting this for everyone (Including me)
User:Thank youu!! Waiting for you to claim your P!!!!
User:Thanks also spill some tips and tricks
User:CONGRATULATIONS! AMG or IMG? Could you tell me when did you take?
User:THANK YOUUU!!! IMG, 28/04
User:Excellent!
User:How did you check? I haven’t received any score report yet
User:Yay congrats!!
User:Thank youu!!!!
User:Congratulations, want to hear about your preparation, can u share , what to study in last 4 weeks ,
User:Sure! My prep was only from FA, bnb for concepts i didnt understand (and exclusively for obg/yn and neurology), sketchy only for micro and used it for pharma for things i didnt understand. Been preparing since December. Uworld did 100% with score of 68%. Last 4 weeks were just NBMEs , every other day or sometimes everyday i’d take one, NO UWORLD (i finished uworld 1 month before my exam to give th last month only for NBMEs)
User:What abou time management? And ethics what sources should i use , pharma and ethics are my weak araes , and what nbmes did u attempt, and did you review all nbmes in last month ,
User:My 27 nbme score in 66 and 28 is 69 ? How did u find it
User:For me, nbme 27 was SOOOO EASY but for example 22 was SOOO HARD For time management always practice with time and stimulate the real exam environment always. Never studied with tutor mode or untimed thats how i never had a problem with time during practice or in the exam
User:Alright, So what would u suggest ? should i go for 21-22 till 31 nbme ?
User:From 20-31
User:For ethics, AMBOSS and Uworld are the best sources for pharma i just used FA and ofcourse i do1 nbme and review it the next day thats why i used to do them every other day for in the last month before the exam
User:I passed too! I sat it 30/04 and if anyone has questions lmk! my scores: NBME 26: 64 (03/09/2025) NBME 27: 69 (03/20/2025) NBME 28: 58 (01/21/2025) NBME 29: 65 (04/06/2025) NBME 30: 70 (04/12/2025) NBME 31: 65 (04/22/2025) Free 120: 74 (04/26/2025
User:Congratss!!!
User:Hey if you don’t mind, what do you think have you that spike for the free 120? I’ve been scoring in high 60’s with one 70 on NBMEs so far, but I’m hoping to get a little boost before the free 120
User:I spent time reviewing each question in my NBME, esp 29, 30 and 31 and I did a lot of amboss and UWorld questions which tends to have long stems like the free 120 so I think that helped to know how to pick out the relevant info. If you have any other questions lmk!
User:Congratssss everyone!!! Amazing newsss!!
User:Thank youuuu!!!
User:Thank youu!!!!!
User:[removed]
User:Wishing you the best!! Thanks!
User:Congratulations!!!
User:Thank youu!!
User:Congratulations
User:Thank youu!!!
User:Yooo congratulations. What was your study plan??
Post 90: Do you guys get your result now?
https://www.reddit.com/r/step1/comments/1kmdit2/do_you_guys_get_your_result_now/
14/05/2025, 09:09:41
21 comments
Waitng email... I'm so nervous.
Took my exam on 02 May.
Comments:
User:Yes I PASSED!!!!!!
User:Congrats buddy. May I ask when did you take your test?
User:Thanks!! 28/04
User:Thank you. Congratulation for your passing again. Hope I get a P too.
User:Praying for you!!!
User:Congratulations 🥳, can I ask you what was your nbmes and free120 score
User:Thank you!! NBME 20: 73% NBME 21: 70% NBME 22: 69% NBME 23: 79% NBME 24: 70% NBME 25: 68% NBME 27: 77% NBME 28: 73% NBME 29: 74% NBME 30: 73% NBME 31: 75% Free 120: 70%
User:That’s superb! I took my exam on may7th , so kind of nervous, after doing exam I felt like i didn’t do well enough, hoping for the best, thank you for sharing!
User:I barely slept after the exam from anxiety. I felt physically sick from the anxiety and felt like i would’t make it but Alhamdulellah I DID! Im sure you will too!!!
User:How did you check? I haven’t received any score report yet 😭
User:They sent me an email that my score report is available.
User:Congratulations again! I think mine comes out next week mostly 🥹
User:Thanks!!! And good luck for the journey ahead
User:Yep. Does anyone know if you are still in the eligibility window and have taken step 1, will it let you schedule another exam at Prometric with the same permit if you failed? Any insight would be helpful. Right now, when I put in my permit info (expires June 30th) it does not let me schedule and says my permit info is incorrect.
User:Is it not 11am ET?
User:I did not receive email yet.
User:do you receive email?
User:No I didn’t. probably next week I guess.
User:May 02 -> next week probably unfortunately
User:Yeah. Another week to go 😢
User:do you think may 08 will also be next week or the week after 😭
Post 91: 30/4 results - are they out?
https://www.reddit.com/r/step1/comments/1kmd6rx/304_results_are_they_out/
14/05/2025, 08:52:31
10 comments
Are results out????
Comments:
User:Tested 4/28. Just got the result. Passed!
User:Congratss! U got the RESULT? Isn't that released at a 11am est
User:How did you get it early? Are you img?
User:I’m IMG, but I took the test in US
User:Congratulations!!
User:nope not yet
User:tested on the same day, didnt get an email yet
User:Not yet
User:Result out .. too much anxiety to open
User:I passed. Reddit has been the best, everyone of you had played a part in it Feel free to message me for any query
Post 92: Need help with registering
https://www.reddit.com/r/step1/comments/1kmd026/need_help_with_registering/
14/05/2025, 08:42:29
2 comments
I’m currently attempting to register for ecfmg certificate, but I am not able to get the mail for Usmle id in the first place
Idk what to do , should I try with a different email or something?
Comments:
User:Like how many days you’ve been waiting to receive ID?!
User:It’s been more than 5 days , and I just refilled the form again and realised I have been clicking the wrong link and filling the wrong form. So , would it be a problem or something?
Post 93: When are the results ?
https://www.reddit.com/r/step1/comments/1kmbg1p/when_are_the_results/
14/05/2025, 07:07:01
1 comment
Tested on 9th May, when are the results expected?
Comments:
User:I tested the same day, the permit disappeared but I don’t think it’s predictable anymore. We’ll get it on the next Wednesday or the Wednesday after.
Post 94: Prometric booking?
https://www.reddit.com/r/step1/comments/1kmbflf/prometric_booking/
14/05/2025, 07:06:07
1 comment
I booked the exam from prometric and they sent me a confirmation email with all the details on my email. Is there something else I need to do before I turn up to the center? Do I need to do something on the ECFMG or prometric website?
Comments:
User:You won’t need this confirmation email, just the scheduling permit and your ID.
Post 95: Who has got email already?
https://www.reddit.com/r/step1/comments/1kmbdrl/who_has_got_email_already/
14/05/2025, 07:02:39
33 comments
Guys if anyone got their email please inform here so that I can know when the e-mails are being sent 🥺
Comments:
User:US MD got the email overnight
User:When did you take the exam?!
User:4/26
User:Def you’re gonna be the first one who gets the Pass
User:Thanks I hope so! Terrified rn
User:Good luck on getting the P!
User:Did you get the results yet?
User:No at 11 am ET
User:did anyone get the results
User:[deleted]
User:Good luck on getting the P! U got this. Just keep distracting untill 11 US MD?
User:Yessir US MD. And you too, good luck!
User:Thanks hopefully I'll get it today too🥺
User:Me too man 😭 hope we both pass!
User:Old IMG, tested 4/28, just received the result. I passed!!! I was so freaked out!
User:Hey congratttss! U got the RESULT already? Isn't that supposed to be released at 11am?
User:passssssss
User:Congratssss! When did you give the test?
User:4/28
User:For some it was even 6:30 am EST some others later It inconclusive
User:No! IMG here, took it on 05/02
User:I tested on 5/3 IMG too. Hopefully we get it today 🥺
User:You took it in the U.S.? I did
User:So I was doing the math like technically I took the exam 12 hrs earlier since that is the time difference. I'll just do whatever the math to get the results today lol
User:No in the Philippines
User:Thank God, I got the P...stressful 2 weeks of waiting
User:Congratssss
User:not yettt
User:When did you take the exam?
User:Is it not in 2 hours?
User:I heard some even get theirs before 8 hrs or smth
User:I got it at like 2am and i happened to be awake
User:Got email this morning, took on 4/30
Post 96: Anyone got the email? (Not the results)
https://www.reddit.com/r/step1/comments/1kma2df/anyone_got_the_email_not_the_results/
14/05/2025, 05:27:41
20 comments
Kinda heard that NBME sends us an email that we will get the results today
1/5 taker, but got nothing by far
Comments:
User:Tested 29/04 did not get an email yet
User:Me too
User:Same, tested 2/5, USMD and no email yet
User:Last wednesday I saw someone who tested 25/4 get the results Maybe that means we will get it today .. or not 🥹
User:Whole process is honestly so annoying
User:Probably means next week for us then.
User:Can still be today, its possible to get the email later
User:Old IMG. Just got the result! I passed!!!
User:Tested 4/28
User:Results are usually delivered around 8.30pm IST
User:🙏🏻
User:Is it not between 5:30-6:30?
User:When is the last time for them to send the email for them?
User:US MD tested 4/26 and got the email around 2 am EST
User:Tested 5/2-5/3 (1.5x extra time) and didn't get any email yet), permit hasn't disappeared either
User:i think they declare result btw 5:30 - 6:30
User:My friend who got his results last week got his email about an hour before his results came out so maybe there’s still hope
User:Tested 5/2-5/3 (1.5x extra time) and didn't get any email yet), permit hasn't disappeared either Feels like we're gonna get GTA 6 before I see my test results
User:So real lmaoooo. Everyone sees it in two Wednesdays we gotta wait 3 bc NBME said f yall specifically
Post 97: Need answer to this qs
https://www.reddit.com/r/step1/comments/1km9v3n/need_answer_to_this_qs/
14/05/2025, 05:12:44
11 comments
Whyy is he even going to discuss About ptss??
Can anyone explain this?
Comments:
User:People on the same care team can talk about a patient, however they should do it in the proper setting, like at morning rounds and not in a public elevator
User:Can I know how you said that the student is on the same care team?
User:It says it in the stem
User:got ya!
User:The answer you chose. A is kind of rude idk. Belittles the student. Wouldn’t my attending to speak like that
User:Eh…with these kinds of patient data privacy questions I think its pretty common for the robotic “these are the rules” type answers to be correct (ex you’re talking to another doctor on another team), but in this case its more important that the stem tells you that the student is on your team.
User:Yes it is, but I idk why I did not notice it at first
User:happens. I still mess these up for step 2 🥲
User:If your colleague asks about a patient who is in the same medical team, you can share patient information with them but in a private setting. And if your colleague who isn’t a part of the same care team asks for any information, just say you can’t share these informations with them or confirm the individual is actually your patient or not.
User:Got this same exact question wrong today…..fuck uworld
User:All the answers here are incorrect. The only correct answer is that you should not indicate in the elevator that you possess any information about the patient. Doing so would imply you are the patient’s doctor, and that information is confidential. You must not reveal this fact. Therefore, option A is incorrect.
Post 98: good NBME scores but still freaking out
https://www.reddit.com/r/step1/comments/1km9g68/good_nbme_scores_but_still_freaking_out/
14/05/2025, 04:42:28
10 comments
so i recently gave NBME 25, got 70% with 65% uworld done. I have yet to do biochem, biostats and ethics. and genetics . Exam is on 1st july and I'm kinda freaking out if it's even possible to do all this along with fa review and solving the remaining NBMEs. Is it realistic to do all this in like 7 weeks? Would really appreciate it if you guys can help me put things into perspective.
p.s. as in img i have been studying for the step since last year along with my medschool exams and clinical rotations, and I'm honestly just tired and burned out. I just wanna be done with the test FFS!!!
Comments:
User:I totally understand your situation. Do you think you can do the test without FA revision? Am currently in a similar dilemma, have made my peace with not completing “EVERY” thing. Say 95% UW, last 5 NBMEs and F120 will be enough in my knowledge. Idk but if I score well in F120 I might just skip the last NBME and be done with it. I’m unable to put in the hours. Just study 5 hrs/d (coming from someone who used to put 12-15 hrs/d for uni exams) Also, answering your ques- yes,it will be possible to do all that in 7 weeks. Just do NBMEs alongside, don’t postpone them for the sake of completing UW.
User:I am in a similar situation and I totally get what you mean by 'EVERY THING'. Although, I am still struggling to make my peace with it. I don't know if I can study everything in First aid, or finish entire uworld 100%, but I know I cannot rely on any amount of luck based on my past. I am also struggling to trust my NBME scores because it feels like there is still so much left to study yet. I have my test in July.
User:Mine is in Mid june🥲 All the best!
User:Good luck to you too!!!
User:Yeah the desire to do "everything" and then letting it screw your mental health over if you don't do "everything". All too real 😔.
User:i don't think it's really worth it to not revise FA. Although instead of reading first aid i like to solve them FA tagged flashcards from anking. I guess my best bet would be to do NBMEs preferably instead of completing UW.
User:How did you get such a high mark without biochem half the exam was biochem
User:I didn't really see a lot of biochem on nbme 25. I did get one or two biochem questions right as the questions were a bit intuitive plus i had some vague background knowledge of the concept asked. I
User:Possible and as long as you're prioritizing nbme over uworld on the last 2 weeks. If you score atleast another two 70's or almost, then you're good. Predict my step score might help if you wanna ease your mind
User:Tysm. I guess that's the plan.
Post 99: Mehlman
https://www.reddit.com/r/step1/comments/1km9ef8/mehlman/
14/05/2025, 04:38:53
3 comments
Is it better to read the mehlman pdf or do all mehlman Anki for each module because I cant passively read pdfs
Comments:
User:Do mehlman Anki and Rv First Aid
User:Thank you!!
User:Where can i find the mehlman anki deck
Post 100: Saturday test takers
https://www.reddit.com/r/step1/comments/1km7ncm/saturday_test_takers/
14/05/2025, 02:38:46
2 comments
People tested on 26/4. Did you all get ur results last week?
Comments:
User:US MD tested 4/26, got the email that results will be released today
User:Good luck!
Post 101: When will result come?
https://www.reddit.com/r/step1/comments/1km4alj/when_will_result_come/
13/05/2025, 23:26:50
16 comments
I gave the exam on 11/05. Will it be two weeks or three weeks that it comes?
I am freaking out here 😭
Comments:
User:I gave the exam on Wednesday, 30th April. Can anyone please guide if I'll get my result today or next Wednesday? I'm going crazy
User:I also had the exam on April 30. I think, as IMGs, we don’t receive that email. It’s just a notification that the results are available, which I haven’t received yet either. It’s 5:00 AM EST, so they’re still outside of working hours. I hope we all pass inshaAllah 🙏
User:Did u get a mail?
User:Noo, not yet. But I'm an IMG from Pakistan and people here were saying IMGs don't get the mail?
User:I have no idea I am also from Pakistan so scared
User:We'll all get the Pass IA
User:I am scared I may have given a premature attempt 💀😭
User:11/05 here too.
User:Dude I am scared shitless 😭
User:Same. Exam was hard for me. Guessed most of the questions.
User:I felt like I knew some and didn’t know some that’s how it was 🫢
User:So u think we will have it on Wednesday the upcoming one
User:Probably yes. Fingers crossed 🤞 Best of luck
User:How will these days go by 😭
User:I mean the exam was the shortest 8 hours but these days are gonna be long af
User:Ikr. Hopefully we will get through this too 🥂
Post 102: Hi! So I filled out form 186 and got the notary cam stuff done as well as paid the fees. Now I’m wondering if this is normal - will they take two weeks to process and then submit to me Form 183 on mail? Or what? I want to get this done asap before my college closes for the summer vacation!!
https://www.reddit.com/r/step1/comments/1km3y1z/hi_so_i_filled_out_form_186_and_got_the_notary/
13/05/2025, 23:08:36
8 comments
I am an Indian IMG MS3
Comments:
User:They submit the form directly to ECFMG
User:I don’t have the form 183 w me and that’s the only way I can do it Will ECFMG send me the form 183 in two weeks as a pdf and then I fill it up and take it to my school?
User:Get it touch with your school. They’ll have to tick to a few checkboxes from their end!
User:My school isn’t connected to the online server option - I have to use form 183 only But that wasn’t sent to me so how do I get the verification done?
User:You should have gotten it at the end of your application after paying the fees. You can email them and ask them to send it to you.
User:Where would I find the mail to contact them? Is it any specific mail or just the contact us mail on their website?
User:I was in a very similar situation Inspite of me going to the authorities multiple times, it still took ECFMG exactly 2 weeks to give me my scheduling permit
User:Did you get form 183 in the mail? Something like that?
Post 103: Just looking for some advice
https://www.reddit.com/r/step1/comments/1km3eiw/just_looking_for_some_advice/
13/05/2025, 22:40:56
6 comments
Testing in ~10 days.
64% of uworld done with 64% correct.
3/5 NBME 28: 67%
9/5 NBME 29: 72%
Plan on taking 30 later this week, 31 early next week, and free 120 mid-late next week.
Any other recommendations or advice on how to proceed the next week or so?
Edit: dropped to 69% on NBME 30
Does this change anything?
Comments:
User:In a similar boat; albeit with slightly lower scores. My plan is to keep reviewing weak points + previous NBMEs in between my next tests. Also gonna rewatch pathoma 1-3 at 2x. Review sketchy images, and go through Mehlmann HY arrows + Risk Factors. Don’t forget to skim through the HY images pdf before your real thing.
User:Continue with what you’ve been doing - it seems like it’s been working out for you. Did you have a particular concern?
User:Just that I have only been doing random 40 blocks and not necessarily focusing on what I feel to be weaker areas. Not sure if this will come back to hurt me later on. Also just looking for any tweaks to my planned schedule if need be
User:Random blocks are the way to go since that’s how it’ll show up on the exam - it’ll get you used to synthesising information. Your scores are fine… barring an uncharacteristic larger drop, you should expect to take the exam on your scheduled date
User:What would you consider an uncharacteristic larger drop..?
User:I’d say anything beyond the low pass range (~62). Any other drop is likely due to burn out rather than not knowing content. But you know yourself best
Post 104: Low NBME scores
https://www.reddit.com/r/step1/comments/1km1q7z/low_nbme_scores/
13/05/2025, 21:16:24
2 comments
one month out from sitting. 3 exams averaging 53%
how do i improve?
content review vs questions?
Comments:
User:Dirty medicine
User:Mehlman videos and pdf
Post 105: How to use NBME to adjust studying
https://www.reddit.com/r/step1/comments/1km125t/how_to_use_nbme_to_adjust_studying/
13/05/2025, 20:44:23
1 comment
Hey guys! I feel like i post in here every other day :D. So I took an NBME and got a 64%, which i am fairly satisfied with as it was my first one and I have a little less then a month till my exam. I was wondering, how do you guys utilize the score report to adjust your studying? Right now I'm in the phase where I'm trying to bang out as many Uworld questions as possible + reviewing them. I was thinking to maybe do 1 set focused on things I'm weakest on? Or would it be a good idea to go back to FA/Pathoma and re-read the things I may have forgotten? LMK PLZ
Comments:
User:Yes, I went back to FA and annotated subjects that I was weaker at - focusing specifically on topics that I felt shakey on (ie. Cardiac curves for CV etc). I’d also use other resources like Pathoma or YouTube videos or ChatGPT to explain things differently. Make Anki cards based on your incorrects and what you learn during your reviews (if Anki works for you). If you have access to a secondary Qbank like Amboss (in my case, my school gave me a free subscription to Amboss), I’d do specific questions on those topics to test my understanding.
Post 106: sketchy micro and pharm pdf
https://www.reddit.com/r/step1/comments/1km0oxk/sketchy_micro_and_pharm_pdf/
13/05/2025, 20:27:11
1 comment
# sketchy micro and pharm pdf
Comments:
User:pirate bay
Post 107: Do it OR not: STEP 1
https://www.reddit.com/r/step1/comments/1klzx6x/do_it_or_not_step_1/
13/05/2025, 19:51:39
1 comment
Hi everyone, I need some advice.
I studied for STEP 1 for about 4 months and reached 67% on NBME 30 and 65% on NBME 31. At that time, I felt close to ready, but I didn’t end up taking the exam because of other commitments.
It’s now been 4 months since I last studied, and I’m planning to study again.
Do you think 2 months studying is enough to review and be ready to pass STEP 1?
I’d really appreciate any advice, especially if you’ve been in a similar situation.
Thanks in advance!
Comments:
User:You just need a clear plan and to stick to it. I think two months would be good, but it is better to make it four months to ensure the best result.
Post 108: Took the exam
https://www.reddit.com/r/step1/comments/1klz0mj/took_the_exam/
13/05/2025, 19:11:33
5 comments
took it on 12/May
is it normal to feel like I failed
when will the grade be out
Comments:
User:Wow, I’m sorry you feel that way. Most people feel this way and end up passing Fingers crossed 🤞🏾 How was your experience, if you don’t mind sharing
User:Yes and likely May 28
User:Totally normal. I was pretty nervous until I got my result. Takes 2-3 weeks
User:Took it on 11/05, haven’t stopped thinking about it since then. Feels like I’m screwed
User:[deleted]
User:yes
Post 109: Did you pass USMLE?
https://www.reddit.com/r/step1/comments/1klyvh2/did_you_pass_usmle/
13/05/2025, 19:05:22
2 comments
Please, anyone who has done USMLE this year, if you could share your NBMEs and the results of your exam. Mine are:
MBME 29: 63%
NBME 30: 58%
1 month later
Free old 120: 68%
2 weeks later
NBME 26: 70%
NBME 31: 63%
I am doing this to decide if I should schedule the exam 5 weeks from now
Comments:
User:Was 26 easy or hard?
User:Easier, NBME 31 has more vague questions, with non classic presentations of some diseases
Post 110: Step 2 studying advice
https://www.reddit.com/r/step1/comments/1klxxbh/step_2_studying_advice/
13/05/2025, 18:25:40
1 comment
I need step 2 studying advices and materials please
Comments:
User:This is step 1 bro
Post 111: Is BOOTCAMP Qbank similar to the real deal?
https://www.reddit.com/r/step1/comments/1klwqfb/is_bootcamp_qbank_similar_to_the_real_deal/
13/05/2025, 17:38:24
6 comments
Title
Comments:
User:I dont think its 100% similar but it def tuned me into presentations that I saw on the actual exam
User:Would it be possible to give me a rough idea % of the exam that’s similar to bootcamp?
User:hmmm I will say I did see verbatin some presentations from uworld from the CV, renal, and resp on my exam so I will say the presentations are very similar but not necessarily the stem type does that make sense?
User:Yes
User:haven’t taken the real deal but compared to uworld the questions are a lot more straightforward but the stems are longer so if you wanna practice longer stems u can try it out
User:Yes. I think between UW, AMBOSS, and Bootcamp, it is the closest to the actual exam. Of course, nothing is exactly like the exam. But I used Bootcamp way more than UW. Also loved that in the answer explanations it would link directly to videos if you needed content review.
Post 112: Exam anxiety😭
https://www.reddit.com/r/step1/comments/1klwfnw/exam_anxiety/
13/05/2025, 17:26:22
9 comments
Can someone please help me get over my exam anxiety, I’m taking it in a few weeks and I just can’t sleep (when I want to).
I’ve always been a night owl (10+ years). I’ve tried melatonin (3 mg but it seems I’ve become resistant now), chamomile tea, exercise, no screens before sleep (hard to follow since I study on iPad).
I’m trying to set my circadian rhythm but it has been over a month now. I’m scared and anxious coz both my study and sleep are getting compromised since I study best after 10 PM. I sleep from 5 AM to 1 PM, and I’m sleepy the whole day.
I end up crying myself to sleep most days, the thoughts just won’t stop. Idk what I’ll do the night before the real deal.
Also- please don’t be rude, I’m really sensitive and stressed at the moment. I’ll see a shrink for my anxiety after the Step (I can’t rn).
Comments:
User:Melatonin supplement every night same time. If sublingually then 1 mg is enough otherwise 5 mg 8 :30 pm. Take a pill form and crush in mouth usually they’re sweet and melt away in your mouth right away. Take it sublingually. If 1 mg sublingual is not enough try 2mg. Sleep at 9 . Wake up at same time every morning. Doesn’t matter how much you slept previous night. Know that anxiety is a protective mechanism. Watch a movie named as inside out 2. It would help you understand your emotions. Acknowledge your emotions. Your body must be trying to communicate something.
User:Okay. It’s usually this recurring thought - “what if I fail?” I know that it’s not in my hands and worrying about it won’t change a thing, but yeah, this thought keeps me up followed by the awful scenarios of getting a “Fail” email. My life is basically and realistically over if I fail.
User:I understand and it must be really hard to feel that way. I struggle a lot with exam anxiety. In my case therapy and medication helped me with these thoughts. I still take medication. However the real thing that helped me was knowing that what i can do is i am doing which is learning concepts and being consistent with it and getting good marks. I realized i was underestimating myself and my anxiety was arising from “ this exam is too hard and i feel like i am not enough, what if i fail”. I tried to do everything that could benefit me including setting routine, managing distractions and learning actual material n concepts along with therapy and medication. If you feel at any point it is overwhelming to the point where it is affecting functionality please reach out for professional help if you can.
User:I will, after the exam. Also, thanks for sharing this🥺 I hope to able to control it till my exam.
User:First at the day of exam you will feel you are super hero and you would do it just fine. The best way I found to help my circadian is waking up at the same time everyday at some point you will start sleeping early. Despite when you sleep wake up 11 every day and then make it 10 and so on. One more tip, be realistic at the beginning and aim for small changes and continue what you do no screen, excercise and no light etc.
User:Okay. Thanks. I’ll try this.
User:If you can’t help yourself to sleep, just chug some coffee and don’t sleep at 5am, push it to 9pm. Wake-up at 5-6am the next day. Yes, it will be brutal but works like a charm!
User:Okie. I’ll try this. Thanks.
User:USMLE - You are not alone (Motivational) https://youtu.be/MBW-BQgffgI To help you feel better ❤️🩹
Post 113: Took the beast today…
https://www.reddit.com/r/step1/comments/1klvvck/took_the_beast_today/
13/05/2025, 17:04:15
26 comments
Long time lurker on this subreddit that had already pre convinced myself that I was going to fail. Here are my thoughts after taking the real thing today:
-The test is extremely doable. Yes, there are some questions that are like what the actual f*ck are they talking about. Yes, those questions are probably experimental. There are also question that you will be shocked at how easy they are.
-Everyone says how long the stems are. The stems are exactly like the “longer” free 120 stems but every question is that length. That being said there are some questions you can see the lab values/image and know what’s going on without reading the paragraph.
-Ethics is not that bad. Ethics is exactly like the free 120 and NBME ethics questions.
-Sletchy micro and Mehlman, specifically the renal PDF, is pure gold.
All this to say, don’t be like me and get fear mongered into thinking you can’t/don’t have the ability to take the test. You’ll be surprised how doable it is.
Scores, for reference: NBME 29 - 66, NBME 30 - 72, NBME 31 - 73, f120 - 73
Comments:
User:was the micro more like very specific low yield bugs or a mix of everything?
User:I remember one specific low yield bug they asked about. Other than that mostly HY bugs but mostly on virulence factors
User:Thank you! Ppl have said that the topics were nothing like the NBME/Free 120, do you agree? Or would you say its worded differently? Also did you see alot on risk factors?
User:Idk I always though risk factors where easy questions so i didn’t realize if there was a lot or not. And there’s only so many concepts in medicine and some more important than others. The exam isn’t going to be filled with low yield bullshit questions one after the other, don’t worry
User:Thanks for this post. I’m testing on 5-19 and this post really breathed some life into me.
User:Hey! Can you mention any long stem question like how it was framed? People were telling very long lengthy ques with no buzz words... Just to clarify!
User:Have you taken the free 120
User:Not yet just started my nbmes
User:Once you take the free 120 then you’ll understand what I’m talking about
User:Thank you so much!!
User:How can I get free 120?
User:I’m very similar, i convince myself it’s probably beyond me but i’m still going to give it a shot. Given your prep time was 8 weeks you must have an amazing study plan or have had a non dedicated study period. Regardless, could you please share your timetable or any tips/ tricks that helped you do more in less time? I’m hoping to take the test in 3 months, but still figuring out my resources. Any advice on those would be appreciated as well!
User:The first 2 weeks I strictly watched sketchy micro and did all the associated AnKing cards. After that I just did 2 blocks of random untimed UWorld a day and put my incorrect into Anki. I only used first aid to review concepts
User:The ethics were kinda hard ngl
User:How were your scores on NBME 25-28? And how long did you prep for?
User:I didn’t take 25-28, 8 weeks
User:I understand it is similar to the nbme. However how does it compare to uworld from your perspective ?
User:UWorld is a learning tool meant to teach you specific points per question. These questions are all 2nd/3rd order questions testing understanding of broader concepts
User:How many questions per block are long stem questions The way people talk about it feels like all of them are long thats the only thing worries since i space out alot when reading long questions sometimes i read the same line 3 times 😂 Am doing well on nbmes but thats the only thing that concerns me
User:Every question is long
User:Damn am cooked 💀
User:congratulations
User:Hi, Can you tell what kind of questions came for Neuro and Biochem Thanks!
User:You must be a USMLE test writer the way you’re asking vague questions like that
User:They ask about the brain and how the body works, hope this helps
Post 114: About 2 days to exam, what to do tonight/tomorrow to solidify any last minute issues?
https://www.reddit.com/r/step1/comments/1klvnu9/about_2_days_to_exam_what_to_do_tonighttomorrow/
13/05/2025, 16:56:02
2 comments
2 or so days until my exam and panicking a little lol. I stopped Anki about 5 days ago and I am scared about forgetting things but my test scores have shown I have been keeping up with things well which is promising. I just wanna know if there's anything I can do to just help me feel a little more confident that won't be too demanding? Already planning to end early tomorrow so I can chill and sleep early enough to be well rested and recharged.
Scores have been upward since I started studying around the beginning of April (I have already done clinicals with my school schedule so that helped a bit with starting at a good baseline).
Started with 64 on CBSE 2 months ago and then really locked in 04/01.
Form 26: 66 (mid April)
Form 27: 71 (04/25)
Form 29: 75 (05/05)
Form 30: 79 (05/10)
Free 120 yesterday: 78 (78,83,75)
Feeling pretty solid and proud of my growth lol but just want to do something productive for the next day.
ETA: did about 50% UWorld with about 73% correct. Planned to finish it, but I really needed to focus on NBME and had already done 1500 questions for Amboss so at a certain point, I switched to just doing targeted sets like all of general principles or pathophys/path/anatomy on weaker areas. Also, coming off of rotations with shelf exams having such long annoying stems, I feel used to the question lengths.
Comments:
User:Feeling same exam in a week I score high on NBME 30(88% lol🤣) and feel proud of my progress but still need someone to tell me what to do for last week revision still have 31 and free 120 I will do them
User:Mehlman PDF for your weakest areas; dont read the first half, just skip to the part where he's doing scenarios
Post 115: Help with Anki
https://www.reddit.com/r/step1/comments/1klsuda/help_with_anki/
13/05/2025, 15:04:17
3 comments
Please, I am new to Anki and couldn’t find any YouTube videos explaining this. How do I retrieve the Pathology deck or any deck if I need to redo it on the same day? I would appreciate it if anyone could help me with images and instructions. Thanks!
Comments:
User:Click on Pathology, then press B on your keypad. If the cards are all show “yellow,” CTL/Commad + A, the CTL + J. If not, send the screenshot.
User:Thank you so much for your help; you relieved the stress I was getting from trying to figure it out on my own. I sent you a DM
User:My pleasure.
Post 116: Advice
https://www.reddit.com/r/step1/comments/1klsbuw/advice/
13/05/2025, 14:44:40
1 comment
Hi I have to take CBSE at the end of october and get above a 67 and then take step 1 2-3 weeks after. I currently have a 64% on bootcamp after doing 528 questions. Still feeling sorta weak overall but i feel like im in a good spot. I have all of october off as dedicated. How should I go about studying?
Comments:
User:How I passed CBSE (NBME Comp) and Step 1 using MM + UW + NBMEs https://youtu.be/3eRen9zBGxY Try this, good luck.
Post 117: DirtyMed Anki deck??
https://www.reddit.com/r/step1/comments/1kls0dp/dirtymed_anki_deck/
13/05/2025, 14:32:25
3 comments
I LOVE DirtyMed videos but I need a way to keep up with the content and not forget about it as I continue to study. Any Dirtymed anki decks out there? I'll take whatever I can!! Thanks in advance
Comments:
User:There's a dirty medicine deck on AnkiHub I believe.
User:i've only found an OMM deck on there
User:https://www.reddit.com/r/step1/s/wYIT5FF0KH
Post 118: CBSSA /NBME Bundle-How to purchase?
https://www.reddit.com/r/step1/comments/1klrjey/cbssa_nbme_bundlehow_to_purchase/
13/05/2025, 14:13:49
1 comment
For anyone that has recently purchased the NBME bundle, where exactly is this found? I only see the option to purchase one test at a time for $62 on the website.
Comments:
User:The offer ended on April 18th.
Post 119: Any 5/13 test takers?
https://www.reddit.com/r/step1/comments/1klqbrf/any_513_test_takers/
13/05/2025, 13:26:21
19 comments
I wanna know ur experience plz.
Comments:
User:5/11 here
User:Is it just me or was there too much micro today?? Psych,MSK, biostats ethics were non existent
User:Felt like there was lots of neonatal/ infant disorders
User:Too fkn much!!!!!!
User:How did ur exam go? Also why were there Soo many hopc long stems....took a lot of time to solve those and my last 2 3 questions were legit blind solves.didnt even read the questions properly.
User:Same exact thing. Even with the questions i felt that i could get the answer if i thought a little more I didn't give it the time, because basically there was no time! So even those had the blind guessing thing! Absolutely unfair.
User:Soo true....I hope we pass.im honestly scared to death.
User:What is your experience of the exam ?
User:i am not sure of anything, i think um not the right person to be asked..
User:Does the exam have a lot of ethics ? Long stem and these thing that people here say it?
User:Long stems sure but ethics was non existent.barely any biochem immuno biostats.
User:Not that A LOT of ethics! Long stems yes ofc. Try to practice this.
User:Same experience....I don't know what I did in exam.felt like I was blindly solving them as fast as I could.
User:[deleted]
User:Do you mind sharing a bit of your experience?
User:[deleted]
User:Mmm I see, I’m pretty sure you would be fine. Don’t think too much about it. Which systems/subjects were heavily tested tho?
User:[deleted]
User:Thank you I pray you pass Fingers crossed 🤞🏾
User:I was making educated guesses for the majority of the part😭 I had decent test scores but this form was genuinely out of the blue. Please give your experience as well.
User:Um not even sure if they were educated guesses or blind ones, and even the ones that i felt a little hope with, i probably missed the right answers because of the time thing. And i am not sure of a single answer. I never felt this in any practice.. in most practices i felt that i have some questions in my pocket. I feel very bad tbh.
User:I am just trying to not think too much about it. Majority of the test takers feel this way after the exam.
Post 120: How to Take Notes from UWorld Effectively
https://www.reddit.com/r/step1/comments/1klq66d/how_to_take_notes_from_uworld_effectively/
13/05/2025, 13:20:13
5 comments
1. Use Flashcards or a Digital System
• Flashcards (like Anki) are best — easy to search, review, and repeat.
• Digital notes help with faster access and long-term retention through spaced repetition.
2. Take Notes Only from UWorld
• Avoid the urge (OCD!) to make notes from BnB, Pathoma, or slides.
• Your focus should be to master UWorld — that’s your highest-yield resource.
3. What to Make Flashcards For
Make a card if:
• You got the question wrong.
• You were guessing or unsure and got it right by luck.
• You didn’t understand the underlying concept.
4. Keep Each Card Simple: One Concept = One Card
• Avoid long, dense cards.
• Don’t make flashcards with massive paragraphs trying to cover “everything.”
• That just feeds your perfectionism and makes you less likely to review.
5. Flashcard Example
Q: What enzyme is deficient in classic galactosemia?
A: Galactose-1-phosphate uridyltransferase
Cloze card:
Classic galactosemia is due to a deficiency of {{c1::galactose-1-phosphate uridyltransferase}}.
6. What Exactly Should Be a Flashcard?
Focus on:
• Mechanisms of action
• Causes of diseases
• Most important risk factors
• Key distinguishing features
• Important comparisons (e.g., Crohn’s vs. UC)
• Management or first-line treatment
7. Make Notes Active, Not Passive
Don’t write passive facts like:
TCA overdose → QT prolongation
Instead, write:
What ECG changes are seen in TCA overdose?
This builds active recall, which strengthens memory far better.
8. Use Tags in Your Cards
• Create parent tags like Cardio and child tags like IHD.
• This helps you quickly review cards by topic or organ system.
(I used chatGPT to organize this post)
Comments:
User:I just copy any high yield infor or uworld charts onto Uworld notebook. I have organised it for various subjects.
User:Any alternative to anki you’d recommend? For IOS users
User:Use website
User:For the first 50% of Uworld I would find the concept in FA, and read the section then add in notes from what Uworld was trying to teach me. That allowed me to get lots of basics down and fill in many gaps. Now for the last 50% of Uworld I am using the Uworld notebook to jot down HY factoids.
Post 121: How to Deal with Forgetting While Studying for the USMLE
https://www.reddit.com/r/step1/comments/1klq3yb/how_to_deal_with_forgetting_while_studying_for/
13/05/2025, 13:17:46
9 comments
1. Forgetting is Part of the Process
• It’s completely normal — don’t panic when you forget something.
• You’re not supposed to memorize an entire book perfectly on first pass.
2. Don’t Get Stuck on One System Forever
• Studying one system endlessly until it’s “perfect” is counterproductive.
• Move on — the key is repetition, not perfection from the first round.
3. You Will See the Concept Again — Many Times
• USMLE content is highly repetitive.
• You’ll encounter the same concept again in:
• UWorld (even in incorrect choices),
• First Aid,
• NBME exams,
• and even in other questions on the same test.
4. Don’t Mourn Forgotten Material
• Don’t dwell on “spilled milk” — forgetting is expected.
• What matters is that you review and reinforce over time.
5. Focus on Doing Your Best Each Round
• Aim to understand and retain as much as possible, then move on.
• Your brain needs multiple exposures to retain complex medical info.
6. Trust the Process
• With enough cycles of active learning and spaced repetition (like Anki or question-based learning), you’ll remember more than you think.
+ Extra Tips for Improving Memory and Recall
1️⃣
Comments:
User:Thats why i jerk off after learning a hard concept. Dopamine reinforces you to remember the concept. We have to think smarter
User:i audibly gasped and laughed in my lecture rn at this
User:Biohacking to the max
User:Dam things are starting to make sense
User:The funniest comment ever 🤣🤣🤣
User:7. Use Spaced Repetition (e.g., Anki) • Don’t expect to memorize everything in one go. • Anki will bring cards back automatically at the right time to strengthen memory. 8. Review Cards Daily — Even Briefly • Best times to review: • Before starting your study day (like a warm-up), or • At the end of the day (like a wrap-up) • Can’t review one day? No worries. Anki will reschedule them. 9. Keep Notes Minimal but Focused • Writing helps memory, but only if you write what matters. • Build a simple system (Anki, Notion, Google Sheets — whatever suits you) and stick with it. 10. Be Consistent, Not Perfect • You don’t have to review every card, every day. • Consistency over time beats intense short bursts. (i user chatGPT to organize this post)
User:Way too many Chatgpt posts on this sub now
User:https://www.reddit.com/r/step1/s/AXBEKDyUGF Please read the last phrase
User:Appreciate it
Post 122: I feel like I’m not doing enough questions — how do you balance content and Qs?
https://www.reddit.com/r/step1/comments/1klq1ra/i_feel_like_im_not_doing_enough_questions_how_do/
13/05/2025, 13:15:20
1 comment
I recently finished studying GIT. I watched videos and understood the content in about 5 days. There were around 220 UWorld questions for that system, but I only managed to complete about 130 in those 5 days. After that, I moved on to the next system, which also had a large number of questions.
I’m starting to notice a pattern: I spend more time consuming content (videos, notes) than actively doing questions. While it feels good to “understand” a topic, I know that actually doing questions is key for USMLE prep.
Has anyone else gone through this? How did you overcome the habit of over-studying and start prioritizing questions more? Any strategies that helped you strike a better balance?
Comments:
User:Early morning questions everyday. For a system in timed mode. Then set a time for explanation including charts and pictures. 10 minutes per questions. If you struggle with focus. Do it 10+10 set in the beginning. Keep the phone away. If you happen to not understand the concept the use chatgpt. Ask it to explain high yield step 1 related stuff briefly. Once you're done with questions in 3 hours max shift to videos. Bnb videos take a lot of time. You can try bootcamp as well.
Post 123: What to do the last 2 weeks?
https://www.reddit.com/r/step1/comments/1klpuqd/what_to_do_the_last_2_weeks/
13/05/2025, 13:07:38
7 comments
I have 2 weeks left of dedicated study and wondering if anyone could give me advice on what to do. Currently I have been doing a block of an earlier offline NMBE a day; redoing my incorrect on past NMBE my subjects; reviewing my sketchy micro/pharm decks. I plan to take a Uworld assessment tommorow( I heard its similar to the real deal); 30 next week then free 120.
also been studying for a while and now i feel like im loosing information.
Current Nmbe scores are
Mid February- CBSE: 40
March 4 NMBE 27: 51
March 18 NMBE 29: 51
March 29 NMBE 28: 57
April 26 NMbe 30: 60
May 5 NMBE 26: 64
Comments:
User:These scores will probably get you in the 40s/50s on uwsa.. do not recommend 2 weeks before exam they will kill your confidence..
User:Do u think I should just do Nmbe 25?
User:I havent taken my exam yet so I can’t give you advice (maybe even my uwsa advice was wrong.. please do you🤧) it’s just that I have your exact similar nbme scores and when I took uwsa I got humbled real quick and started thinking about postponing.. but people have been saying to focus on nbme 30-31 and free120w
User:You’ve shown a steady improvement in your nbmes so I don’t think I would do anything different. Stick to your plan, focus on the incorrects and you should be good
User:Why don you do free 120 (old and new)??
User:I heard there’s some similar questions. But I’ll consider that.
User:The length of the qs is supposed to be the same. I think you can do new free 120, review it, old free 120 review it, then read mehlman arrows pdf, risk factors pdf and if you still feel weak, take a day to review the top concept NBME qs of HY guru. Each video is about 1 hour, you can do all systems in a day to make sure you know all those concepts. Youre good to go with that i think. When are you taking it? Im taking it on the 30th
Post 124: Help!
https://www.reddit.com/r/step1/comments/1klmsvl/help/
13/05/2025, 11:05:58
29 comments
I'm unable to get questions like these correct. Just one month out and this part of the genetics is one of the weakest. Any resources, please?
Comments:
User:Pretty sure it is 1/2. She cannot give birth to a male, so that’s out. When giving birth to a female, she has one X that is faulty and one that is normal. There is a 1/2 chance that she passes the faulty chromosome to the child who will then be a carrier assuming the father is normal. EDIT nvm I’m an idiot I forgot she can give birth to a healthy boy it’s 1/3
User:You scared me first cause my ans was 1/3
User:1/3 If dad was carrier he would be dead/aborted child lol. Have to assume that he is alive to make a baby lol. Mom is carrier. She can give one recessive allele to daughter. Mom can successfully give birth to 3 children. 1 carrier. 1 non carrier boy and girl. Draw a punnet square. Stick to basics don’t over think.
User:C) draw a punnet square, you’ll end up with these combos, XX, Xx, Xy, xy. xy would die in utero, so really you only have 3 possibilities. XX, Xx, and Xy. Therefore 1 in 3 would be a carrier. Edit: spelling
User:Can you explain why xy would die in utero? I don’t get the question stem how to interpret which genetic combination would result in the spontaneous abortion.
User:It’s x-linked recessive and Leads to death in utero, therefore unless a healthy X could compensate the pathological x there’s no way the mother could give birth. In xy you have a Y chromosome which is irrelevant here and a pathological x chromosome -> death in utero. usually X denotes the healthy allel and x the pathological one
User:I wanna know too
User:becuase xrY dies; so theres only 3 left, and so only 1 of those 3 is a carrier
User:Is it 1 in 3? I asked chatgpt and it explained it for me But I guess you have to assume the man is normal, I've not done any genetic questions but with this assumption the question seems clear The mum is XdXN The dad is XNYN Where the N = normal and d= the recessive allele So we have 4 combinations 2 daughters of which one could be a carrier and the other can be unaffected. XNXN or XdXN 2 son combinations where one would be unaffected XNYN and the other would have XdYN meaning is affected and dies before birth hence 1/3 as only 3 children could be born Of course a male is XY and a daughter is XX
User:Only females children can be carriers And the question asks the probability of a carrier child being born So 1 in 2 Hence option B
User:I think it still has to be 1 in 3. The question asks what is the probability that the next "child" is a carrier. This child could be a boy which would be normal She could give birth to a girl which is normal She could give birth to a girl that's a carrier So the answer should still be 1 in 3 because we still have to account for the probability of a girl being born Another way could be this: 1) What is the probability in this specific case that a girl will be born: 2/3 (Because 2 out of the 3 scenarios that result in a live baby require that a girl is born) 2) What is the probability that this girl will be a carrier: 1/2 3) So, 2/3 x 1/2 = 1/3
User:Yeah chatgpt lied to me but you would be right in which case this question becomes even easier I guess
User:It’s 1/3, option C. It’s the probability that next child **she gives birth to** is a carrier. Do a punnet square, 1/4 chance aborted male. Of the 3 children **she could give birth to**, two are healthy (1 male 1 female), and one is a carrier. 1/3 chance
User:For those that are visual learners, I drew it out [here](https://www.reddit.com/r/AmericanClinicals/comments/1km0lyy/solution_to_rstep1_post_on_genetics/): [https://www.reddit.com/r/AmericanClinicals/comments/1km0lyy/solution\_to\_rstep1\_post\_on\_genetics/](https://www.reddit.com/r/AmericanClinicals/comments/1km0lyy/solution_to_rstep1_post_on_genetics/) Hope it helps.
User:It’s 1/3. If we do a Punnet square it’d look a bit like this where X1 is the carrier gene X Y X XX XY X1 X1X X1Y X1Y would die in utero, so that only leaves 3 viable offspring. As such X1X is the only carrier and comprises 1/3 of the live offspring. I apologize if this isn’t the best Punnet square, but I think hopefully the message comes through.
User:Option B
User:The correct option is C according to the PDF. But I'm unable to get such questions right. Any video resource or anything to strengthen the concepts?
User:In Randy Neil we trust! But fr his genetics videos, just full send and watch them all while locked in and you should get more correct
User:I think it is 1/3. \- we assume the original gene is x'x, which is x' for faulty x and x for normal x. \- we assume father is heathy, and he is xy \- x'x -> x'+x, xy -> x+y => 1.x'x(carrier) 2. x'y(dead) 3. xy(healthy) 4.xx(healthy) \- carrier= 1(x'x)/3(x'x+xy+xx) = 1/3 apologize for my poor english if i made mistake grammar.
User:Oh no, it can be 1/2, sry \- i read comment below, i found maybe it has a hidden consumption, which is only female can be carrier(x'y in male is deadly and xy in male is healthy, we don't need to discuss and guess these circumstances) \- so , it must be 1(x'x)/2(xx+x'x) \- i.e. we talk under an only circumstances that only female can be carrier
User:i think this question is not say accurately
User:Why is the male deadly (x’y) ?
User:\- because it is x-linked recessive disorder, which means x' is a disorder gene, x is a healthy gene. \- female has two xx, so she will dead only under two x are both x'x', when it carries only one x', she is carrier, when it carries two xx, she is healthy. \- But male is xy, which means it only have one chance on x (because y is another chromosome, there is no relationship between them), so if x is x ,he is healthy, when x is x', he is dead.
User:Since woman can only give the diseased X chromosome to only 2 of her births cause she is carrier, and since it is fatal disease meaning no male birth will be Alive, Then 1 out of the 3 alive will be female baby who is carrier like the mom. The answer is C\ 1 of 3
User:1/3 ??
User:The key word here is carrier. Mothers have 2 X one faulty X is compensated for by a normal X. But when yoh only have 1 X and no other X to compensate you get the disease i.e XY. For the child to be a carrier she has to be a female. So the likelihood the child is a carrier is 1/2.
User:1/4 ; 1/2 to be a boy, 1/2 to have the disease 1/2x1/2=1/4
User:Hi for this question we know that the mom has to be genotype X’X, (given X’ is recessive allele). Since the question states that the disease is lethal in utero we know that anyone who is affected by this disease is not born, thus the father has to be XY. When we draw out the punnet square we can immediately cross out the X’Y probability since we know they won’t be born due to them being affected by the disease, and thus won’t be factored into the probability. That leaves us with XX, XY, and X’X. From here we can see that there is a 1/3 probability that the next child she gives birth to will be a carrier.
Post 125: Someone please save me for MSK and Neurology. Bnb or Physeo?
https://www.reddit.com/r/step1/comments/1klmpcq/someone_please_save_me_for_msk_and_neurology_bnb/
13/05/2025, 11:01:50
13 comments
I have tried bootcamp for both and hated it. First aid and Mehlman looks so dense.
Do you think only Bnb or Physeo would be good enough for MSK and neurology? Which one do you recommend?
Comments:
User:MSK : Physeo alone will do. I swore by it till the very end. Disclaimer : I dont really like msk, so couldn't really stand multiple resources for it lol. But I did end up passing, so yeah, should work. But do know, that my form was kinda heavy on msk, all the peripheral nerve injuries and all Neuro : physeo alone is not sufficient. I personally loved Kaplan's neuroanatomy lectures. Long and tedious, sure. But worth it. Swore by it even for revision. Coupled it with mehlman's neuroanatomy pdf. Went through it twice. Nice one, too. And uworld too, goes w/o saying. And oh yes, Bnb's video of the rule of 4's for brainstem strokes is golden. Disclaimer again :3 - I love neuro, specifically above the neck neuroscience, so yeah did not mind playing around with sources or even sticking to multiple ones for that matter
User:Oh thank you so much! Did you only do Physeo for MSK and found it enough? Or did you read FA or Mehlman as well?
User:Oh hah, I almost forgot about FA. yes, I used FA for msk as well as neuro. Both are nicely given in it.
User:I think bnb neuro is amazing and the best out there although it lacks a bit in the neuroanatomy part. For that bootcamp has a dedicated section and OP can look into that and then start bnb
User:For sure, haven't personally used bootcamp ever, so cant comment on that, but have heard a lot about it, must be good
User:Bootcamp neuro is literally everything used it for my step prep
User:brute force practice questions and anki
User:Physeo definitely
User:Kaplan Neuroanatomy Dr White lectures are best
User:Uw pictures and uw itself is very good as well
User:Open amboss articles —> read them and pick a few Neuroanatomy pictures ( functional areas of the cerebrum, coronal sections of basal ganglia and whatever you fancy ) —> image occlusion anki cards —> hammer the cards for 2-3 days —> start “Neuro proper” You’ll have a better idea of Neuroanatomy before hitting the pathologies, especially strokes.
Post 126: rollercoaster
https://www.reddit.com/r/step1/comments/1klmovs/rollercoaster/
13/05/2025, 11:01:20
4 comments
20/4 form 26 = 57%
28/4 form 27 = 52%
4/5 form 28 = 61%
8/5 form 29 = 55%
13/5 form 25 = 74% (offline)
Scheduled to sit in 11 days...
wtf, can I trust 25?! quite a few repeat Qus, but I thought most of them were 'easy' ones I got right the first time anyway...
I know I need to back it up - planning to sit 30 in a couple of days (followed by 31 and new free120) - I did 25 because I didn't want to keep burning names on crap scores.
I don't know what to think or do! Maybe I'm just crap under pressure, I took 25 under exam conditions but a PDF is less scary than the NBME website.
Comments:
User:I would postpone. See dirty medicine videos
User:You can still do uwsa tests, RX practice tests, free120, nbme 30-31 The real deal won’t be a PDF, so u gotta stimulate the testing environment to know if you’re truly ready
User:Take two more 30 or 31 and free120 and if you’re consistent 70s then go for it. Also this looks like a mindset issue, try to repeat the same diet and rituals that you scored good in
User:Most predictable: Forms 29-31 (take online) and the New (2024) Free 120. Check all your correct and incorrect answers. If your scores cross the 69-70% (EPC), you are ready, otherwise it is risky.
Post 127: Help to a helpless human
https://www.reddit.com/r/step1/comments/1kll4uc/help_to_a_helpless_human/
13/05/2025, 09:51:26
1 comment
Does anyone have the anki decks for Sketchy micro and pharm? Pls 🫶
Comments:
User:Just download AnKing it is included
Post 128: 8 weeks until my step 1 exam and I'm not studying IMG
https://www.reddit.com/r/step1/comments/1klk74w/8_weeks_until_my_step_1_exam_and_im_not_studying/
13/05/2025, 09:05:33
1 comment
I did no content review but did 50% of the uworld till now but I don't feel like studying anymore like it's not imp or I dunno what anybody has been through such slump T.T ...plus share experience..
Comments:
User:If you’re feeling a slump and need to take a break, do that. But don’t rush into the exam until you’ve done a few practice tests and content review in FA. Sharing my experience here! [https://www.reddit.com/r/step1/s/bvdkhEodfL](https://www.reddit.com/r/step1/s/bvdkhEodfL)
Post 129: Wrote Step 1 waiting for Results
https://www.reddit.com/r/step1/comments/1kljryi/wrote_step_1_waiting_for_results/
13/05/2025, 08:43:34
13 comments
I walked out of the exam feeling alright, brain was absolutely cooked but exam itself felt alright. But now looking through this reddit, I'm worried that maybe i overestimated how the exam went. Everyone seems to think they absolutely failed then pass. Kinda worried that maybe i just missed the tricks in the questions. idek I wrote it pretty early for my med school year because my results we're all decent. Haven't really been thinking about the test until now.
How long does it take to get your results? Is it always 2 weeks after your exam or does it depend on the day of the week you wrote your exam?
\*wrote mine 7th of may
Post 130: Spent a year studying, poor NBME scores
https://www.reddit.com/r/step1/comments/1kljnyp/spent_a_year_studying_poor_nbme_scores/
13/05/2025, 08:37:20
12 comments
Hi guys I’ve spent more than a year doing my prep, (got serious around a year ago). I have my exam booked for Mid June.
Took NBMEs about 3-4 weeks ago and scores are as follows:
Form 26: 69%
Form 27: 65%
Form 28: 66%
Gonna be taking 3 more (29-31) soon but how to improve my scores and will I really be ready by Mid June?
Comments:
User:ur gonna be fine
User:I'm testing mid-june and lurking this sub to see if I'm ready as well. I've got similar NBME scores and I believe some HY reviews/covering weak topics/ learning from nbme mistakes will go a long way in helping us get that 70+ for safety. We got this man, goodluck!
User:Sound advice. Thanks! Best of luck to you too!
User:Same boat Same boat
User:Review NBMEs more thoroughly
User:Hey man Same here Studying for step during my med school in-class exams and i have similar scores. Taking the exam right about that time. Right now i’m planning to finish my finals and do the rest of the nbmes with content review from dirty + hy arrows pdfs and free 120 (aiming for both old and new).. May god help us both to succeed!! Let me know what happens w you brother
User:Ameen bro. Best of luck to the both of us!
User:Didnt they say 60% is good enough to pass? If im mistaken please help n let me know
User:I’ve heard the same but I see people around this score saying they failed. Not sure if it’s fear mongering or exam day anxiety or a multitude of other factors or perhaps that 70% and above should be the benchmark. Which is why I came here to find out exactly what should I be aiming for and how to improve my scores.
User:People fail with >70% correct NBME’s too tbf. Ultimately we just need to get ~60% of the non-experimental questions correct. Good luck and believe in yourself.
User:my brother those are good scores. Passs on the real deal is 62 ish %
User:First step of improving your scores is recognizing what your weak points are Example: i was getting 90% of questions wrong that required recognizing pictures (histologic slides, gross appearance and radiologic slides) so i focused on those then took the next nbme and my score went up by 10 points
Post 131: Should I push the exam? - in 3 weeks
https://www.reddit.com/r/step1/comments/1klj5ku/should_i_push_the_exam_in_3_weeks/
13/05/2025, 08:08:02
4 comments
I feel like some of the questions I got correct were by instinct and ruling out and that sometimes I didn’t fully understand certain topics. I kinda rushed my prep for this exam because I did step 2 first and already had some understanding of the nbme topics.
Here are my nbme results in order 27(69%), 25(65%), 28(74%), 29 (72%), 30(72%) I still have nbme 31 and the free 120s.
Comments:
User:Mo , you are good
User:you can take it tomorrow
User:do u say that from experience cause I've been delaying this exam for months with high 70 nbmes and still dont feel prepared
User:You’ll never get 100%. Take the exam soon and good luck.
Post 132: Is the real deal like UWSA ?
https://www.reddit.com/r/step1/comments/1klj29w/is_the_real_deal_like_uwsa/
13/05/2025, 08:02:40
14 comments
I find UWSA extremely difficult ..
I wonder if they’re representative of the real exam!!
(Since people have been saying the real deal doesn’t look like nbmes) Please someone tell us
Comments:
User:Not at all lol. Uwsas are INSANELY difficult. I think people are saying that mostly because of the length of the exam. Supposedly it is closer to the free 120 in length (also the bootcamp SA) and in concepts closer to the 30 and 31
User:Omg thank you! I got extremely depressed this morning bc of low scores on uwsa and exam in 3 weeks
User:so you wouldnt recommend doing the uwsa? or should i just think of it as good UWORLD practice?
User:I would think of it as a session that can train you on on how the exam works (timed bla bla) like a self assessment timed… and i would definitely study the explanations.
User:I'm in my last 5 weeks of preparation and have completed NBMEs 20–25. I want to do at least one UWSA1 because I feel it still has some value. Would you say I’d benefit from doing it, in terms of knowledge as well
User:Of knowledge yes. But again, dont shatter your confidence if youre not in the 60s, those questions are terrifying compared to the NBMEs 😅😅 do it and then Review every concept.
User:Bro, skip the UWSA. They are trash. Use NBMEs to predict score.
User:I got 55% on UWSA3 while passing NBMEs easily, not comparable but like someone said Free120 and surprisingly Bootcamp captures a good picture of step as well
User:I felt the real deal was the closest to the **concepts** of **nbmes** and closest to **free120 i**n terms of the **question length.** I did the UWSAs just for practice/ stimulate the test setting
User:FWIW I made a 247 on UWSA1, 243 on UWSA2, and like a 230 on the real deal. It’s worth considering however that I did terrible (<60%) on the two or three NBMEs I did. This was probably just law of averages at work.
User:UWSA are definitely harder but great to find your weak points. even if you find them difficult make sure you do them all. Use NBME's more to gauge your progress and put it all into predict my step score to make sure you're gonna pass.
User:I didn’t find them representative at all and they really discouraged me. I was using UWSA and getting 55’s and thinking I was never going to be ready. When I switched to NBME and free120 my scores shot up. The style of the questions are just very different.
User:I personally don't think that the original exam was similar to any of the practice exams. They have the same concepts of course. But "similar" i wouldn't say.
User:Never did UWSA. I found doing the NBMEs and reviewing them were more beneficial
Post 133: UW + Mehlmans PDFs + FA + (Micro + pharm) - Sketchy - worth the prep?
https://www.reddit.com/r/step1/comments/1kliy1s/uw_mehlmans_pdfs_fa_micro_pharm_sketchy_worth_the/
13/05/2025, 07:56:07
1 comment
Please help me as I am so down to restart the process again?
Comments:
User:How I passed CBSE (NBME Comp) and Step 1 using MM + UW + NBMEs https://youtu.be/3eRen9zBGxY Start here, and adjust accordingly based on your academic needs.
Post 134: What are the most high things to do/revise in the last two days before your step 1 exam?
https://www.reddit.com/r/step1/comments/1kliqpt/what_are_the_most_high_things_to_dorevise_in_the/
13/05/2025, 07:44:01
1 comment
Hi guys, what are the most high yield things? PDF’s? You recommend doing in the last two days before your exam?
Comments:
User:mehlmann HY arrows
Post 135: Geographic location in micro
https://www.reddit.com/r/step1/comments/1kliq1z/geographic_location_in_micro/
13/05/2025, 07:42:50
6 comments
Guys how important is it I know the the states and places where a particular bug affects? I’m not from the US so learning the areas are a pain 😭. Advice please
Comments:
User:There is only 5 bugs where you have to know the location and they are not that hard
User:Really? Which ones if you don’t mind. Because of lot of the bugs have locations mentioned under them
User:Histoplasma capsulatum Ohio & Mississippi River Valleys Coccidioides immitis Southwestern U.S Like LA Blastomyces dermatitidis Eastern & Central U.S. Great Lakes Paracoccidoides: South America
User:Thank you so much
User:Use sketchy it will make it easier, you are welcome
User:Hi! Do you happen to maybe have the anki for sketchy micro?
Post 136: Tomorrow results
https://www.reddit.com/r/step1/comments/1kli9aj/tomorrow_results/
13/05/2025, 07:12:46
43 comments
Anyone getting results tomorrow??
How are you feeling guys????
Comments:
User:I hope we pass
User:How long does it take to get your results? Is it always 2 weeks after your exam or does it depend on the day of the week you wrote your exam? \*wrote mine 7th of may
User:+1
User:Wrote mine 29/04, fingers crossed 🤞🏻
User:+1
User:Same here
User:me too. freakin out
User:tested 27/04 hope we all pass amen to that
User:i passed الحمد لله
User:Wrote mine on May 2nd but my permit hasn't disappeared yet 💀
User:Where do you check your permit disappearing?
User:Where you initially sign up for step
User:Oh well I just realized they changed platforms.
User:looks like its dropping. anyone got the email?
User:US MD tested 4/26, got the email
User:😰😭
User:Taking into account that it even might be 3weeks
User:Do we know what time the results come out?
User:11am EST
User:5/1 tester here; maybe tmo? Maybe next week?
User:I think next week.
User:Tested on 30/4.. full of anxiety
User:Will 30/04 results come out tomorrow or next Wednesday?
User:Let’s see .. seems so .. may we all get the pass .. Ameen
User:How can we check the results , will they always send an email or do we have to check the website ? also took it on 30/04 !
User:Freaking out
User:5/2 score release tomorrow too?
User:i hope so i’m seeing conflicting information between this week and next…i can’t wait another week😭
User:Yeah same idk what to expect.
User:Same can't wait one more week. Tested may 3
User:tested on may 6, but my permit disappeared on monday. does that mean i’ll get it tomorrow?
User:My permit disappeared the next day after my exam. They don't mean anything anymore ig
User:When did ur testing window end?
User:june 30
User:4/26 and am freaking out 🥲
User:Good luck to everyone getting results today. I tested on may 3 (Saturday) will I get it today?
User:prolly next week
User:🥲sed lyf. Thanks tho
User:Did you get the email?
User:No not yet
Post 137: Tutorial vid
https://www.reddit.com/r/step1/comments/1klghr6/tutorial_vid/
13/05/2025, 05:07:33
3 comments
Hey guys, where can I find the 15 minute tutorial video that’s shown before STEP1? Please help out, I want to add that time to my break.
Comments:
User:Online free 120 on their official website has a section for tutorial
User:Hey thanks so much!
User:no problem. all the best for your exam
Post 138: usmle step1
https://www.reddit.com/r/step1/comments/1kleqh4/usmle_step1/
13/05/2025, 03:05:39
2 comments
NBMEs and UWORLD question banks need to upgrade themselves acc to new USMLE step1 Exam changes.
#usmlestep1
Comments:
User:😅
User:What are the changes
Post 139: Struggling with Biochem cycles, 2 wks out
https://www.reddit.com/r/step1/comments/1kleafw/struggling_with_biochem_cycles_2_wks_out/
13/05/2025, 02:37:55
11 comments
I scored pretty bad on NBME 26, but a lot of it was because I’ve completely skipped the metabolism cycles. Can someone please recommend what to do? Please give some HY for biochem
Comments:
User:Dirty Med’s biochem playlist is absolute gold—perfect for a solid first pass through the material. You can reinforce your understanding after with First Aid. Biochem wasn’t heavily tested on my actual exam, but exam content can vary, so it’s better to be prepared than not.
User:funny cause its all over the NBMEs!
User:Thanks!
User:Pixorize has videos on them, that makes them super easy to memorize.
User:Thanks, I’ll check those too
User:For biochem metabolism, you mostly need to know 5 things: 1) substrates 2) end products 3) ATP, NADH, or NADPH dependent steps 4) transporters / change in compartment 5) relevant clinical conditions
User:Mehalman mehalman mehalman …
User:But do you think doing mehlman before nbmes will inflate scores?
User:Nah !! Not always i think that ( mehalman)soldifies ur concepts like his material give direction to concepts application
User:Biochem isn’t heavily tested. I had maybe 2 typical biochem questions. I think it’s better to focus on diseases
User:Yeah that’s what I was thinking but there’s so many questions in NBMEs
Post 140: NBMEs
https://www.reddit.com/r/step1/comments/1kldv7z/nbmes/
13/05/2025, 02:11:08
3 comments
Is NBME 25 still considered an NBME to prioritize or should I just take 26-31 since those are online and the free 120? I have exam in less than a month
Comments:
User:25 is good too
User:I mean if you have limited time, I would take the online ones to simulate the real thing. If you have the time to spare then I’d do the offline ones.
User:Online ones first bc they have repeats from the offline ones and you want to make sure your most accurate performance is on the newer ones
Post 141: Did Anki skew my NBMEs?
https://www.reddit.com/r/step1/comments/1kld2ju/did_anki_skew_my_nbmes/
13/05/2025, 01:22:55
4 comments
Hi. I'm a med student currently in my third year and I'm planning to take Step 1 two months from now.
So the thing is, after doing one full pass of BnB and First Aid, and staying consistent with Anki throughout, I scored 67.5% on NBME 25. A month later, after completing about 30% of UWorld, I got 76% on NBME 26. What I noticed, though, is that a lot of the questions I got right felt like they were because I had memorized certain factoids from Anki.
For context, I used the AnKing deck (~21k cards) but suspended all except the very high-yield tags (~8k). My main goal with Anki was to retain what I was learning from FA and BnB—not to learn everything from Anki itself. I ended up doing about 4k cards over 4–5 months. Once I started UWorld, I shifted to prioritizing my incorrects from the day before, even if they weren’t HY-tagged—but I’ve only been doing that for the past month or two.
So did I really earn those NBME scores, or did I just get them because I happened to do Anki cards that directly targeted the HY questions that show up on NBME exams? Are the real exam questions going to be that similar? I checked the NBME deck and found that around 25–30% of the NBME 25 and 26 tagged cards were unsuspended in my deck, and I’d done more than half of those.
Has anyone else experienced something similar? Is getting 76% on NBME 26 actually reliable in this case? I know I still have two months left, but I’m just a bit worried that my scores might be inflated and I’m not as prepared as I think I am.
Comments:
User:The Anki tags for NBME 26 are low key trash imo, I would say you’re fine, I’ve had NBME questions that mirror exact UWorld questions closely and I wouldn’t say that inflates my score either
User:Thats reassuring to hear. Thank you
User:Yea bro just keep doing your thing, now if you only do the NBME tags then it’s cheating lol but if you’re just learning loads of stuff and they come up randomly that just means you know the infor
User:Are there copy paste screenshots of exam questions in the deck?
Post 142: Should I take another NBME?
https://www.reddit.com/r/step1/comments/1klcief/should_i_take_another_nbme/
13/05/2025, 00:51:19
2 comments
I’m about 3 weeks out and so far have already taken NBME 29 (75%) and 30 (73%) and about 65% of UW done with ~67% correct. I plan to take NBME 31 a week before the test and both new and old Free120 the week of, but I potentially could have a free day this week to take another NBME if I really load some of my days with 160 UW questions instead of my usual 120/day.
I know people always say that NBME > UW, especially if you’re on a time crunch, but I think I am pretty comfortable with my scores as they stand right now; I’m not sure if I will gain much (and having to spend another $65) on another NBME, or if my time will be better used finishing UW at a more ~tolerable~ pace. Or am I going about this wrong and I should focus on taking as many NBMEs as I can at this point and taper down the UW? Any insight is appreciated, thanks!
Comments:
User:You don’t need old 120, do 31 and new free 120 and you’ll be good to take it
User:I'd say slow your roll so you don't get burnt out before the exam. Sounds like you're at a peak and want to have that kind of momentum the week of the exam.
Post 143: When will the results come up ? tested on May 2nd
https://www.reddit.com/r/step1/comments/1klbvr4/when_will_the_results_come_up_tested_on_may_2nd/
13/05/2025, 00:16:55
5 comments
Non-US IMG here. Had the test on Friday, May 2nd. Do you guys think they will announce our results this week or next Wednesday? Also, can you tell me how I can access my score using fsmb?
Thanks! I hope we all get our P’s 🙏
Comments:
User:Same here! Permit disappeared. Not sure if we’ll get it tomorrow or next week
User:2 wednesdays
User:Tested on may 3. I want my results today. Not sure tho. Let me know if you get ur mail today. Non-US IMG too
User:25/4 test takers got their results last week, so I think we may get ours today
Post 144: Sharing my First Aid flashcards
https://www.reddit.com/r/step1/comments/1klbt69/sharing_my_first_aid_flashcards/
13/05/2025, 00:13:08
97 comments
Hey. I create flashcards that cover first aid paragraphs fully. You could let me know on Instagram if you need a specific one! Linking my page here- https://www.instagram.com/usmle.littles?igsh=bGNuYTJtaTIzb2pk
Comments:
User:Very cool\~! Love how you relate everything to the letters of the condition.
User:Thanks🤍
User:Some people are incredibly artistic and make really good notes. I can never.
User:I guess everybody learns differently and we could help eachother out in our own ways :) what's your learning method ?
User:Good luck
User:Thanks :)
User:DM ,I don't have Instagram
User:Check your DM. I've sent them:)
User:Also don’t have insta, care to share w me as well? <3 Edit: totally forgot to say how cool this looks, greedy me wanted them right away 😅
User:Awww thanks!! Check your dm :)
User:how much are you charging, don't ask to DM pls
User:Hey. I don't charge any money. You could let me know which flashcard you need next :)
User:thats a rare sight, and a good sight. i'm currently studying neuro and i'll let you know if i need a particular topic!! thankyou
User:Thanks :) and yess let me know!
User:Please send me also if possible thank you
User:[deleted]
User:Hey. I don't have a deck yet :/ But I could make flashcards if you list out topics (specific diseases etc)!
User:Reproductive pharma and the breast pathologies like Paget disease and rest etc
User:Please dm me 🙏
User:Hii check your dm
User:This is the cutest thing I have seen :)
User:Awwwthankyou🤍
User:Can you please dm me?
User:Hii check your dmss
User:Pls could you dm me?
User:Of course! Check your dm
User:Hey! Would really appreciate it if you sent them by DM here (kept insta inactivated)
User:Hii check your dm
User:Can you also send them to me please 🫶🏽
User:Of course :) Check your dm
User:Would appreciate a DM! TIA
User:Check your dm!
User:where are the flashcards?
User:Reproductive pharma and the breast pathologies like Paget disease and rest etc
User:Noted. Keep an eye out on my insta page for the posts :)
User:Sure
User:tbh its very cutee!! can you make for respi please
User:Thanks. Absolutely <3
User:I really hate to be a bother, but could u DM me. I don’t have instagram :(
User:I understand:) Check your dm
User:Thank you so much!
User:shoot me a dm pls! <3
User:Yes check it :)
User:Hello Please can you DM them to me?Thank you
User:Hey check your dm!
User:Plz dm 🙏
User:Done!
User:I Don't have Instagram. Please DM me
User:Of course! Check your dm
User:Please DM me, thanks 🙏
User:Hii check your dm
User:Please DM me, don’t have instagram. Thank you in advance 🥰☺️
User:Hey check your dm :)
User:Hey! Can you send them to me please?
User:Hey check your dm!
User:Please dm
User:Hi check your dm
User:Dm me as well please 🙏
User:Of course! Check your dm
User:Kindly Dm me 🙏💜
User:On it :) Check your dm
User:F
User:lulz
User:Dm please no Instagram
User:Check your dm :)
User:Please dm me as well🙏
User:Wow these are amazing, and I’m a visual learner! ☺️
User:I hope they help :)
User:Please dm me
User:Hey check your dm
User:I requested you at instagram- hope you will start some kind of anki deck or a youtube channel 😍
User:Thank you so much!! I don't have anki deck yet, but I do run a [youtube](https://youtube.com/@usmle_littless?si=XzCToklLTj7CqqNX) channel :)
User:Would appreciate a DM ! I don’t use instagram! TIA
User:Of course! Check your dm
User:Kindly share it with me too, I don’t have instagram
User:Yes :) Check your dm
User:I've sent you a DM, kindly check please
User:I need thes cards plzzzz
User:Yesss. Pls dm
User:May I know your instagram?
User:https://www.instagram.com/usmle.littles?igsh=bGNuYTJtaTIzb2pk
User:Could you send it to me please?
User:Of course. dm me :)
User:Could you please share with me as well in DM!!!
User:Of course! Pls dm
User:Amazing!!! Thanks alot! Great work! Dm please!!!
User:Hey thanks :) check your dm!!
User:❤️❤️👌
User:These are so cute and fun! I just sent you a DM on insta <3
User:❤️
User:Possible to dm me please? 🙏
User:Hey check your dm
User:Can you please send me too
User:Hey check your dm
User:Join the usmle littles reddit community for updates- [usmle littles](https://www.reddit.com/r/usmlelittles/s/NficL5nhFi)
User:More - https://www.reddit.com/r/step1/s/jUSdSmfr8X
User:Disclaimer: These are original visual study aids based on high-yield medical facts. While inspired by First Aid content, they do not copy any copyrighted text verbatim. The term “First Aid flashcards” refers only to the topics inspired by First Aid, not to any official affiliation or reproduction. “Full paragraph” means -the scope of facts. All illustrations are my own, created for educational use only.
Post 145: What score on the free 120 is considered acceptable or safe? (someone who did not take old free 120)
https://www.reddit.com/r/step1/comments/1klb6tc/what_score_on_the_free_120_is_considered/
12/05/2025, 23:40:01
4 comments
Hi all! Was wondering, what score would you consider to be safe. I have seen 65% and then 70%.
Wanted to hear thoughts, haven't taken it yet...
Comments:
User:There is no safe score, the free 120 is not a scaled exam to tell you your readiness. The free 120 is just 120 questions in the format of step 1 to get you familiar. Your free 120 can be an indicator that you know a lot or could make you say maybe you dont know as much. I had 1 repeat on my actual exam from the free 120 but the nbmes are your gauge of readiness, use those to determine when to take the exam.
User:Thank you!
User:70
User:I'd aim for 75% if you can!
Post 146: Am I ready to take STEP 1? Or Postpone
https://www.reddit.com/r/step1/comments/1klaab7/am_i_ready_to_take_step_1_or_postpone/
12/05/2025, 22:53:38
11 comments
Wanted to get the thoughts of others on if they think I am ready. So nervous to take STEP 1 later this week on Thursday.
* Form 27 – 65%
* Form 30 – 66%
* Form 31 – 69%
* Form 29 – 69%
I know that 69% is above the passing threshold, but I wonder if it is high enough to still take it this week. I am taking the free 120 tomorrow!
Comments:
User:Long time lurker here…. This is v hard and I’m wondering what others are gonna say You’re def range of having a probability of passing but i those scores still allow quite a buffer of knowledge gaps/question technique to get you comfortable with a sure thing I think if your last nbme felt good while deciphering qs and recalling knowledge you can use that as a trajectory point of where you’ll be on test day but if you feel very uncomfortable I would fill whatever knowledge gaps
User:highly agree, if you felt comfortable with your last NBME and the free 120, do it
User:Taking the free 120 tomorrow, what would be the threshold for being comfortable with my free 120 vs. not? I hear its around 65%
User:UPDATE: got a 72% on new free 120 so going to take it!
User:Thank you!
User:Good job, those are solid and consistent scores. I would say get above >65 on the free 120 and you should be good to go. If you have a school advisor, please verify with them first.
User:Thank you!
User:I passed with lower scores than you. Take free 120 and if it’s got a similar trend I’d put $100 on you that you pass.
User:Thank you!!
User:The most important thing is that your scores remain stable (do not fluctuate). This means that you have a good foundation of knowledge.
User:Thank you! Took the free 120 today and got a 72% so I am going to take it!
Post 147: Step 1 in a few days—how do I know if I’m still ready?
https://www.reddit.com/r/step1/comments/1kl9y7p/step_1_in_a_few_dayshow_do_i_know_if_im_still/
12/05/2025, 22:37:15
14 comments
US MD student. I was supposed to take Step 1 on May 7 but pushed it to May 16. I’ve been studying non-stop for over 2 months and scored:
NBME 26: 54%
NBME 27: 57%
NBME 29: 57%
NBME 28: 67%
NBME 30: 66%
NBME 31: 67% (96% pass chance)
I also took Free 120 about 10 days ago and scored 67%
I started rotations last week, and they’ve been exhausting. I haven’t been able to study like I used to, and I’m worried the info isn’t as fresh anymore. I don’t want to walk into the exam and blank. How do I gauge if I’m still ready—especially since I took Free 120 a while ago? I have the day off before my test. I am burning out, Am I overthinking this?
Comments:
User:Go for it, I had to delay into rotations and it was very hard to go up in score. 96% chance of pass is a good sign and your previous nbme was around that range
User:Thank you for that advice!!! It’s tough taking this exam during rotations, what did you do to get your confidence back up?
User:I had huge imposter syndrome starting third year because most of my peers had already taken step but I did not. But I was passing all my shelf exams which gave me the confidence that it’s possible
User:So did you do a week of straight studying for step or did you take a break from rotations to focus?
User:I took a break and focused on the first rotation just to make sure I can pass. Then I studied a little every weekend for step. I took it after 5 rotations.
User:Depending on when u took the most recent NBME and free 120. If it was within a week, just review the weak points and do a quick skim of the weak points from the ones prior. There is also additional older nbmes available if it has been longer than that and you want to see how you would score.
User:Send it. I delayed my exam for a month because one 59% scared me. The other scores during dedicated were 65, 65, and 70. I wish I took it a month earlier. Had to sit through classes meant to prepare us for clerkships. They were 8 hours long for 4 weeks and were preventing me from studying. I started to forget things. By the time I took another NBME 3 weeks later and the Free 120 a week before the actual exam I got 2x68%. Ended up passing.
User:That’s really helpful, thank you so much. I did NBME 24-31 already. Any suggestions of what I can do to see if I still have the knowledge/to prove I’m ready? I can only imagine how stressful being in class for 4 weeks was, I’ve been in rotations 2 weeks and I’m losing my mind haha. This test is consuming bc I don’t want to throw away the last 2 months of hard work.
User:Retake an NBME, your oldest one. Review it thoroughly and analyze why you missed certain questions. Was it your thought process? lack of knowledge? questioning yourself? target those areas and move forward. And give yourself grace that you're doing your best in tough position. Good luck on your exam!
User:Thank you so much. I’ve taken 26-31 online, do you mean retake 26?
User:i also have rotations currently, and i can’t seem to find any time studying. recently nbme 31-67, f120 66, and 2 weeks ago had 57% and 62% on other nbme’s. I can’t afford to delay my exams either. im so scared as to gow my exams will go
User:Ahh it’s exhausting have to balance the two. Did you have a dedicated at all?
User:i did have a dedicated period of study (5 weeks). My first 3 weeks during rotations were super chill, but now its super hectic. I take my ipad and try to study in any downtime i get and listen to divine intervention podcasts when in too tired to read. Crossing my fingers and hopng it is enough
User:good luck!! you are ready!! review your nbmes today and tomorrow and you are golden
Post 148: do i give up?
https://www.reddit.com/r/step1/comments/1kl9n9p/do_i_give_up/
12/05/2025, 22:21:54
2 comments
I am so lost....I graduated Medical School and had a baby (not planned) ive been trying to study for USMLE and itss soo hard. I watch my baby until my husband gets home from work and then i try to study but im just so exhuasted from the day..how do i do it???
Has anyone else found a way to study for the USMLE while having to take care of there baby??
Comments:
User:You have a baby. It is normal to be overwhelmed, especially because of the hormones. I would suggest putting mehlman and dirty medicine videos on the background while you take care of the baby. But you are doing great, and there are other women here in Reddit who have passed the step 1 while taking care of their children. And it is normal to be tired. You are taking care of a baby.
User:stay strong mama! you are very brave to do this while in your med school journey. If you want to continue to be a doctor you might want to strategically plan how you want to do the next few months. Can you get support? from family to watch the baby for few hours in the morning so you can go to library or study? can you put the baby in day care for a few weeks? It is worth investing 5-6K in child care for a few weeks then taking the exam just to be done with it because this can drag on and you might not complete your journey. I understand that it is hard but we just have to find solutions
Post 149: 11 Days Out
https://www.reddit.com/r/step1/comments/1kl5w5d/11_days_out/
12/05/2025, 19:24:08
11 comments
Hi! I just took NBME 29 and got a 55%. I'm 10 days out. I'm planning on taking 30 and 31 this week and Free 120 next week. My exam is next friday. Any tips? I really cannot move it.
Comments:
User:Do nbme 30, if <65% postpone, don't risk
User:do you know what the % to pass is?
User:on the real deal its 62 ish %
User:You need 64 percent +
User:I started nbmes i just got 55 in nbme 26 .. i feel disheartened .. any advices
User:thats not a bad start, we all need to start somewhere. review that test thoroughly and concepts in it, find sketchy or videos that correlate and help the material stick. keep moving forward to the next nbme, it will get better.
User:Thanks 🧡
User:You should postpone though
User:Why can’t you move it?
User:i'm a DO student and I have to take comlex. if I move it i would have to push it to december
User:Take 30 or 31 asap and review it thoroughly, it will help improve the score on the other NBME. Also review NBME 29 thoroughly before taking another NBME. Quality over quantity when it comes to these questions. If you're looking for a tutor, you can DM me. I hope you push through this and see improvements in your scores. Good luck on your exam!
Post 150: Step 1 score report
https://www.reddit.com/r/step1/comments/1kl5r00/step_1_score_report/
12/05/2025, 19:17:58
18 comments
Does anyone know how to find my exact score and how many more points or questions I would’ve needed to pass?”
Comments:
User:As someone who failed in the past by being almost right on the pass line, it's not worth wondering how many questions you were off by (you'll never get an answer). I failed 3 times all almost on the pass line, each time. Brush yourself off, figure out what you need to work on whether it's personal or content and keep going, don't lose hope. - signed by a 4th yr who matched and graduates this Saturday
User:Omg this just brought tears to my eyes! I just passed on my 3rd attempt and there’s always naysayers making you think you won’t make it after a failure. BIG CONGRATULATIONS and thank you for sharing your story!
User:Reading this makes me so happy🥹 I’m about to take my 3rd attempt but tired of all the negative comments around me. I’m so so happy for you
User:I’m so so happy for u! Giving us all hope💗
User:What did you match into if you don’t mind me asking
User:Internal medicine, it was my intended specialty from day one of med school. I received a total of 22 interviews, matched at my #2 spot. Many of the PDs I interviewed with saw resilience and determination in my step story especially since I scored a 250 on step 2, a PD who saw it as a weakness (which was only one) dropped to the bottom of my list. I have my story posted in a different post if anyone is curious. Happy to answer any questions or help anyone in need!!!
User:💜💜💜💪🏿💪🏿💪🏿
User:I love a story of redemption and resilience!
User:Look at a score report from when the Step1 had a numerical score (on google) and then compare the box. You are on the 3rd box from the left, which is a 171-185. And on the 3rd box, you are a bit right of middle, so around 180-182. And then use a NBME score coverter from when step1 was numerically score. For example, the formula for NBME 25 was *score= 277.04 - 1.113(*incorrects*).* For a score of 181, it means you got roughly 86 incorrects or 57% correct. Since NBME says the 60% is pass, it probably means you got a slightly easier form since you were a bit far from the passing line.
User:I believe you scores around 40s according to this graph. 40th percentile. Low pass is 60% and the blue bar I think ranges from 52-62, meanwhile the orange line is around 44 ish. You probably needed to reach the black bar to pass. I’d say around 15-20 questions more correct. Is this your first time or second time taking it? There might be a suspension coming along if it’s your 2nd time fail.
User:You can put the score report on chat gpt and ask about analysis about ur score like how many points u are short off
User:Could you share your NBMEs?
User:NBME 26 60 NBME 27 62 NBME 28 61 NBME 29 63 NBME 30 60 NBME 31 63 Free 120 62
User:Were you anxious on exam day?
User:They don’t give it to you
User:hey did anyone take uswa3 step1? I got 228,im afraid this is my 3rd attemot
User:You failed by a large margin you weren’t ready for the test sorry
User:Accurate
Post 151: Step 1
https://www.reddit.com/r/step1/comments/1kl5fe0/step_1/
12/05/2025, 19:03:51
2 comments
Got 63% on nbme 25 ,exam on 26th 🥲
Comments:
User:you got this
User:Same as my scores exam on 22th we got this do not worry
Post 152: Mehlman
https://www.reddit.com/r/step1/comments/1kl59gq/mehlman/
12/05/2025, 18:56:54
7 comments
I am taking my exam in a month
Which files of mehlman is the most important?
Comments:
User:Arrows,ethics,Risk factors,immuno,neuro
User:Thanks alot Could you give a link to download this files
User:Just search in google .its free. Mehlmann free stuff
User:Thanks I got it
User:I struggled with renal and the immuno is a must imo
User:I only used mehlman arrows and MSK was really lit for the buzzwords
User:I'm doing everything, however, the best are Arrows, Immuno, Renal, Ophtha, Ethics, Neuroanat, Risk Factors, and Endo. If you are short on time, read the question part only.
Post 153: Revision
https://www.reddit.com/r/step1/comments/1kl54kv/revision/
12/05/2025, 18:50:59
2 comments
I want to revise all systems I read in a month and half, any system took how many days? And what is productive way to revise, any schedule?
Comments:
User:Help
User:I
Post 154: How am I doing
https://www.reddit.com/r/step1/comments/1kl51f9/how_am_i_doing/
12/05/2025, 18:47:12
2 comments
Hello IMG here 👋
Form 26 (56%) and form 28 (67%) with nearly 70% Uworld completion at 57% correct. My Uworld has been getting better now. Consistently getting 60% plus on random 40 question blocks untimed. Planning on testing end of June 😬
Comments:
User:IMG here, youre good!!! Start NBMEs now, if you scoring the 60s in uworld, drop it and start NBME content.
Post 155: Hi everyone, I’m the SO of a 2nd year that will be taking Comlex and Step1 late June
https://www.reddit.com/r/step1/comments/1kl4l6w/hi_everyone_im_the_so_of_a_2nd_year_that_will_be/
12/05/2025, 18:28:46
10 comments
I’m trying to help make her life easier and she kinda snapped when I told her she should maybe have at least one rest day a week 😬 I honestly didn’t mean to upset her with that suggestion, but I know she’s tremendously burnt out. She starts her day at 5:30am and doesn’t get home til midnight sometimes.
Is this just how it is? Or does anyone have any advice on how to help her from burning out?
Comments:
User:just be there and support her i don’t have a s/o but im thankful i haven’t snapped on my friends.. yet. i hope i don’t but all you can do is support her, get her favorite coffee, do her laundry , cook her a meal, anything to make her life easier :))
User:Gotcha. Thanks for responding and best of luck to you if you’re studying too 🙏🏽
User:thank you! just started studying so ima be in her boat soon haha, i wish you guys the best of luck and just love her and support her through these tough times, it get stressful and incredibly lonely and the best feeling that we can have is knowing someone is loving and supporting us!
User:Just be supportive bro, my family walked on eggshells when I was in dedicated but I honestly didn’t think I was dramatic at all just panicky. Albeit I agree with you because 5:30am to midnight is intense
User:Yeee. So I’m guessing no one takes a day off?
User:It depends bro, some schools set students up for success so they can chill and some schools teach nothing board relevant so you have to cram during dedicated
User:Fshoo. Thanks for the intel! Best of luck to you if you’re studying too 🙏🏽
User:Nah I did my time lmao but goodluck to yall!
User:You’re doing great just being there for her to vent and be with!!
User:I mean you can just keep the chores done and the food going. Maybe uber eats her something. I will say though that yes it's a hard time but I still make time for my spouse. I'm out of the house from 11am to 7pm and that already feels like too much time away from my SO.
Post 156: Result for step 1 30/4
https://www.reddit.com/r/step1/comments/1kl4ds2/result_for_step_1_304/
12/05/2025, 18:20:30
4 comments
Hi
When will we get the results for 30/4?
Scared.
Comments:
User:Tested 28/4, scared too
User:Wednesday i guess
User:So if your results are this Wednesday, if I took it 5/5, can I expect it next Wednesday?
User:did anyones permit disappear? bc i took it on the 30th and mine is still there
Post 157: First NBME
https://www.reddit.com/r/step1/comments/1kl4b6q/first_nbme/
12/05/2025, 18:17:36
2 comments
Hello, I’m a USMD student who planned to take step early next month. My schools CBSE exam scores plateaued in mid-low 60s (last taken early April). Officially started dedicated in May. Took UWSA1 last week and got a 68%. Today I took my first NBME exam, form 29, and got a 76%. This caught me off guard as I planned an additional month of prep before sitting. This score seems too high and a bit of an outlier? The NBME exam also felt more straightforward than random times Uworld blocks, with less “gotcha” questions. Not sure if that’s just the form I took or if it’s that way on the real deal too. Should I trust this? Should I sit sooner than anticipated? Any advice or comments are appreciated. Cheers.
Comments:
User:take the test
User:Nice job you could def move it up sooner. Take another NBME and shoot for 70+ score again. Then do the Free 120 (get above 65) and take the test. Good luck!
Post 158: Forgot
https://www.reddit.com/r/step1/comments/1kl3fw1/forgot/
12/05/2025, 17:43:10
7 comments
I'm done with my first read, I watched all b&b videos now when I used to do UW as revision , I found that I forgot everything thing. So I started to watch b&b again and I think this is a time wasting, what shall I do?
Comments:
User:i believe is part of the journey by taking each system at least 3x times
User:You could try watching and taking notes?
User:Just do dukes Pathoma, sketchy micro +pharm Anki deck and get back into uworld.
User:Complete your uw incorrects and then do second pass. 3 blocks a day everyday. This has helped me so much
User:You need active learning, not passive. Try meshing practice questions while reviewing. Anki works great as well
User:Been in similar situations before, and what works for me is revising weak systems from the BNB PDF when I get questions wrong on mastered parts before watching it again x2 . Then, I do the same with NBMEs, rewatching guided by the forms. If time allows, do Melhaman. The key point is to not redo BNB.
User:Anki?
Post 159: My exam in 2 days im stressing
https://www.reddit.com/r/step1/comments/1kl165s/my_exam_in_2_days_im_stressing/
12/05/2025, 16:13:46
1 comment
My nbme score so far
nbme25 :58
Nbme 26 :62
Nbme 29:62
Nbme 27:66
Nbme 28:70
Nbme 30:72
Nbme 31 : 71 drop in score
Is it good what should i do right now didn’t take free120 yet
Comments:
User:Ur good take it 31 is hardest and u got great score
Post 160: NEED HELP
https://www.reddit.com/r/step1/comments/1kkyr7z/need_help/
12/05/2025, 14:39:51
3 comments
Guys I really do not know what to do. Just took bootcamp’s self assessment…. Man that was hard. Was always stuck between two answers.
Got a borderline passing score of 59%… my exam is 18 days… cant postpone it. What do I do? I still have nbmes 30, 31 and free and old free 120 left… previous NBME’s were no more than 58%, after studying a lot i redid the 25 and got a 65%.
I am out of words.
Comments:
User:Hey! I would focus on why you got the questions wrong. For example, was it because of content, 50/50s, not having enough time, being confused by what the vignette was asking, etc. This will help you target your weakness. At this point, doing NBMEs and really dissecting the questions will help you the most. How long have you been preparing for?
User:Ive been studying since mid january… tbh tried many routines, could not stick to one, and I think thats mainly my problem. At this point i think ive covered everything and my problem with this self assessment specifically was kinda 50/50 and also lack of time (long stems) I really dont know what to do at this point.
User:I understand. Have you been doing Uworld, Pathoma, etc? What is your current approach to answer questions? The stems on the real exam will be long so it is best to devise a strategy to tackle these longer vignettes.
Post 161: Advice for the actual exam, from people who gave it recently.
https://www.reddit.com/r/step1/comments/1kkybez/advice_for_the_actual_exam_from_people_who_gave/
12/05/2025, 14:22:58
21 comments
I have been hearing a lot of people say the actual exam question stems are super long. Someone once gave a trick to read the last few sentences, and then more often than not, you have your answer (at least for the Uworld questions for some specific systems). I use this, and I think I am usually able to get the answer without having to read the whole paragraph of random information most of the time. Is this good advice or super risky to do for the actual exam? What do you'll think based on your experiences?
Definitely not doing this for psychology, ethics, etc, questions where 1 single word can change the criteria.
Post 162: NBME Question
https://www.reddit.com/r/step1/comments/1kky4m7/nbme_question/
12/05/2025, 14:15:38
2 comments
If I have a question about a specific question on an NBME am I able to post it here?
Comments:
User:you can ask someone if you want
User:and what's better than chatgpt
Post 163: How I Helped IMGs Improve NBME Scores by 15–20 Points in 4 Weeks (Step 1 Strategy)
https://www.reddit.com/r/step1/comments/1kkwue4/how_i_helped_imgs_improve_nbme_scores_by_1520/
12/05/2025, 13:25:27
8 comments
I’ve worked with multiple IMGs who were stuck in the 180–195 range on NBMEs. With a focused strategy, we consistently raised scores by 15–20 points in just one month. Here’s what worked:
\- Active learning, not passive re-reading
\- Daily NBME-style timed blocks with review
\- Custom topic targeting based on weaknesses
\- Strategic use of First Aid + UWorld + Anki
Drop your challenges below—I’m happy to share free tips and strategies that match your study style.
Comments:
User:i hate anki( and feel it takes up way too much time in these last few weeks) but have trouble remembering previous content i reviewed! any suggestions? im abt 20 days out
User:Anki isn’t easy, especially this close to test day. If you're 20 days out, here’s what I usually recommend for memory retention without heavy flashcard use: \- **Daily mixed NBME-style question blocks** — review each one actively and write down key takeaways \- **Sketchy/visual resources** for tough-to-remember subjects (e.g., pharm, micro) \- **End-of-day recall** — take 15 minutes to write down everything you remember from that day’s review Want help customizing a last-20-day plan? Happy to share a quick roadmap!
User:I am stuck at 59.. ive studied for a month and a half after being stuck in that same range (57% on nbme 29) and still unable to higher my score. Im devastated… at this point i dont know how to study, or what to study anymore… if focusing on high yield ? Targeting only whats missing. I dont really know how to use anki at this point…. Im 18 days left with 3 nbmes left
User:Many students hit this wall and feel lost, especially when the score won’t change. At this stage, doing more of the same won’t help. Here’s what I’d suggest: * **Identify weak topics** from your NBME breakdowns and *laser-focus* on those. * Replace passive review with *active recall,* practice daily NBME-style timed blocks, and thoroughly review why each answer is right or wrong. * Skip random Anki, use targeted cards or custom decks that match your gaps. * In 18 days, you still have time to pivot to a tighter, high-yield strategy.
User:Could you suggest me insights ? It really took so long for the preparation. I got 04/15 NBME 29- 47%, 05/13 28- 50% offline. I have prepared one month hoping it will increase, but it didn't. Please guide me, I really need your help to improve to 70 plus. Thank you.
User:It's frustrating when scores don’t rise. Here’s what I’d recommend: * **Stop general studying**—start *targeted remediation*. Your NBME scores show where you're weak. Build a topic list from incorrect answers and focus only on those. * Do **timed question blocks** every day (20–40 Qs) with detailed review. * Use **First Aid + UWorld** like a GPS—cross-reference weak topics and review only what's missing. I can help you build a *custom plan*. You're closer than you think!
User:Thank you. Is timed block for NBME or U WORLD ? I am doing Weak topics
User:Ideally, you should do both. NBME timed blocks (full forms) help simulate the actual exam environment. Use them to measure progress every 1–2 weeks. UWorld timed blocks (20–40 questions daily) are perfect for practice and learning.
Post 164: nbme scores-22 days out!!!
https://www.reddit.com/r/step1/comments/1kkwlqj/nbme_scores22_days_out/
12/05/2025, 13:16:03
12 comments
hi all, started with a 46 percent on my first nbme (26) about a month ago, this was my pre dedicated score and at this point i was so scared, took another nbme 2 weeks ago (27) and got a 50 percent. i was crushed but followed advice and just did 80-100 questions a day. yesterday got a 60 percent on nbme 29!!!
with 3 weeks left i plan on taking an nbme every week, while also taking free 120 in the days before my exam. any tips would be great, is this a good spot to be in? what are some things that boosted your scores in the final weeks and made you feel prepared and ready? should i keep doing questions (about 51% through uworld) or just focus on reviewing?
any advice or encouragement would be helpful :)
Comments:
User:3 weeks is plenty of time to get to where you need to be to pass! Many people will say a general rule of thumb is to get 3 NBMEs >65 or 2 NBMEs >70 so you’re well on pace to get there! I’d truthfully aim to get 70s on the NBMEs and Free 120 to give yourself to wiggle room for test day anxiety. Keep doing what you’re doing and thoroughly review your incorrects! Good luck!
User:thank you!!
User:how do u increase ur nbme??any advice??
User:just did a bunch of practice questions on uworld. used my break down from my previous nbme to identify weak areas and did 2-3 blocks of questions on those topics till i felt more confident. supplemented with mehlman pdfs or dirty med videos if needed
User:and did u review all questions or just the one that was incorrect??
User:F
User:How are you going to say F, knowing you also struggled during the prep phase of step (based on your post history).. the turnaround is crazy. It doesn’t take a lot to be supportive or not comment anything if you’re not gonna be helpful 💀
User:Do you think they were just trying to follow the post?
User:if that’s the case then I’m sorry for coming across as accusatory. I was thinking that is was more in the negative context given the downvotes
User:F
Post 165: test day advice?? freaking out
https://www.reddit.com/r/step1/comments/1kkwf9d/test_day_advice_freaking_out/
12/05/2025, 13:09:02
6 comments
hello all pls get ready for a tmi but i need help from my fellow period havers. i test on saturday and i have pcos so my period *might* be on that day (obviously can’t be sure). my question is: did this happen to anyone else? how did you deal with it? my best idea is to pack some NSAIDs but does anyone have any other ideas.
for example, it would majorly help to have a heating pad but idk if that’s allowed but maybe in my locker? or anything else that might help even a little. i’m aiming for maximum comfort. fml. ty in advance
Comments:
User:there are sticky heating pads that you could wear! [https://www.target.com/p/thermacare-menstrual-heatwrap-4ct/-/A-89581952?sid=2868S&TCID=PDS-1738608672&gclsrc=aw.ds&gad_source=1&gad_campaignid=1738608672&gbraid=0AAAAAD-5dfY_V6EamU83_xkqUvyJLGPdX&gclid=EAIaIQobChMIkoboiM2ejQMVzEp_AB1iOgbHEAQYBCABEgIEBPD_BwE](https://www.target.com/p/thermacare-menstrual-heatwrap-4ct/-/A-89581952?sid=2868S&TCID=PDS-1738608672&gclsrc=aw.ds&gad_source=1&gad_campaignid=1738608672&gbraid=0AAAAAD-5dfY_V6EamU83_xkqUvyJLGPdX&gclid=EAIaIQobChMIkoboiM2ejQMVzEp_AB1iOgbHEAQYBCABEgIEBPD_BwE)
User:tysm i really hope they let me bring them in the room😭
User:they allowed me to bring my cushion for back pain when sitting in the chair so they most likely will allow you too?
User:this is good to know ty!
User:Take progesterone or oral combined contraceptive to postpone your period
User:real but also i can’t do that for literally 1 week bc i don’t wanna be on OPC in general. this is like a 1 time issue for me lol
Post 166: Uwsa
https://www.reddit.com/r/step1/comments/1kkw3b6/uwsa/
12/05/2025, 12:56:22
2 comments
Are uwsa imp for step 1
I gave uwsa1 scored 53% . Should i worry??
Comments:
User:when is scheduled your test?
Post 167: don’t listen to all the fear mongering on here
https://www.reddit.com/r/step1/comments/1kkuymt/dont_listen_to_all_the_fear_mongering_on_here/
12/05/2025, 12:10:50
19 comments
Took my test May 9. Overall I felt like it was hard but fair. Is it harder than your schools tests & all the released forms & the free 120? No shit! It’s the real test obviously it’s gonna be.
The classic stuff was very easy, the moderately hard stuff was very approachable, and the hard stuff was very vague. Simple as that. Lot’s of long question stems and H&P’s.
I used Boardvitals, school provided materials, Sketchy micro, and lots of free youtube stuff. Also got through ~90% of the Anking deck. Attached are my practice test scores for reference.
No need to freak out or listen to posts from this sub about people who weren’t prepared to take it and then got banged by it. Study hard & know your shit and you’ll be fine.
Comments:
User:I had a bad test experience. Felt pretty badly while taking the test (I passed). I want to make it clear that people like me are not unprepared. I was fully prepared with scores in the high 70s and 80s. 82 on new free 120. Congrats on having a good experience but I’m not a bad student, I’m not a fear mongerer and I was not unprepared. We just had different experiences for various reasons. Everyone should believe they can pass but no one should take any of these posts as THEIR reality. Read different experiences. Maintain your confidence and prepare yourself for a possibly very tough day.
User:Not everyone has the same experience even they are well prepared. In fact, those kinds of rant posts often help relieve the people struggling with post exam anxiety. I know you are trying to make people comfortable and feel confident with their preparation but speaking dismissively from a limited perspective doesn’t seem very considerate either. Actually i had better scores than you on practice exam and severely struggled with the anxiety until i got the p
User:I felt really discouraged by posts like this after my exam, so I just want to say to anyone struggling with anxiety rn your feelings are valid. Don’t let this post make you feel terrible and remember there are bunch of people got p after the worst experience including me.
User:Exactly. My stats were better than the ones posted here and I \*still\* felt like shit coming out of the exam. People feel how they feel. It's not lack of preparation that makes people feel like the exam was super hard and did not reflect the NBME prep materials.
User:Give us your results when they are out.Hopefully you pass cheers!
User:Hey CHM friend haha
User:whats up!!
User:CHM 🗿
User:🗿
User:Preach brother
User:Did you feel any topic was over emphasized? Like Micro or Neuro or was it well balanced
User:felt pretty balanced. know your ethics/best responses like the back of your hand!
User:What’s a good way to “know these” I find myself just going off vibes
User:lol that’s what I’ve always done and I seem to get majority of them right. I’m sure there’s a better way to study them
User:"going off vibes" sounds like how I answer every question on UW
User:What is H&Ps? and what was the most tested topic And the hard Qs was most or the easy direct one ?
User:What are the HMs?
User:human medicine. just school provided CNSEs or CBSAs
User:It’s just the course code for our school. Changes each semester.
Post 168: Running out of NBMEs?
https://www.reddit.com/r/step1/comments/1kksz9k/running_out_of_nbmes/
12/05/2025, 10:48:54
3 comments
After taking NBMEs 28, 29, 30, 31, and the Free120 I chose to delay step by a few more weeks. I only have NBMEs 26, and 27 left to gauge my progress.
I've been reading online that the earlier NBMEs, especially 26, aren't as representative of the exam. If I do well enough on the 27 and 28, should I be confident enough to take Step?
Post 169: 12/5 takers
https://www.reddit.com/r/step1/comments/1kks5e9/125_takers/
12/05/2025, 10:11:21
10 comments
My hopes got demolished just after finishing the first block. Every block had 5 WTF questions. Five ridiculously long questions that you had to scroll down. I just hope I can somehow pass.
nbmes score fa ged 70-78%.
New free 120 69%
Comments:
User:I took form 31 and had an EPC of 89. I still walked away from this exam humbled. It was a hard test. Many third order questions. I think it’s normal to feel this way. I’m frantically browsing this subreddit and this is the trend. No matter how good u feel going in, u r likely to feel disheartened because all u can remember are the hard questions. The reality is, we probably got more questions correct but we don’t think about it. Now we’re stuck with our own thoughts until they grade our test…brutal.
User:Had similar NBMEs and same free 120. Thought I completely failed bc very few questions felt like slam dunk but passed. In retrospect, no one I’ve talked to has felt great after the exam. This goes out to OP and all the neurotic lurkers looking for a reassurance: try to relax and enjoy life and forget about the exam for a second.
User:Hey how was the length of the questions everyone’s saying it’s long and the answers were vague
User:It’s not like the NBME that’s for sure. At least this was my experience
User:I tested on 11/5, exam was very tough.
User:By long how many sentences do you mean? Aprox
User:Talking a whole page. Like a whole patient chart. But, you shouldn't read the whole page. Read the question at the end and then reverse engineer an answer.
User:Thank you! Hoping you get that P !!
User:I took the exam on 13/5. It was very hard.
Post 170: Low NBME Scores What Should I do?
https://www.reddit.com/r/step1/comments/1kkoyaa/low_nbme_scores_what_should_i_do/
12/05/2025, 07:16:58
7 comments
So I'm a non us-img
I have been preparing for this exam for almost 1 year
I have had a weak base while I was a student.
I have done FA 2 times and 1 pass of uworld with an average % of 60.
Still have around 950 uworld incorrects.
My triad is going to end after june
I seriously want to improve my scores.I need to get 75% plus.
Don't want to waste anymore NBMEs until I improve.
HERE ARE MY NBMEs
NBME 25 18/4/2025 60.5 %
NBME 26 26/4/2025 66.5 %
NBME 27 2/5/2025 63.5 %
NBME 28 11/5/2025 63.5 %
Currently I review around 50 nbme qs per day and a block of 50 uworld incorrects a day.
Should I begin from nbme 20?
I feel my anatomy,micro and biochem is weak and I also have a weak perception of the questions in nbmes I get stuck in two options and end up choosing the wrong one.
Should I expecf myself to improve by mid june and will I be ready by mid of june?
I dont't want myself to burnout.
Comments:
User:Start doing anki. Get the anking deck for $5. There’s an add on that lets u import cards related to uworld blocks. There are also tags separating cards by NBME. You are not retaining information
User:Can you give link of that anki
User:Chrome step toolbox
User:Increase your questions per day and work on weak areas.100 questions minimum since it is sec pass. Check your pattern of mistakes
User:Try the Janki deck on anki. It helped me improve my scores
Post 171: Micro
https://www.reddit.com/r/step1/comments/1kkows2/micro/
12/05/2025, 07:14:17
7 comments
How to do micro? Where to start, I’m very confused. Can someone please help me
Comments:
User:Sketchy is the best. I mean it is absolutely undisputed when it comes to micro BnB +First aid is a good alternative
User:Thanks 👍🏻
User:I hope this helps you…. I was💩 at micro…. Started one day randomly two years ago with BnB because I was sick of being 💩as an intern rotating in IM…. Watched every single video…studied it really hard! If I encountered something I didn’t know that was mentioned in the videos I took a detour and studied it and returned to where I was! Took me about 5weeks to finish the whole thing (the bacteria and viruses)….. thankfully for me I had also found a video on YouTube about all the antibiotics( https://youtu.be/rw16kJvEx5A)….then for the parasites there was a YouTube channel called RWJF micro and I had done the parasites sections on it in med school so I just reviewed my notes from this channel!!! Then I did all the divine intervention podcasts on micro along the way! Like if I did gram negative rods I’d play his podcast part on gram negative rods even when I was at the gym…it was so freaking awesome! Someone then recommended I try sketchy but at this point I had studied everything so I literally knew almost everything so I basically watched most of it on 2x speed! By the end of my rotation I was casually holding myself in conversation with the infectious disease consultant! I literally went from 0-95! Studying for USMLE now….i find the micro questions easy……BnB Changed my life! If I can do it so can you! Just dedicate your time start from 0 and within 4-6 weeks you’ll be there!
User:Thank youu so muchh for the detailed guidance!! Means a lot. 🫶🫶
User:I'd also like to add in Mehlman on youtube. He has 4 videos that are broken down by Gram +/- , rod/cocci and some atypicals that are more lecture style. It's pretty much targeted at just HY Step relevant micro. Will take about 2 hours to get through all of them, but it's nice review with great clinical pearls that add to what you learn from sketchy. Good luck you got this!
User:Thank you 😊
User:Sketchy
Post 172: USMLE Ethics: Order of Operations
https://www.reddit.com/r/step1/comments/1kkobxu/usmle_ethics_order_of_operations/
12/05/2025, 06:35:47
3 comments
No content found.
Comments:
User:Wow! Thank you very much!
User:omg thanks for information
User:thank u!!
Post 173: What’s going on with Recent test takers experience!!
https://www.reddit.com/r/step1/comments/1kkjm31/whats_going_on_with_recent_test_takers_experience/
12/05/2025, 01:12:28
32 comments
I’ve observed recently and many might agree this subreddit is flooded with people ranting how NBMEs are not representative at all and I kinda get it. Experiences may differ and people who only want to rant out tend to be majority who are posting here(would love to see people who had different experience to this too so please do post too it’d help a tone of us ).BUT! what i and many others don’t understand is why are people recommending NBMEs still then! i mean the same people who say NBMEs are nothing like the real “thing “ would still have you get online NBMEs over offline and ignore UWSA or other assessments out there. So please someone make it make sense all this. It’s literally confusing . Here are some of the questions that many of us have in this confusing NBME situation :
1- Are NBMEs not representative because of content wise or just length of Questions or both Or nothing at all ???
2-would you still recommend Them because they are worth it or just because nothing else out there is better?
3- for those who think otherwise. is there any other better self assessment out there that represents better in content wise and real deal stamina preparation?
Please share your thoughts and suggestions everyone. It’d be helpful to many many many of us.
Comments:
User:I took three CBSEs and all the online NBMEs, but the real exam didn’t feel like any of those in terms of style. The Free 120 was the closest in format, but the actual test had longer passages and more H&P-style questions than the Free 120. That said, I thought the exam was difficult but doable. I've had a lot of exposure to USMLE material, and many of the same concepts were tested on the real thing. Personally, there were maybe two questions where I had absolutely no idea what they were asking or the answer choices were completely unfamiliar. For the rest of the exam, I at least understood what was being tested—even if I forgot the correct answer or second-guessed myself and changed it to the wrong one, lol.
User:Alright thank you so much for your input. So what i understand from this is “NBMEs are still great in testing the concepts needed to master the USMLE and give you a great exposure but not the perfect assessment yet. Got it. Will see what others have to say
User:Yep, that’s a great summary of what I was saying! Totally agree. I really think the NBMEs should include longer stems to match the style of the actual exam, since that’s what tends to catch people off guard and mess with their timing. A lot of the info in those long stems is just fluff, but it can still feel overwhelming. It can take practice for some to get good at filtering out the noise and spotting what actually matters.
User:This post is literally all the doubts in my mind. I hope recent test takers share their perspective.
User:People suggest NBME just bcz it tells you if ur ready for the real thing or not. It's not at all representative of the real test. In there you will see questions that you know of but still you wont be able to find a suitable answer for and you have to somehow click the best one taking help from the clues given in question stem even if it doesn't makes sense to you. What my take is NBME and free 120 just tells you if ur ready or not they don't at all indicate what the test is going to be like which is a very common misconception that i had too
User:Totally agree. I’ve seen so many people INSIST they are similar and call people fear mongerers which is a term I wish people would use less on this sub bc it’s invalidating and unkind. A LOT of people feel the test wasn’t like the NBMEs or Free 120. I was shocked by my exam as it was nothing like any of the practice tests I’ve done but I stayed cool and trusted that despite this, they had prepared me to pass and I did. Took it 4/14. Also, no one is going to say “don’t do the NBMEs” bc that’s insane. You have to do the prep offered by the USMLE. But I did feel that there is no practice test that I took that really represents what the real thing was like. After taking the exam, I would not recommend anyone sit for it with less than 70s on several NBMEs - but that’s just me. My school requires a 62 on CBSE before sitting for the exam and our pass rate was like 96% compared to the national US MD average for 2024 which was 89%. So I’d say we can still “trust the process” but yeah, I felt pretty shook even after scoring in the 80s on the free 120s. Almost all of us can still pass, but I think that people should just be aware going in that it might be a little shocking bc it may not be like any practice test you took before, but the key is to stay calm and do your best. Do not freak out. You are probably ready and you will probably pass despite this.
User:Thanks, you actually made a really good point, you have to trust yourself and your preparation.My last three NBMEs (28, 29, and 30) were all 80%, and on the actual test, I just had to use everything I’d learned through UWorld, NBME, and Free 120. You won’t know everything, but if you keep your nerves in check and trust your gut, you’ll make it through. You dont have to chase that attractive score on uworld in every block but actually learn how are you going to deal with questions you dont know of, time and focus
User:I see now and I really do trust people’s experience since many people almost everyone is agreeing with the same thing nowadays Thank you for sharing this experience we need to be prepared for that. Good luck to us.
User:Almost everyone knows FA in and out, did uworld and every topic there is out there but what really matters is that can you actually use this knowledge when it comes to the actual test. It will be different, but it wont be something you dont know. Learn to apply your knowledge and how to solve your way to a suitable answer. The common mistake we make is we do uworld untimed and tutor mode, and are so hooked on seeing our answers right away. I would always suggest people to do it timed and test mode so you can get a feel of the test and actually think on a question you dont know rather than just clicking at anything to see whats the correct answer
User:I think the length of the questions are not represented on the nbme but the HY concepts that are tested are similar. The real deal is a mix of uworld style questions, nbmes and free120(in terms of the length. There are so many more questions and 80 are experimental, thosw questions are really weird and really overwhelm you. After the exam you feel like shit and that is why I think people say that.
User:Understandable. I mean people should atleast talk about if the distribution was fair and not all lengthy tho. Because from what i get whenever i read those rants people make it as if the whole or atleast 90% of the form was so lengthy which makes alot of us nervous.
User:IMO, almost all of the stems were longer than the typical Uworld or NBME length. There were stems that spanned an entire computer screen length bc they’re set up like progress notes and it’s a TON of info. The test was a BEAST. I did manage to have at least 5 mins remaining after finishing each block but I focused really hard on maintaining my pace and I watched that clock the whole time. You will have adrenaline on test day. You can finish in time despite the extra length. You just have to know when to move on from a tough question and when to be confident enough to quickly pick an answer without rereading or second guessing.
User:Wow that sounds rough. We need to be ready for that. Thank you for sharing this. means a lot.
User:I passed. You can too!
User:Does anyone know if there has been a pool change since May? Cze, everyone's pretty much saying it's different and difficult since then.
User:No one can know that i guess . I hope someone can enlighten us about this if that’s the case though.
User:Hey, I tested yesterday (5/12), and I have to say that the exam is tough but doable. I did NBMEs 20-31 (skipping 23, 24) + new free120 + UWSA3 before going into the exam. Here are a few reasons I believe people find the real exam "completely different": • **Time management** - the question stems are LONG. Much much longer than any I faced while doing NBMEs or UW for that matter. I think the closest idea you can get is that "patient file" type of questions on the new free120, which are almost 1.5 pages long. If you have not tested yourself repeatedly with full-length tests (e.g., NBME + 2 UW blocks), exam WILL feel different. • **Breaks** : You'd be surprised how fast your time goes when you're taking a break. The whole process of checking and rechecking as you enter and exit the center takes a couple of minutes, which you should be mindful of. Every "5 min" break ate ~8 minutes. You need to have that safe margin in your mind if you're someone who has to stretch after every block like me. • **Questions** - they were vague and ethics-heavy. However, the concepts and general idea are not something completely unseen. NBME concepts were repeatedly tested, and I would go as far as to say at least 6-8 questions were completely repeated. • Trust your **NBME scores** - can not emphasize this enough. Whoever says NBMEs are irrelevant to the new exams probably has not done enough or reviewed deeply. Hysteria of spreading "new question bank" is not completely misplaced but overhyped. Lastly, I just want to wish you the best of luck! If you have any further questions, feel free to DM me. If you want to learn more about the real test and my journey, do let me know, too. I would love to make a separate post on this subreddit. Good luck!
User:Testing in a month Scoring 60-65 on NBMEs 28,30,31 pending Help me out!
User:Hey, getting mid-60s on the newest NBMEs, a month out from the real exam, is a pretty solid score. However, I would highly encourage you to first thoroughly review the one you attempted before proceeding to the new one. Also, it is suggested to reserve your newer ones towards the end as they are generally more predictive and boost your confidence. A lot of concepts are repeated throughout the forms. With that said, I think for now, your priority should be reviewing the incorrects + supplementing the concepts with FirstAid. Make sure to review your flagged ones as well. Complete your UWorld if you haven't before proceeding with the other forms. Also, try to go through earlier ones (20-25) as you are a month out from real deal before attempting newer ones (26-27). Good luck!
User:Thank you so much @tahabid05 I appreciate your suggestion and detailed insights! Would it be fine for you if i Dm you sometime in the future for advice and guidance?
User:Ofcourse! Here to help in any way I can
User:Hey thank you so much for sharing your experience i pray you get the PASS iA. and it’s been eye opening reading all this as i plan on taking the real deal beginning of June I’m currently doing the NBMEs similar to you i started with the old NBME20 done all the way to 28 which I’ve just finished today as i type this. And scored 80.5% raw. But all OFFLINE tho. i find them very straightforward with similar concepts repeatedly coming and even some exact questions repeatedly coming back and definitely easier than UW and i wish the real deal to be similar lol. i plan on doing the rest online i have couple questions regarding those long stems and ethics will dm you. Thank you
User:Thank you for your kind wishes! Also, you are absolutely crushing this prep. Keep up the great work! And best of luck
User:Took the exam last week, I’m from Europe so I didn’t have anyone in my uni or even have a contact with someone studying for step. I’ve been preparing for over a year alone during classes and went through this whole study period hearing every fucking day from this group that it’s nothing like NBME and free120 which is the biggest fucking bullshit ever. After my exam I wrote down all the questions I remember I had over 160 and literally 158 of them are from NBME concepts. People that say otherwise is just an excuse to potentially excuse their potential “fail” if they do “oh my god mom dad the exam was not like the practice exams”. The other people from my testing center said the same thing as me and the exam was very doable. If you worked hard and gave your fucking all then you will know the answers and that’s it.
User:Oh wow this tells a different experience and as much as i agree with everyone i also take everything in this sub with a grain of salt. For instance people here mostly disagree with what NBME is hard and you can clearly see a big difference on opinions on different NBME. For instance if you search any NBME you will see people who always say it’s the hardest lol and vice versa. so yeah I understand your point and thank you for sharing your experience it’s encouraging as someone who’s studying alone and doing all this by myself on top of doing residency and this sub has been the only place i see people’s experiences as i have no one to ask for real exam experience around me. Thanks buddy.
User:F
User:You know if you sit UWORLD self assignment after multiple NBME and we ask you about your opinion, is it similar? You probably gonna say no but guess what if you have done well on NBME you probably gonna end doing well on UWORLD self assignment or Amboss or FREE 120, although each one of these has its unique structure.
User:I understand your point and i agree with you on this. However unlike uWorld or AMBOSS the NBMEs are said to be designed by the same people who write the real deal or (whatever idk where i get that lol). Which is why people do these tests plus previously on numerical STEP1 people used to even predict scores let alone just PASS with the NBMEs from what I understand but nowadays people here make it totally different so that’s why the confusion comes. Anyways seems like there has been a big change and NBMEs need to upgrade their forms to atleast prepare us for what’s coming.
User:It's true that contents wise NBME is the best but that doesn't mean it prepares you 100% despite how much you study when you sit you will find it hard but 99% you will pass ( assuming you score well in NBMEs ) , as I told in the post I wrote 90% of people who studied well and reading reddit posts will end up feeling the exam is kind of fair and easier than they was expecting but they will understand people who rants why they ranting and they may end up ranting themselves 😂
User:Makes sense!
User:Gave the exam on may 8th can tell you it’s nothing like the nbme mine was more like the uwsas
Post 174: I want to do step 1 at the mid of the june
https://www.reddit.com/r/step1/comments/1kkjbve/i_want_to_do_step_1_at_the_mid_of_the_june/
12/05/2025, 00:56:40
1 comment
Kindly guide me about how to compile things i just completed uworld nd going through wrongs and reading first aid
Kindly tell me about what should i do .??
Post 175: My algorithm to pass.
https://www.reddit.com/r/step1/comments/1kkj63k/my_algorithm_to_pass/
12/05/2025, 00:47:25
21 comments
Any NBME above 70%
Free 120 above 65%
Uworld 45-50% with above 50% correct
Use resources that work for you.
If you meet these benchmarks. I recommend taking the exam. Don’t wait to “feel” ready. Everyone I know who passed, including myself, didn’t feel 100% ready.
Good luck everyone!
Post 176: Amboss
https://www.reddit.com/r/step1/comments/1kkhyfd/amboss/
11/05/2025, 23:39:09
6 comments
Has anyone used Amboss for practice questions? What are your thoughts? I’ve also heard of USMLERx?
Post 177: Score UW 33% complete 65%
https://www.reddit.com/r/step1/comments/1kkgn5g/score_uw_33_complete_65/
11/05/2025, 22:27:42
4 comments
NB: I study FA after complete the system in UW
Post 178: My exam experience
https://www.reddit.com/r/step1/comments/1kkg56a/my_exam_experience/
11/05/2025, 22:00:49
37 comments
I've been active on this sub since I began studying for Step 1 (you can find my early posts asking how to get more than 30% on UWorld Lmao). A few days ago, I finally took the exam.
I'll write this more like a journal than anything; this is my personal experience.
# The week before the Exam
I felt like I had completely forgotten everything and was definitely not ready. I went through tons of Mehlman questions and tried memorizing drugs, but everything I recalled felt wrong. It was incredibly discouraging. Still, cramming has always been my way through med school exams, so my brain was used to this last-minute pressure. So I did just that, I crammed hard again, and the day before the test ended up being one of the most intense study days I've had. It was worth it for me.
# Night before the Exam
I couldn't sleep properly. My Airbnb had incredibly loud flooring, and the upstairs neighbor inexplicably walked around for 4 hours straight (from 11 PM to 3 AM). Incredibly, I managed about 4.5 to 5 hours of sleep, which is the only reason why I may have a shot at passing this exam. This is also my biggest advice: GET SLEEP! srsly, 8h exam, your brain needs energy.
# Morning of the Exam
I woke up energized from adrenaline, but couldn't eat much. My girlfriend made oats (as recommended by Dirty Medicine) and coffee, but I could only manage a few sips. My appetite was completely gone, which is unusual because I normally eat a lot.
# Arrival and Check-In
I arrived at the testing center on time, though check-in took an additional 30 minutes. I'm a social person, so I started chatting with people, trying to help everyone feel a bit more relaxed (including myself). We joked around, and it was pretty nice. Before starting, I also had a brief episode of diarrhea, likely stress-related, as it had been happening for the past 2-3 days. (very unusual for me, but because I studied for usmle, I understand this can happen)
# Exam Experience
* **First Block:** Felt surprisingly manageable, though the questions were very long, as many have mentioned. I felt prepared.
* **Second Block:** Significantly harder. I started second-guessing many of my answers.
* **Third Block:** I began feeling dizzy and thought I might faint. During the break, I saw one of the nerds from earlier hunched in a corner, quickly eating his protein bar with both hands like a rat. He looked like the smartest guy in the world, so I immediately copied him—grabbed my own protein bar, rushed next to him in the corner, and ate quickly. It completely resolved my dizziness, probably a hypoglycemic episode or smt. - I still have the cute-drammatic, war picture of me and him eating in the corner like little rats.
* **Fourth Block:** Ethics questions threw me off completely. Unlike practice questions, the "correct" answers seemed counterintuitive. They were forcing me not to choose the answers I learned in my training (you always saw this q, everywhere, you always chose A, but this time, A sound so much worse than D. I ultimately went with instinct rather than textbook responses, which turned out to be a mistake according to ChatGPT.
* **Fifth Block:** Dizziness returned, but water and another break helped again.
* **Sixth Block:** Only had four minutes left of my break. The test-center attendant (bless her heart, prob saved my exam) advised me against going to the bathroom to avoid risking an unauthorized break. -Another guy next to me was in the same situation, but somehow, he forgot to press the continue test button, even tho he was at the computer :/
* **Seventh Block:** Done. all that work, everything I sacrificed for this exam... it's all over, my hands are clean now from every responsibility.
# Post-Exam Feelings
Immediately after the exam, I felt incredible—I felt so free and nice, I went home smiling, it was raining, but the cold rain falling on my face felt so good. I've never done drugs, but this is how they must feel like. I was super happy and super energetic. Weird after 8h exam, right?
The exam was tough. Questions were super long, and I consistently finished each block with only 10-20 seconds left—no time at all to recheck anything. But still, it was about medicine, stuff I've actually studied, not random questions from Tarzan's jungle.
Another thing that I subevaluated was the break time, I wanted to call my sister in one of the breaks, but she didn't answer, and I am glad she didn't bcs It would have killed my time - I took about 8 mins at the start to write biostat formula (I aced biostat, I am sure I got 100% correct, but had very few biostat q:( )
If I had to prepare again, honestly, I don't know what I'd do differently. If I fail, I probably deserve it; the exam was hard but fair, not impossible. But as more time passes, it's starting to hurt more. Now I understand the post exam "I will fail" - I do think that I am going to fail, a lot of other ppl with better nbme failed.
I felt good at the very end of the exam, but with time, I started remembering more and more q I got wrong, especially the easy ones, and it's consuming me. I legit think you could go crazy from this (I could go crazy rn 😂).
I keep remembering my mistakes, especially that particular easy ethics question. It's haunting me, I keep seeing it every time I try to sleep or even when I just close my eyes. That one easy question I should've gotten right keeps coming back. - I hope writing this post will help me somehow
Previously, I always thought long questions were easier because they gave more information, helping narrow down answers. But this exam was different—extra info was just noise, completely useless and not buzzworthy at all.
One thing I felt extremely lucky about was that the topics I struggled with the most ended up being tested in very basic ways. Weirdly enough, the areas where I felt most confident had the toughest questions, loaded with countless tricky traps. Of course, it's totally possible that I just overthought everything and got those answers wrong, or maybe I’m actually too dumb to even understand the questions 😂. Still, some questions felt genuinely difficult and unusual, yet I felt really proud figuring them out—like initially it seemed obviously answer A, then spotting a hidden trap made me consider B, but finally seeing another trick clarified it was definitely answer C.
\-BTW, my gut feeling always sucks—whenever I'm stuck 50/50 between two answers, I usually pick the wrong one. So, during the exam, I just opened the calculator, randomly multiplied two numbers, and let fate decide: if the result was even, I chose the second answer; if it was odd, I chose the first. Sounds silly, but hey, desperate times call for desperate measures.
Also, there were some incredibly easy questions scattered randomly. They threw me off because I'd waste extra time re-reading them, confused by their simplicity in the middle of all these monstrous questions.
Another essential tip is to bring a water bottle! With all that adrenaline pumping, your mouth will become incredibly dry. By my last block, I had no break left, so I couldn't drink much water because I would need to use the bathroom after, and my mouth got painfully dry, affecting my focus. I ended up just holding water in my mouth, without swallowing. I bet I was looking like a squirrel
# General Exam Impressions
* **Question Length:** Extremely long questions with lots of unnecessary information. Unlike practice exams, the extra information wasn't particularly helpful in choosing answers.
* **Question Difficulty Breakdown:**
* 20% felt completely certain (100% sure)
* 50% reasonably sure (60-70% confident is A, but couldn't really rule out B )
* 30% uncertain, stuck between two equally plausible options
* Only 1 question was entirely incomprehensible (legit, the answers were: a)bfiwvbbb2323232 b)coabssuobuwbndo223242 so I just laughed, chosed C, and moved on)
# Practice Exam Performance
Time- IDK 😂 between 9 and 12 months (total forest time +-900h - yeah, I had 0 discipline at the start, so I was skipping days, about 350h in the last 2 months - last month I did 6-8h/day and the rest I would play video games or smt)
I did NBME forms 20-31 and both old and new Free 120:
* Highest scores: Old Free 120 – 77%, NBME 25 – 71%
* Recent scores: NBME 31 – 66%, Free 120 (new) – 67%
With all that said, I am proud of myself. I've never put so much work into anything in my life, and knowing the dedication and hard work I invested makes me feel accomplished. Even if I fail, this exam has gifted me discipline and made me a better doctor. It made me feel like I deserve to be a doctor.
I know it might seem like I'm treating this exam lightly, but I really tried. To give some context, if I fail, it would confirm that staying in the EU (where I'm about to graduate) isn't the end of the world, as things look pretty good here too. I get that many others are in a much tougher spot, and it might come across as insensitive if I seem carefree. Honestly, I truly want to pass, and the past few months have been incredibly stressful for me. I can't even imagine how challenging it must be for someone facing even greater pressure.
Good luck to everyone preparing. Prioritize sleep, manage your energy and glucose levels during the exam, get water with you, and trust your preparation.
Post 179: Anyone with good test experience recent test taker ¿
https://www.reddit.com/r/step1/comments/1kkg208/anyone_with_good_test_experience_recent_test_taker/
11/05/2025, 21:56:00
1 comment
n
Post 180: STEP 1 stats
https://www.reddit.com/r/step1/comments/1kkfchk/step_1_stats/
11/05/2025, 21:18:16
3 comments
Hi! Just found out I passed STEP 1 and wanted to share my scores since Reddit was so helpful to me when I was debating to take it! Consistency is key, and listen to your instinct! You got this!
week1-52: form 26
week2-54: form27
week3-57: form28
week4-63: form 29
week5-63: form 30
week6-64: form 31
week7-68: old nbme form 24. Also took nmbe 25 and old free 120 not scored as well
week8-65: new free 120 6 days before test
Took test and Passed!:) Good luck guys!
Post 181: I need help, feeling desperate and not scoring well on self assessments
https://www.reddit.com/r/step1/comments/1kkdee7/i_need_help_feeling_desperate_and_not_scoring/
11/05/2025, 19:39:21
18 comments
Hi, I'm an IMG, graduated last year and been studying for the USMLE step 1 since I graduated.
My school curriculum is very different from the Step 1 material, so I had to do First Aid with BNB just to learn what am I supposed to know for the exam. I used sketchy for microbiology, didn't do much anki, and finished 90% of uworld with 57% correct of the qbank (doing timed sets of 40, scoring 55-70% on each set). I don't know if im doing well, my exam is a month and a half away, and Im doing terrible on the self assessments. My scores are as follows (taken a week or two away from each other):
NBME 27: 56%
UWSA 1: 44%
UWSA 2: 53%
NBME 28: 59%
NBME 29: 54%
After this, I panicked with the score drop and started using anki more, reread the questions of the NBME that I did wrong, refocused on my weak areas from the NBME using FA and made my own ANKI cards, which I feel helped.
I went feeling confident to the next uworld SA (3) and then I got this:
UWSA3: 53% and this one was very hard
I don't understand why my score is fluctuating that much, I keep remembering the stuff I revised most recently and forgetting the topics I haven't reviewed in more than 2 weeks. I'm just feeling devastated and I don't know how to improve from here. Any help would be appreciated.
it's starting to affect me and im having depressive thoughts now, I don't know what exactly im doing wrong especially with all the time I gave it..
Post 182: I need help, feeling desperate. not scoring well on self assessments
https://www.reddit.com/r/step1/comments/1kkd8f9/i_need_help_feeling_desperate_not_scoring_well_on/
11/05/2025, 19:31:19
1 comment
Hi, I'm an IMG, graduated last year and been studying for the USMLE step 1 since I graduated. my school curriculum is very different from the Step 1 material so I had to do First Aid with BNB just to learn what am I supposed to know for the exam. used sketchy for microbiology, didn't do much anki, and finished 90% of uworld with 57% correct of the qbank. I don't know if im doing well, my exam is a month and a half away and im doing terrible on the self assessments. my scores are as follows (taken a week or two away from each other):
NBME 27: 56%
UWSA 1: 44%
UWSA 2: 53%
NBME 28: 59%
NBME 29: 54%
here I panicked with the score drop and started using anki more, reread the questions of the NBME that I did wrong and refocused on my weak areas from the NBME using FA and made my own ANKI cards which I feel helped. I went feeling confident to the next uworld SA (3) and then I got this:
UWSA3: 53% and this one was very hard
I don't understand why my score is fluctuating that much, I keep remembering the stuff I revised most recently and forgetting the topics I haven't reviewed in more than 2 weeks. I'm just feeling devastated and I don't know how to improve from here. Any help would be appreciated, it's starting to affect me and im having those depressive thoughts now, I don't know what exactly im doing wrong especially with all the time I gave it..
Post 183: Is there a probability of passing based on Free 120 score?
https://www.reddit.com/r/step1/comments/1kkc775/is_there_a_probability_of_passing_based_on_free/
11/05/2025, 18:42:55
6 comments
All the NBMEs give you a chance of passing in a week if you maintain your level of knowledge. Has somebody figured out an unofficial version of that based on Free 120 performance? Just curious
Post 184: Do people report raw NBME’s or EPC?
https://www.reddit.com/r/step1/comments/1kkbbqn/do_people_report_raw_nbmes_or_epc/
11/05/2025, 18:03:50
6 comments
Basically what the title says.. is it generally accepted to post your EPC or the actual raw score. I just wrote form 30 and got an EPC of 67% but raw score is 70%. I test in 2 weeks so kinda bummed about this
Post 185: Center closed and exam was cancelled.
https://www.reddit.com/r/step1/comments/1kka6up/center_closed_and_exam_was_cancelled/
11/05/2025, 17:13:58
18 comments
Hey, im an img,i had the exam scheduled for 10th may, but due to a literal frigging war drone crashing near the center they closed it for that day, im in so much stress now because prometric did email me that the exam is cancelled and they said they’ll send a followup in 3/4 days. I wanted to ask if anyone has experienced the same and will they offer me dates relatively soon or what? Im so confused and they aint replying.
Post 186: Step 1 Mentorship – One Free Introductory Session Available
https://www.reddit.com/r/step1/comments/1kk9pfj/step_1_mentorship_one_free_introductory_session/
11/05/2025, 16:53:23
3 comments
Hello everyone,
I'm a final year medical student and passed my USMLE Step 1 exam in September 2024 and have since mentored local students with a 100% pass rate. I would love to extend my mentorship to the Reddit community.
I currently have four students preparing for their exams over the next 2–3 months and am open to mentoring a few more. The sessions will be live and fully tailored to your needs — including study planning, resource guidance, question-solving strategies, and ongoing support.
To get started, I’m offering one free introductory session. After that, we can discuss further details via DM.
If you’re interested, feel free to message me directly. Looking forward to helping more of you succeed!
Post 187: Not fun
https://www.reddit.com/r/step1/comments/1kk862c/not_fun/
11/05/2025, 15:46:48
14 comments
IM SO TIRED!!!!!
Im trying i swear!!!!
Been studying for almost 2 years and im in the 50’s in the NBME’s..
I have a really bad ADHD and I really don’t know how to manage this anymore.
CLEARLY MY TECHNIQUE ARE NOT WORKING.. I NEED A REALLY GOOD TUTOR THAT HELPS ME WITH THIS.. I KNOW IM NOT BRILLIANT but I know im not DUMB!!!!
Post 188: is skin HY?
https://www.reddit.com/r/step1/comments/1kk6g8r/is_skin_hy/
11/05/2025, 14:32:26
7 comments
whats high yield in skin? is it important overall? can i skip it? what should i ABSOLUTELY do in it? please help.
Post 189: Should I sit for step this week, or should I still take 31 and the free 120?
https://www.reddit.com/r/step1/comments/1kk4y6n/should_i_sit_for_step_this_week_or_should_i_still/
11/05/2025, 13:27:02
15 comments
76% on 26 (5/3) and 27 (5/10). I've read here that two >=65s means you're good to go. Should I sit for step this week, or should I still take 31 and the free 120? Or just the free 120?
Post 190: Nbme 25 fail
https://www.reddit.com/r/step1/comments/1kk2gn8/nbme_25_fail/
11/05/2025, 11:36:05
12 comments
just done with nbme 25 got 57% it was so tough
nothing like uworld or FA
i dont know what to do
Uworld average 78% with 60% done
exam in 1.7 months
i am freaking out and i am about to cry
is it normal to get this?
i see everyone passing first nbme 25 idk what i missed during my study but the questions were very unique
i wasn’t familiar with a lot of the options and i had to guess most of them
Post 191: Guessed every single question!
https://www.reddit.com/r/step1/comments/1kk18bw/guessed_every_single_question/
11/05/2025, 10:38:29
25 comments
As the title says, I guessed every single question even though I scored 75+ in last 3 NBMEs and new free 120. It was hard, damn hard! Vague questions, long scenarios, never ending lab values, had time management issues, brain fog and what not.
Kind of disappointed in myself at this point. Don’t think I’d make it.
Post 192: Anki
https://www.reddit.com/r/step1/comments/1kk102r/anki/
11/05/2025, 10:26:59
6 comments
I’ve been doing anki for micro and pharm from the pepper decks and they’ve been helping me a lot to retain information
I’m giving my exam around the end of August. I’m wondering if I should do Anki for the other chapters as well. If yes then which deck? I’m assuming Anking is standard. I’m not sure if I can get through all the cards in 3 months since I know there’s a lot. So what chapters are more important to do from anki? Immuno? Biochem? What systems?
Post 193: Should I Schedule STEP1?
https://www.reddit.com/r/step1/comments/1kk0yym/should_i_schedule_step1/
11/05/2025, 10:25:25
2 comments
Have given 2 Online NBMEs
NBME 26: 73
NBME 27: 78
Havent attempted the rest, in the middle of Uworld First pass, about 70 percent through, i was scoring 65-80 percent,but then randomly last 4-5 blocks have been in 60-65( i have no shitting clue why)
Net average UWORLD:62
The NBMEs are way easier than Uworld,are these predictive? Should I book the test for about a month from now?(7th June)
Any guidance is much appreciated! Give it to me straight:)))
Edit: If your opinion is to withold and base it on UWorld percentages, do let me know how much i should be scoring before applying
Post 194: UWSA advice
https://www.reddit.com/r/step1/comments/1kjz0d3/uwsa_advice/
11/05/2025, 08:37:45
2 comments
When should I opt to do the UWSA’s? I’ve only completed 37% of UW (took a break in the middle for a few months due to med school exams) but I’ve resumed UW again and it’s going very poorly, hence why I’m waiting to take an NBME.
When do you guys suggest I do the 3 UWSA available on UW? I’m planning on hopefully starting NBMEs in mid August once I cover most of the content and around 60% of UW. Please give your insights. Thank you!
Post 195: Why people says EXAM is nothing like NBME and other says it's exactly like NBME and BOTH is CORRECT!
https://www.reddit.com/r/step1/comments/1kjy3t1/why_people_says_exam_is_nothing_like_nbme_and/
11/05/2025, 07:39:36
15 comments
This is my experience and advise that I would have gave to my self before I sit my exam.
I have sit the exam on 9th of May and read few people experiences who sit at the same day and let me tell you, so much of what they say about the content of the exam i had completely different experience e.g. they says they have nothing from renal I had many from renal ! So focusing so much on content advice is not really helpful as you probably gonna have different set of questions but focus on learning from the experience itself.
First let me assure that we really need better exam experiences in our practice from NBME or free 120 even if they have to include few experimental question cos the vibe from the real exam is so different from the vibe of NBMEs and for such expensive and important exam we deserve to feel the real situation of it.
Also let me assure many of you that if you studied well and you have been on reddit for quite long time reading posts about how shit and hell the exam is that you will find the exam is easier than you was expecting. But if you just left everything and focusing on nbme and you are coming thinking it's like they release nbme 32 and you gonna smash it quickly and go home you will be SHOCKED!
Also whenever I did NBME or Free 120 many times I have felt bad after a block but I end up doing ok and I think many of you have similar feeling is just you guys forget after you see that you scored good grade. And let's be honest we are medical students and we all have classmates will be crying and complaining and they end up smashing the exam and people who did bad but get out thinking it was fair and they end up failing, I'm from those :) so don't panic if you see a guy getting out of the exam crying and his nbmes is 90+
Lets back to the exam, blocks in general are similar and it happens in many blocks that I have five minutes on time and im in questions 36 or 37 ( only one block I didn't have time of the last 3 questions and only one block I have few minutes to check the flags questions).
We go back to the important part, content is the same, the content of first aid book we all know about is gonna be tested but the trick is the way of writing the questions.
Questions is longer ( you will have short questions and easy ones and you read them seventy times thinking there is a trick and there is not but these are jot many ) and the long of questions is not really helpful as they will give alot of irrelevant information and less buzzwords we know and even the descriptions used is a little different than normal.
Also like i would read micro question ( I'm sure i got so so many wrong ) and i feel ok i probably know this, its rash in particular way or diarrhea but when i read the answers nothing is what I was expecting! And i feel I'm sure all of these is wrong! And this because we only know certain type of presentation to each bug that may not be the case in real life. ( that's why people who have real life experience may have an advantage)
How to prepare?
Shorter time on each question! In nbme alot of questions you answer in 3 seconds cos you know for sure and this leaves u plenty of time for harder questions it will not be the case in this exam.
The drugs and diseases may not highly asked by nbme could be asked on real exam like i feel ok i know this drug but its been a while so i forget exactly the mechanism cos at the end of prep you only focus on what we call high yield even if you read it from first aid your mind is not focusing!
There are questions copy paste from our prep i can't recall exactly from where nbme of free 120 but i have seen them ( not many but they exists)
Make sure anything you know you really remember in very short time you will not have time to slowly recall from your memory and write down mnemonics ( I have made silly mistakes :(
Finally to answer the question on the title, people who saying exam is like nbme they are right cos the diseases and drugs and bugs you will be asked about that you have never heard of is very very tiny but at the same time people who saying its nothing like nbme they are right cos the structure of the questions and they way question is writen is completely different from nbme.
I believe I have very good chance of failing but not because the exam is that bad or because I haven't studied enough ( I studied very very hard ) if I fail I wouldn't know who to blame and as I read from someone else it will be even hard to know what to do different in prep if I need to retake it ( I won't ), which may show that there is maybe something wrong with the exam :)
Post 196: Fresh start to step1. Need advice
https://www.reddit.com/r/step1/comments/1kjwx39/fresh_start_to_step1_need_advice/
11/05/2025, 06:17:30
5 comments
Hi I have just started my step1 prep, FA Uworld and bnb. Any advice that nobody gave you but you wish you knew? Appreciated ^-^
Post 197: Need help for USAT
https://www.reddit.com/r/step1/comments/1kjv4yk/need_help_for_usat/
11/05/2025, 04:12:40
2 comments
Hi 19 ( M ) . I have an exam called USAT which is conducted by HEC Pakistan . I'm desperate to win a scholarship which is granted after getting more than 70 marks in this test out of 100 . Last year I tried this test and got only 56 and was brutally humiliated by myself and family . I have studied pre medical in my college because of which I get problems in solving the maths portion of the test . We are not allowed to carry our calculators because of which the calculations become puzzling and time taking . The HEC has not given any sort of resources for this test as they just give some instructions about the marks breakage. Has anybody passe this exam . I really need your help guyyys!!!
Post 198: I hate MSK anatomy and injuries with a passion!!
https://www.reddit.com/r/step1/comments/1kjumuq/i_hate_msk_anatomy_and_injuries_with_a_passion/
11/05/2025, 03:38:36
14 comments
My brain is allergic to this content. Every time a nerve injury, a guyton canal or whatever that shit pops up, my brain is like “skip it!!!!! “ and I skip it. I am pretty sure I’ll score 0% if I do a block on this topic. No, may be a ~20% if I pick all ‘d’s.
Does anybody know a method/resource that I can use to consistently get ~70% in this area that would not leave me brain dead?
Thankyou
Post 199: Need advice
https://www.reddit.com/r/step1/comments/1kjsszc/need_advice/
11/05/2025, 01:37:25
3 comments
What should be the last 15 days strategy?
I am really confused i am exactly 30 days out.
Post 200: Biostatistics
https://www.reddit.com/r/step1/comments/1kjogmb/biostatistics/
10/05/2025, 21:28:24
1 comment
What to study for biostatistics for step 1 , im very much weak , exam in 4 weeks , and what to study in this 4 weeks , im so confused
Nbme _ 27 ----66%
Nbme-28 ----69%
Post 201: Opinions on Step 1 Prep
https://www.reddit.com/r/step1/comments/1kjnkjl/opinions_on_step_1_prep/
10/05/2025, 20:41:06
5 comments
I’ve been preparing for step 1 since the start of my blocks in the second semester of school (last ~6 months) and have finished Neuro/Psych, Infectious Disease, and Cardio so far. Here is what I do as a USMD student.
Anking - BnB, Bootcamp, Sketchy
I do basically all these cards for all these blocks.
Q banks - USMLE Rx, Amboss, UWorld
Once I finish getting all the Anki cards for the block into my review rotation, I do USMLE Rx, then Amboss then UWorld, so far my USMLE Rx average is 82%, Amboss is 71% and UWorld is 82% (have done 26% of the Qbank so far). Why is Amboss so much harder? The Anking’s old Step 1 prep video said he averaged 87% correct in Amboss and 88% in UWorld so I’d like to get closer to those numbers, but I’m not exactly sure how.
Thoughts on my progress/plan? I’m approaching Step 1 from the mindset of treating it like it’s scored so I can prep as responsibly as possible for Step 2. One thing that’s kinda annoying is I’m plateauing at those Qbank scores and wondering if people have advice on how to get closer to 90% on UWorld, or if that’s even something I should be aiming for.
Thank you
Post 202: Passed after feeling like crap for the past 2 weeks!
https://www.reddit.com/r/step1/comments/1kjl7e5/passed_after_feeling_like_crap_for_the_past_2/
10/05/2025, 18:45:10
12 comments
**Step 1 Experience – Trust Your NBMEs, Not Your Feelings**
Just wanted to echo what so many others have said: **DO NOT trust how you feel after the exam.** I walked out in complete shock, couldn’t talk to anyone for at least an hour. It didn’t feel *crazy* hard, but I got hit with unexpected topics that really threw me off. I genuinely thought I failed and spent two stressful weeks spiraling—thankfully I had a trip planned to distract me.
When I opened my score report, I still **passed**.
**So please, trust your NBMEs and Free 120. They really are the best predictors.**
# My Scores
* **CBSE 1 (2/18)** – 55
* **NBME 28 (3/4)** – 60
* **CBSE 2 (3/18)** – 65
* **NBME 29 (4/1)** – 71
* **NBME 30 (4/8)** – 76
* **NBME 31 (4/15)** – 71
* **Free 120 (4/19)** – 75
* **Step 1 (4/24)** – Passed
# Timeline & Resources
* **Dedicated started in February (9 weeks total)**
* This pace worked for me—busy but had daily downtime
* Resources I used:
* **UWorld** – 53% complete, 65% avg
* **Pathoma** – watched all videos after CBSE 2
* **Sketchy** – rewatched all Micro + Pharm
* **Anki** – UWorld miss cards daily, Pathoma/Sketchy deck when I had time
# Study Details
* **UWorld strategy:**
* Started by systems → switched to random in March
* Timed mode began \~1 month before the exam (wish I’d started sooner)
* Focused on understanding missed Qs + suspending relevant Anki cards
* **Pathoma:**
* Watched all vids at 2x speed, didn’t take notes
* The textbook it comes with is super readable
* **Sketchy:**
* Micro = essential (pure memorization)
* Pharm = helpful, but less of a priority if short on time because the videos aren't as good
* **Anki:**
* Always finished my UWorld miss deck
* Pathoma/Sketchy deck = secondary priority
* Doing Anki daily gave me a small sense of progress and motivation
# Practice Test Tips
* Took most NBMEs untimed (except CBSEs, NBME 30, Free 120)
* Not ideal, but I had bad test anxiety and couldn’t sleep before them
* I still tracked time and never came close to running out
* **If possible, practice under timed conditions earlier than I did**
# Test Day Recap
* Woke at 5:30 AM, ate oatmeal + coffee
* Got to testing center early, started before 8 AM
* Finished each section with 10–15 mins to spare
* Skipped stats Qs, came back to them at the end
* **DO NOT change your answers unless you're 100% sure**
* I would’ve missed 2 questions on test day if I changed my gut answers
* Snacks: granola bars, turkey jerky (ate more jerky than I have in my life lol), soda before the last section
* Felt alert the whole time despite nerves
* By the last section I literally thought, "*let’s finish this b\*tch*" and that got me through
# Final Advice
* **Do not take the exam until your practice test scores make you feel ready.**
* You’ll probably feel unsure during the test and while waiting for your score—normal!
* Having solid practice scores helped keep me grounded
* **Be proud of yourself.**
* This test is incredibly hard
* Take a moment post-exam to look in the mirror and say: *“I did my best, and that’s enough.”*
Happy to answer any questions! Wishing you all the best—you’ve got this
Post 203: [Step 1 Passed] From 45% CBSE to 75% Free 120 — ChatGPT, UWORLD, Mehlman, and Last-Minute Grind Saved Me
https://www.reddit.com/r/step1/comments/1kjkp20/step_1_passed_from_45_cbse_to_75_free_120_chatgpt/
10/05/2025, 18:21:37
78 comments
Hey everyone—just wanted to share my Step 1 story in case it resonates with anyone out there in panic mode or feeling behind. I was there. My scores started low, I postponed my exam, and I doubted myself constantly. But I pushed through—and passed. Here’s how I did it.
# Assessment Timeline
* 02/24 – CBSE 01: 45%
* 03/07 – UWorld SA1: 47%
* 03/14 – NBME 27: 63%
* 03/20 – CBSE 02: 64%
* 03/20 – NBME 31: 63%
* 04/03 – NBME 30: 64%
* 04/08 – NBME 29: 75% → This jump made me second-guess everything—I thought maybe NBME 29 was just easier.
* Free 120 (week of exam):
* Block 1: 75%
* Block 2: 73%
* Block 3: 75%
Originally planned to test on April 7, panicked, and pushed it to April 17. That 10-day grind turned out to be worth it.
# What Helped Me Most
**\*\*Pathoma Ch 1-6 is a must. Love Dirty Medicine for specific topics I found myself struggling on. Med School Bootcamp for targeted review on topics not covered by Dirty Medicine.**
# ChatGPT (Medical Questions Tutor)
I uploaded PDFs and used the **Medical Questions Tutor program** on ChatGPT to:
* Teach back topics I was shaky on
* Break down complex systems (especially glomerulopathies)
* Practice clinical reasoning It honestly felt like having a personal tutor available 24/7.
# Mehlman PDFs
I added Mehlman Rapid Reviews in the last few weeks and they were 🔥 for last-minute consolidation. Super clutch when UWorld burnout kicked in.
First Aid Textbook?
Barely touched it—only used it for glomerular diseases. Otherwise I leaned on UWorld + ChatGPT (with integrated First Aid pdf) for understanding.
# UWorld Stats
* Completed 54% total
* 50% average
* Final week: scoring 65–70% on random, timed blocks
I was worried I hadn’t finished the full bank, but turns out you don’t need to as long as you review deeply and intentionally.
# Key Takeaways
* **Upward trends matter.** One bad score doesn’t define your readiness.
* **Free 120 is gold** — 70–75% is a great sign.
* **Quality > quantity on UWorld** — better to understand half than rush through 100%.
* **Teach-back with ChatGPT** helped lock in weak spots.
* **Mehlman PDFs** are killer for review if you’re burned out on questions.
# Anki? Not for Me
I’ll be honest—I didn’t use Anki. I tried it early on but couldn’t stay consistent, and it just didn’t fit the way I learn. Instead, I focused on active recall through teach-back, using ChatGPT’s Medical Questions Tutor to quiz myself, explain concepts out loud, and drill weak areas. If Anki isn’t clicking for you, you’re not doomed—there are other ways to reinforce knowledge.
# Final Thoughts
If you’re drowning in doubt and second-guessing your timeline—same. But progress compounds fast when you’re intentional with your review. You don’t need to be perfect, just consistent. You got this.
Message me if you’re in your final stretch and want to bounce ideas. Happy to help.
—An M3 who panicked, postponed, and passed anyway.
EDIT:
Just wanted to share my Step 1 experience in case it helps anyone preparing.
For me, the **first block was definitely the hardest**—not because the questions were that bad, but because of the nerves and just getting used to the testing environment. It took that whole first block to settle in. After that, things felt more familiar and similar to UWorld, and I found my rhythm.
I **skipped the tutorial** at the beginning to save time for breaks. Then I followed a consistent break plan: took a **5-minute break after every single block**—just enough time to breathe, grab water or a snack, and reset. At the halfway point (after 4 blocks), I took a **30-minute lunch break**. Ate something light, stretched, and didn’t think about the exam at all. That helped me come back with a fresh head for the last half.
Honestly, having that break structure made a huge difference in staying focused and not burning out. Test day is long, but if you pace yourself and keep your head clear, you’ll get through it strong.
Wishing everyone good luck—you’ve got this
Post 204: NBME 25
https://www.reddit.com/r/step1/comments/1kjkjvd/nbme_25/
10/05/2025, 18:15:07
5 comments
i am taking my nbme 25 tomorrow
i am scared i am gonna mess up and get disappointed and discouraged to not study anymore
Post 205: Should I do NBME 31 or Free 120?
https://www.reddit.com/r/step1/comments/1kjf7w8/should_i_do_nbme_31_or_free_120/
10/05/2025, 14:13:04
7 comments
I'm 1 week away. I plan to do one tomorrow and the other one on Wednesday. Which one first? Which ones more important?
Post 206: Hello guys
https://www.reddit.com/r/step1/comments/1kje9vs/hello_guys/
10/05/2025, 13:30:24
5 comments
I just started studying for step 1 exam
And
I need advice from you guys
Is uworld morethan enough for the exam?
If there is something i don’t understand where should i go to look for it and
What is advice that you will give me that you should knew it earlier?
Post 207: Uworld pricing
https://www.reddit.com/r/step1/comments/1kjcvm8/uworld_pricing/
10/05/2025, 12:27:39
1 comment
Why are the uworld prices different on the app and the website?
Post 208: Can’t find Sketchy pharm videos
https://www.reddit.com/r/step1/comments/1kjcpgh/cant_find_sketchy_pharm_videos/
10/05/2025, 12:20:02
1 comment
Hello, can anybody please give me the link to sketchy pharm videos (I couldn’t find them on telegram or anywhere else)
Post 209: Golden rule to score higher.
https://www.reddit.com/r/step1/comments/1kjchx9/golden_rule_to_score_higher/
10/05/2025, 12:10:13
25 comments
Ok so some of you may already know it and that’s cool. But I’m here to spread the wealth. I learned something during my grad school years that’s worth discussing a bit. The golden rule to test taking. It’s a rule that has boosted my score on every exam I have taken. Decided to post it here, could have put it in r/MCAT or whatever testing subreddit. This is it:
**NEVER NEVER NEVER CHANGE YOUR ANSWER**
Unless read it and see an “except” that you didn’t see before or you look at your answer while reviewing and tell yourself “this is an OBVIOUS mistake”, “clearly wrong”.
If you tell yourself “But maybe that is the answer…” don’t change it. If you have ANY doubt NEVER switch your answer. It’s SUPER tempting but you have to remember this rule.
Good luck!
Post 210: Is the second year of med school in most U.S. schools enough to pass Step 1?
https://www.reddit.com/r/step1/comments/1kjc6yu/is_the_second_year_of_med_school_in_most_us/
10/05/2025, 11:56:37
50 comments
Hey everyone,
I’m curious — for those of you who’ve gone through the process, do you feel like the standard second-year med school curriculum at most U.S. schools is enough to pass Step 1 (now that it’s pass/fail)?
Assuming someone pays attention in class and does reasonably well on school exams, would that foundation be sufficient to pass Step 1, or is dedicated board prep with things like UWorld, First Aid, and Anki still essential?
Trying to gauge how much extra work is realistically needed these days. Would love to hear your experiences.
Thanks!
Post 211: past NBME images docs before taking NBMEs or nah?
https://www.reddit.com/r/step1/comments/1kjc0di/past_nbme_images_docs_before_taking_nbmes_or_nah/
10/05/2025, 11:48:20
3 comments
Ik it inflates but does it really matter if the same pics might pop up on the actual?
Post 212: I passed after 2. whole. years.
https://www.reddit.com/r/step1/comments/1kjbyam/i_passed_after_2_whole_years/
10/05/2025, 11:45:46
46 comments
Yes, the title is true 😭 I have a semi-long story, so if enough people are interested in hearing it, I would love to tell it! For now, here are my stats and resources I used:
* **Dedicated / focused studied**: 09/23/2024 to 04/25/2025 (seems long, but again, it's kind of a long story. My initial Dedicated started in March 2023, but my latest study method started in September 2024)
* **Practice exams**:
* NBME 26 on 02/14/25: **68%**
* NBME 27 on 03/14/25: **62%**
* NBME 29 on 03/28/25: **72%**
* CBSE 1 on 04/03/25: **69%**
* my advisor did not let me take the CBSE until i broke 70% on an NBME, thank goodness!
* NBME 30 on 04/11/25: **68%**
* NBME 31 on 04/18/25: **73%**
* Old Free 120 (2021): **84%**
* I did the 1st block on 04/21 as a warm up and the last two blocks on 04/23
* New Free 120 (2024) on 04/22/25: **66%**
* highly suggest taking it at the testing center you're going to be at for your real exam
* used [Medschool Bootcamp](https://bootcamp.com/blog/new-free-120-nbme-step-1-explanations) to review
* STEP 1 on **04/25/25**: **PASSED** (1st attempt!)
* First Aid 2024, AMBOSS Q Bank, UWorld Q Bank
* I paired the FA topics and the articles in AMBOSS and did as many questions as I could. **EDIT**: Here is the list for the [FA 2024 and AMBOSS topics](https://docs.google.com/document/d/1_LejRvV4Q4pX__8p_YaEb19qyf0kGOM5-pyCNjYEsIs/edit?usp=sharing). I'm sharing the link instead because I'd like to keep the emails of everyone who sent me a request anonymous as well as I didn’t want to accidentally not send it to someone! It looks like a lot (bc it is lol), but I honestly would not have gotten these scores without exhausting all of these questions.
* Additional questions (physiology was alwayssss my lowest score and prevented me from getting higher scores, so I did some \[not all\] of these and saw an improvement shortly before the exam! I downloaded each from Lib genesis)
* BRS physiology, Linda Costanzo Physiology Cases AND Problems (4th editions)
* Anki (I got all decks from this [reddit](https://www.reddit.com/r/step1/page) page OR the [medical school anki reddit page](https://www.reddit.com/r/medicalschoolanki/); most were downloaded 2 years ago, so I don't exactly which comes from where!)
* Sketchy Micro and Pharm, Pixorize, Anatomy HY, Netter Better (from AnKing), Hoopla (for quick concepts that I could not remember)
* Videos (mainly watched for the last month of studying for concepts I still didn't know)
* Medicosis Perfectionalis, Rhesus Medicine, Dirty Medicine
* Misc
* Med Mnemonics app (also added my own and reviewed as much as I could the week before and week of my exam)
* A friend willing to review a few NBME's with me
* An advisor who didn't give up on me for 2 years
I'd like to note that I was on a leave of absence and my preclinical knowledge was nonexistent. I **had to relearn everything on my own.** Therefore, I had the privilege to essentially take 1 to 1.5 weeks to learn each system in great detail. I did content review from 09/23/2024 to 03/07/2025.
This was an incredibly long journey and I am beyond ecstatic to **finally** move on. Now that I have the clinical knowledge, I feel a bit more comfortable going into my rotations. Please do not give up. I went from failing majority of my preclinical exams to passing STEP 1 on the 1st try. **I know it's cliche, but if I can do it, so can you <3. Best of luck!!!**
Post 213: "Struggling with Physiology or Pharmacology for USMLE? Let’s Discuss Strategies!"
https://www.reddit.com/r/step1/comments/1kj9s1n/struggling_with_physiology_or_pharmacology_for/
10/05/2025, 10:02:05
1 comment
**\[Discussion\] Physio & Pharm Pain Points – What Trips You Up?**
I’ve been teaching USMLE-focused Physiology and Pharmacology for a while now, and I’ve noticed a few recurring pain points students face:
* Memorizing drug side effects (looking at you, verapamil-induced constipation + hyperprolactinemia 🥛)
* Connecting clinical vignettes to basic science (e.g., why beta blockers mask hypoglycemia in diabetics)
* Retaining high-yield concepts without drowning in details
Over time, I’ve found that **mnemonics, case-based learning, and breaking mechanisms down step-by-step** really help. Some examples:
* "BETA-BLOCKERS blunt BRAIN, BRONCHI, BLOOD sugar" for side effects
* "SHIPP-E" for drug-induced lupus (Sulfa, Hydralazine, INH, Procainamide, Phenytoin, Etanercept)
If you’re prepping for Step 1 or Step 2, I’d love to hear:
💬 What’s *one* Physiology or Pharm topic that keeps tripping you up?
✅ And what’s helped you finally understand it?
Let’s crowdsource some solid strategies that actually work.
Post 214: Most accurate self assessment?
https://www.reddit.com/r/step1/comments/1kj9d9x/most_accurate_self_assessment/
10/05/2025, 09:41:22
6 comments
Hey guys, I took the bootcamp 160 question self assessment and I felt like it was actually a lot similar to step 1. I failed step 1 and I kind of remember how it was taking the exam. With medium to long vignettes and a lot of third order questions.
It wasn’t as hard as UWSA but there was a lot of concepts that were in my exam that showed up on the self assessment and I am afraid it might be more reflective of the real exam.
Has anyone taken this self assessment? I wanted to know what their thoughts are. And if they also feel the same. Personally I do not believe any of the nbmes or free 120 are truly reflective of step 1.
Post 215: Help understanding metabolic acidosis/alkalosis and relationship with potassium
https://www.reddit.com/r/step1/comments/1kj5t70/help_understanding_metabolic_acidosisalkalosis/
10/05/2025, 05:52:05
2 comments
Why is metabolic acidosis associated with hyperkalemia and why is metabolic alkalosis associated with hypokalemia?
And can someone clarify the anion vs non anion gap metabolic acidosis and how they relate to potassium and also why is non anion gap metabolic acidosis also called hyperchloremic acidosis?
Post 216: NBME 25 & 26 got me flustered...
https://www.reddit.com/r/step1/comments/1kj5lke/nbme_25_26_got_me_flustered/
10/05/2025, 05:36:59
8 comments
Hello! After a thorough revision of FA 3rd time (Uworld completed on Jan 25th), I gave my first NBME 25 on April 28th, only got 53%. I reviewed ALL the topics that I got wrong/guessed on in depth. Like if I forgot the MoA of a diabetic drug, I did revision of all the diabetic drugs. It took me almost a week. I gave NBME 26 yesterday, instead of a rise in score, I fell to a 51%.
Is this normal?
I felt that the topics heavily tested on NBME 25 barely made an appearance on 26. It was heavily based on DNA genetics portion, biochem & virology.
I want to keep going and give NBME 27 (14th) & 28 (20th), give each next NBME with a review of the previous so that I have a comprehensive list of topics AND review of those topics from FA. And after NBME 28, give NBME 29 with a full fledge revision (27th) The reason I want to do this is because I dont have particular weak systems, I just have gaps in those systems. Like in renal, I'd get questions on physiology correct, patho of renal cell carcinoma or renal artery stenosis correct but nephritic wrong. Same for endo, like I'd get concept based questions correct but I'd forget the mnemonic for IP3 pathway signalling hormones and the question would ask that.
Is this the correct approach? Was NBME 25 & 26 an anomaly where the tested topics were so far apart that a review wasn't beneficial particularly for these NBMEs but will be significant for later, upcoming NBMEs?
I will follow one of two routes after my score in NBME 29
1) Score < 60% in NBME 29 (Plan B)
I will move to 2 random Uworld blocks in the morning and review FA in the evening till night (1 to 2 days per system) until I complete all 10 systems (June 8th to 10th). After that, give NBME 31, 30 after since I have heard it is tougher.
2) Score > 60% in NBME 29 (Plan A)
Continue with the next NBMEs as scheduled, keep building up a bank of HY repetitive topics, keep revising them and incorporate any new info.
I would really appreciate if someone could give me a solid, blunt, detailed answer to my queries.
Thank you soooo much!
Post 217: how to jump my nbme score?
https://www.reddit.com/r/step1/comments/1kj5heg/how_to_jump_my_nbme_score/
10/05/2025, 05:28:20
1 comment
i need to get a ten percent increase in my score to reach the safe zone
i am quite stagnant rn, nothing seems to work and every nbme i score the same
what can help?
Post 218: Should I reschedule
https://www.reddit.com/r/step1/comments/1kj4gnh/should_i_reschedule/
10/05/2025, 04:15:51
5 comments
I’m an IMG, and I think I made a serious mistake. In January, I scheduled my Step 1 exam for July without first studying, gathering resources, or figuring out how much time I’d actually need. I did it hoping that putting myself under pressure would stop me from procrastinating and help me overcome my fear of the exam. But now it’s May, and I’m nowhere near ready.
I’ve tried studying with First Aid and doing 40 UWorld questions a day, but it’s not working. I keep making mistakes because I can’t properly differentiate between pathologies. It’s started to feel like I’m just trying to game the test instead of truly understanding the material.
I also tried Bootcamp and followed their 9-week schedule. I study 12 hours a day, but it takes me a really long time to get through things because I’ve forgotten so much of the basics. Even when I do understand a topic, the process is so slow and overwhelming that I end up losing motivation.
Now I’m wondering: should I reschedule my exam and give myself more time? Or is this just how everyone feels during Step 1 prep? I feel like I keep learning and forgetting everything. I know I’m not prepared—but I don’t know what to do.
Post 219: Post exam rant time..
https://www.reddit.com/r/step1/comments/1kj3la5/post_exam_rant_time/
10/05/2025, 03:16:48
45 comments
My brothers and sisters in Christ, what the FUCK was that.
Tested 5/9, 5 week dedicated. I barely slept the night before, got maybe 4-5 hours of sleep because I was so anxious. First block sent me into a panic attack immediately. I marked about 25 of the 40 questions off the bat. I feel like my brain didn't fully "wake up" until halfway through the exam. I did some practice questions when I woke up to get my brain working but it wasn't enough.
Things I expected would be on the exam since people called them HY: biostats, endocrine, renal.
Nope. There was ONE question with a calculation and it was simply subtracting two numbers. Only a handful of endocrine. And not a SINGLE nephrotic/nephritic syndrome or nephrolithiasis question on my form.
Things that instead showed up ALL OVER my exam: neuro (holy shit so much neuro), every single Sketchy bug, drugs that I haven't heard of, genetics (literally had a question about founder effect/genetic drift/equilibrium like come on in what doctor world do I need to know this), pure biochem (pathways).
Almost every single question stem required me to scroll (I use the second text zoom option tho). Some of them were literally an essay and then the question at end asks a completely left field question. For example (this isn't a test question but just to illustrate my point): "Patient comes in complaining of shortness of breath. \[insert the entire H&P here\] What question should you ask next to solidify the diagnosis? Diet, sexual history, mood, relationships?" Like bro please he just has asthma 😭 A lot of questions felt like I was trying to be a mind-reader.
And as expected, a lot of third-order or even fourth-order questions. For example, questions like "What drug might this patient have taken that would have interfered with another drug for his condition to cause his symptoms?" But neither the "other drug" nor the condition was named. So if you mess up on any one of those four steps, you're toast.
A lot of "trick questions" too. The vignette would describe what seemed like a totally obvious disease, complete with buzzwords, but there would be one tiny phrase that hinted at an alternate diagnosis. If you missed that phrase in the PAGE of text then welp, sucks to be you.
First three blocks were the worst. By block 4 I feel like I basically just dissociated my way through the entire exam. The last 2 blocks felt much more like the Free120 but by that point my brain was so tired I really just wanted it over with and probably rushed to mark answers without thinking them through.
I feel absolutely miserable. I had 70+ on almost all my NBME forms, 78 on Free120. I thought I was ready but I found myself super discouraged. I'm sure that also affects things too -- there were some simple recall-type questions where I walked out and then remembered the correct answer, but I was so panicked during the blocks that my mind completely blanked out.
Post 220: Passed! You can pass too.
https://www.reddit.com/r/step1/comments/1kj3idw/passed_you_can_pass_too/
10/05/2025, 03:11:25
45 comments
My recommendation is simple. Crack 70% on an NBME. Get >65% on free 120. Take the exam. I started studying the first week of March and took it 04/19.
These are my scores:
Form 28: 56%
Form 29: 56%
Form 30: 64%
Form 31: 74%
Free 120: 65%
I really feel like if you just take the time to go over the exams (one exam over the course of 3-4 days). You’ll see progress. They all test similar concepts.
I used Uworld (48% of the bank at 56% correct) and YouTube AND CHATGPT. I watched Osmosis (most), Hermando Hasudungan (most), Ninja Nerd (some) and dirty medicine (some).
My opinion is that this exam looks way scarier than it actually is. The buildup is crazy. It’s super stressful. You have to trust the numbers and take a leap of faith. You will pass. Good luck everyone!
**Trust the numbers**
Edit: I took notes with a pen and paper from the start. I didn’t really review them until like a week out. I read them before bed. I ran through 3 pens and 6 notebooks. I should have just used my iPad…
The writing was just another way for me to try to memorize things.
I also ankied pathoma chapter 1-3. I feel like chapter 3 is easy points. Straight memorization!
Post 221: How much did your NBME scores jump during dedicated?
https://www.reddit.com/r/step1/comments/1kj2l5a/how_much_did_your_nbme_scores_jump_during/
10/05/2025, 02:10:39
17 comments
I just took my first nbme and got 60% correctness. I really want to try and hit a high score on the last one to convince myself I could realistically score high on step 2 since the topics build on each other. If my test isn’t until near the end of June, how much of a score jump should I see?
Post 222: I feel like i will drop this whole exam
https://www.reddit.com/r/step1/comments/1kj0t3d/i_feel_like_i_will_drop_this_whole_exam/
10/05/2025, 00:24:27
13 comments
Hi
So i ve been studying since last October with my bf for the exam.
At the end of april we wanted to sum things up and solve nbme since it’s been ages
I got 52% nbme 21
And 53% on one of the blocks of nbme 20 ( yes i solved one block only )
So i decided to study like the finals and serious mod on
And i studied really lot of topics that i had been weak with
Never the less i saw my progress in these topics while trying some uworld questions related to them
Today i wanted to measure my progress
And bam nbme 25 54%
NO PROGRESS
My bf is not having any problems his scores are really good
I just feel like i am dumb and cant do it
Didn’t book yet but i was planning to book in july
I just cant give up i dont want to disappoint him and leave him to this devilish exam alone
I am sure he got it but i would feel like i am a disappointment to him…
Otherwise i am really fed up and feel like i cant do it
Post 223: Why is it called “overriding Aorta” in ToF?
https://www.reddit.com/r/step1/comments/1kizsp8/why_is_it_called_overriding_aorta_in_tof/
09/05/2025, 23:26:35
5 comments
It doesn’t make much sense to me, I am probably missing something. But, isn’t the aorta at its regular location?
Post 224: Postponing
https://www.reddit.com/r/step1/comments/1kizqlu/postponing/
09/05/2025, 23:23:15
1 comment
I’ve been studying for 16 weeks straight, only taking a some days off here and there when sick or taking a break for loved ones.
CBSA1 (11/15/24): 31 —> CBSA2 (2/6/25): 42 —> CBSA3 (4/1/25): 45 —> NBME 26 (4/20/25): 56 —> NBME 28 (4/28/25): 60 —> NBME 29 (5/2/25): 60 —> NBME 31 (5/9/25): 61
My exam was scheduled for 5/15, but I have decided to extend my eligibility period and push my exam to August. This means I will have to take it while doing rotations, but I need to take a step back and figure out why I’m plateauing.
Such a heavy decision as I see all my peers posting their Ps, and really wish that I had this exam out of the way by now, but at the same time I feel so much peace and weight off my shoulders. I don’t wanna risk an F… Definitely need a break and think that extra time can only do me good. If anyone has any advice for continuing to prepare while on M3 rotations, lmk. I’m a current USMD M2.
Post 225: Advice for changing exam prometric centre
https://www.reddit.com/r/step1/comments/1kizmg7/advice_for_changing_exam_prometric_centre/
09/05/2025, 23:16:54
4 comments
Hi, Im from India
My testing is in 16 days
And there's a war like situation boiling between India-Pakistan
If it breaks out (god forbid no) then what options do i have to change the prometric centre to a safe location
My NBME's are like 72 to 76% so i think i am ready to sit the exam
i just want some opinions...... Thank you
Post 226: Last NMBE 30 64%💔💔😭😭😭exam in 10 days can’t extend plz help me
https://www.reddit.com/r/step1/comments/1kivde6/last_nmbe_30_64exam_in_10_days_cant_extend_plz/
09/05/2025, 19:38:12
10 comments
I’m exhausted 😭
Post 227: Help
https://www.reddit.com/r/step1/comments/1kiv19w/help/
09/05/2025, 19:22:26
3 comments
I have watched all b&b videos and now I solve UWorld, I found that I forgot everything thing, Now I'm watching b&b, and I think this is a time wasting, what shall I do?
Post 228: Exam 29/04 Freaking out about score this week
https://www.reddit.com/r/step1/comments/1kiu7d2/exam_2904_freaking_out_about_score_this_week/
09/05/2025, 18:44:20
17 comments
Edit: got the P!!!!!!!!
Hey everyone,
I took Step 1 on 29/04, and *wow*—it was way more challenging than I expected.
So many strange question stems and weird answer choices. The whole thing felt kind of surreal. I came out of the exam feeling like complete trash, second-guessing everything.
* NBME 25: 68.5%
* NBME 26: 73%
* NBME 27: 67%
* NBME 28: 69.5%
* NBME 29: 73%
* NBME 30: 73% (online)
* NBME 31: 72% (online)
* Free 120 (new): 74%
I know I should trust the process and these numbers, but it’s hard not to spiral a little post-exam—every tough question keeps replaying in my head.
Please tell me I’m just overanalyzing this 😅
Would really appreciate hearing from others who felt the same and passed trying to stay sane while I wait.
Post 229: Sick and test is in 1 week
https://www.reddit.com/r/step1/comments/1kiu48k/sick_and_test_is_in_1_week/
09/05/2025, 18:40:17
5 comments
I got bodied by a mystery virus (not COVID or the flu, thankfully!), but my exam is in 1 week. I can't reschedule because no dates are available. I really don't want to take the exam while sick, especially since my NBMEs have been low. This is also my second attempt at Step 1. I don't know what to do besides check Prometric every day and hope a later date opens up. My brain feels fried and everything hurts right now -- I've been cramming UWorld and Amboss questions and doing much worse than usual. I just want the P!!!
Post 230: Rapid Review FA2025
https://www.reddit.com/r/step1/comments/1kitk6g/rapid_review_fa2025/
09/05/2025, 18:15:30
1 comment
https://drive.google.com/file/d/1FIY105mr2BF2pXPgetbE-NHEN69pZsJ4/view?usp=drivesdk
Post 231: Testing tomorrow
https://www.reddit.com/r/step1/comments/1kit5ze/testing_tomorrow/
09/05/2025, 17:58:51
10 comments
Today has felt like the longest most adrenaline fueled day
Post 232: Pdfs mehlman
https://www.reddit.com/r/step1/comments/1kisfs3/pdfs_mehlman/
09/05/2025, 17:27:48
6 comments
Can anyone plz tell is it worth to do mehlmans pdf
any recent test taker can plz guide
Post 233: Lower on NBME 26 to 27
https://www.reddit.com/r/step1/comments/1kiqufs/lower_on_nbme_26_to_27/
09/05/2025, 16:19:43
1 comment
I want to cry! I got 53% correct score on 26 (52% chance of passing) and one week later got a 52% correct score (46% chance of passing). I have 42 days until I take Step. Is this enough time? I thought it went slightly better and now I’m panicking!!
Post 234: After basics, should I study system-wise or subject-wise for Step 1?
https://www.reddit.com/r/step1/comments/1kin0tp/after_basics_should_i_study_systemwise_or/
09/05/2025, 13:39:26
1 comment
Hi everyone! I had a question and need some advice.
I'm using AnKing, Pathoma, and Sketchy for Micro and Pharm, and Physeo for the rest of the subjects. My plan is to first go through the basic subjects like biochem, immuno, micro, ... etc.
Once I finish those, I’m unsure how to proceed:
**Should I switch to a systems-based approach (CVS, Neuro, Renal, GI, ...),**
**or continue with a subject-based approach (Embryo, Anatomy, Physiology, Pathology, Pharm)?**
My foundations are still weak, so I want to build a solid understanding, not just cram for the test. I plan to take Step 1 in Jan 2026, but my bigger goal is to truly understand and connect the material so I can become a good doctor — I’m aiming for Family Medicine.
Would love to hear how others approached this, especially if you started from a similar place.
Thanks!
Post 235: Any Sketchy discount groups active right now?
https://www.reddit.com/r/step1/comments/1kimjap/any_sketchy_discount_groups_active_right_now/
09/05/2025, 13:19:26
2 comments
Hi I saw that some people use the platform to make a sketchy group code discount with enough sign ups! Please let me know if there is any going on right now!! (:
Post 236: FA or Uworld
https://www.reddit.com/r/step1/comments/1kim50v/fa_or_uworld/
09/05/2025, 13:03:08
4 comments
hey everyone i just need an advice of if i should go through FA one more time or thats enough
my exam in 1.5 month and i already feel like ik everything i don’t think i miss a lot
i think Uworld is enough to give me a recap on everything
but i see people here reviewing FA and depending on it
any advice???
Post 237: NBME Socres STEP1
https://www.reddit.com/r/step1/comments/1kilo6n/nbme_socres_step1/
09/05/2025, 12:43:59
4 comments
Nbme 25: 71%
Nbme 26: 66%
Nbme 27: 69%
Nbme 28: 71%
Nbme 29: 73%
As I ready to take the exam within a week? If not when?
Post 238: Step1 pregnancy physiology
https://www.reddit.com/r/step1/comments/1kil1ab/step1_pregnancy_physiology/
09/05/2025, 12:17:29
1 comment
How important are concepts for pregnancy physiology? How heavily tested are these?
-Fertilization, implantation, cleavage, blastogenesis.
-Sex differentiation.
- Maternal adaptations to pregnancy.
- Labor physiology.
- Lactation physiology
What resources would you recommend?
Thanks in advance.
Post 239: test in 6 days, FA or mehlman
https://www.reddit.com/r/step1/comments/1kijd5i/test_in_6_days_fa_or_mehlman/
09/05/2025, 11:07:10
2 comments
Thinking about going over the systems one more time in the last 6 days, so much content in limited time, any advice that I should go with FA or mehlman HY? Thanks!
Post 240: Looking for advice from those who Passed + Neurotic
https://www.reddit.com/r/step1/comments/1ki55dm/looking_for_advice_from_those_who_passed_neurotic/
08/05/2025, 21:04:06
1 comment
Post your night before or before the exam morning tips please!
Yes, I know we need to trust our prep up to this point, but I already know I'm likely going to end up with insomnia and be up a few hours before the exam. I plan to do a hard workout the night before and tire myself out.
I also have roughly a 30 minute drive to the center.
Any last minute advice is helpful. Thanks and congrats on beating the beast!
Post 241: Main Mehlman PDFs
https://www.reddit.com/r/step1/comments/1kiim81/main_mehlman_pdfs/
09/05/2025, 10:34:02
11 comments
Hi, I’m very short on time but I’ve heard that Mehlman’s PDFs are HY and very helpful. Can someone please share which ones should I do?
Post 242: Memorizing
https://www.reddit.com/r/step1/comments/1kihyym/memorizing/
09/05/2025, 10:03:32
8 comments
What are some things on step1 that MUST be memorized (not just read/understood) ?
Post 243: First NBME-how to proceed with Uworld?
https://www.reddit.com/r/step1/comments/1kihpos/first_nbmehow_to_proceed_with_uworld/
09/05/2025, 09:51:32
4 comments
Hey all,
I made a post a little while ago but have an update.
2nd week of dedicated took form 25 got 64%.
Have 56% of Uworld completed testing in 4 weeks. How would you guys recomend continuing from here? I’m planning one revising the NBME but then after should I continue through uworld until the next NBME? Or should I focus more on NBME material?
Any tips would help, thank you for your time!
Post 244: Behavioural science questions are f** up!
https://www.reddit.com/r/step1/comments/1kihc6y/behavioural_science_questions_are_f_up/
09/05/2025, 09:32:43
3 comments
I have been doing u world lately and i am unable to answer even a single question right from behavioural science.Please help me where to learn this subject and how to answer them correctly.Pls 😔!
Post 245: Last 3 weeks… Any tips before I delete this app for a while?
https://www.reddit.com/r/step1/comments/1kigwie/last_3_weeks_any_tips_before_i_delete_this_app/
09/05/2025, 09:09:51
4 comments
I got 3 weeks left, still lowkey feel like I don’t know anything but is what it is lmao. Can’t really push back the test anymore.
Any last min tips/ advice?
Done NBME 25-28 so far (63, 70, 64, 68) and just took UWSA 1 today (65). Will do the rest of the NBMEs and review over next 2 weeks.
Planning do a sketchy micro rewatch, Mehlmann arrows, Mehlmann pharm module and a once over my notes (basically from UW) for the subjects I’m weak in.
Still got ~700 AMBOSS questions left but I think I’ll just do the ethics and stats parts of them and call it a day on the other MCQs.
Post 246: Any German IMG who passed Step 1 here?
https://www.reddit.com/r/step1/comments/1kigr5z/any_german_img_who_passed_step_1_here/
09/05/2025, 09:01:58
1 comment
Hey, I am a German IMG wanting to pursue IM residency and then Gastro Fellowship. I want to do both Usmle Step 1 and Step 2 directly after my M2 State Exam and would really appreciate some advice. How long will I have to realistically plan for Step 1 if I only need to pass? How to study for it considering I am doing it right after M2? Is only doing Uworld enough? How long to study for Step 2 for a high score? And what is the best way? Thanks!
Post 247: IMG preparing for step 1
https://www.reddit.com/r/step1/comments/1kigilo/img_preparing_for_step_1/
09/05/2025, 08:49:15
1 comment
Im an indian IMG who is an intern,started preparing 1.5 months ago. Still haven’t started my dedicated period as I work until 4PM on most days.
Currently using First aid (with the help of AnKing deck) and Uworld as my main sources of prep and supplemented learning concepts I couldn’t understand using boot camp.
Decided to start with Cardiology as it was one of my stronger subjects but I was humbled very fast.
Dropped cardio before beginning its pharmacology part and decided to change it up by doing dermatology which I successfully completed
As I’m an IMG and MCQs were not the primary testing form during medschool I have been struggling to find a way to use Uworld and first aid side by side as learning tools.
I’m having a hard time going back and completing cardiology as I have so many flagged questions to review in Uworld (as well as the entire pharmacology portion left out) and doing anki isn’t helping as much as I thought it would.
What is a good way to review Uworld? And is there any way to approach cardiology and successfully finish it entirely without having doubts about how strong I am in it? I’m new to this entire testing and learning method so please give advice!
I was also considering starting to do anki along with first aid for micro on the side as I’ve been reading that it’s vast and it’s better to start off early.
I would ideally like to give the exam 6-8 months from now (assuming I feel prepared but it’s looking doubtful at this rate)
Would love some input!
Post 248: NBME 25: 55%, Exam in 3 weeks
https://www.reddit.com/r/step1/comments/1kiev0h/nbme_25_55_exam_in_3_weeks/
09/05/2025, 07:07:18
9 comments
Hey Guys! I hope everyone’s doing well.
I posted here almost a week ago when I took NBME 26 and scored 44%
Today I took NBME 25 and scored 55%
The time in between the two NBMEs I reviewed some systems, didn’t review repro and msk (these were also my weak areas)
Did uworld 80 questions everyday with average around 45-50%
At this point I don’t think there’s anything I don’t understand. I think I struggle with memorising stuff. Anking is not my jam, I’ve tried it multiple times
While solving questions I remember studying that stuff but I cannot recall it properly in my head.
My exam is on June 4th, I really don’t want to postpone it and want to get done with it
Can anyone please guide me how to improve my nbme scores further and how to retain stuff long term without anki?
Post 249: Anyone willing to provide their U world Step 1 remaining subscription for free?
https://www.reddit.com/r/step1/comments/1kieohu/anyone_willing_to_provide_their_u_world_step_1/
09/05/2025, 06:55:15
8 comments
Hey guys! Is anyone that has a remaining U World Step 1 subscription (that they won't be using anymore) willing to give it to me for free?
I fell sick the past 2-3 months and was unable to effectively use my subscription to the fullest. I believe I still need more time to prepare for the exam, as I am also recovering from my condition both physically and mentally. Given my financial circumstances too, I don't think I can afford to renew my current sub.
I reached out to U World Support but unfortunately they replied saying they can't renew my current one.
It's a big ask but please if there is anyone that has a subscription>90 days ,and is willing to give it away for free - please inbox. I would really appreciate it. Thank you so much 🙏🏽
Post 250: nbme 28 56%! exam in a month 😭
https://www.reddit.com/r/step1/comments/1kieatj/nbme_28_56_exam_in_a_month/
09/05/2025, 06:27:58
6 comments
i have been giving nbmes since 2 weeks now and I plan to the the step in early june
my scores on nbme 26 27 were 62 percent
i dont know what I am doing wrong!
did FA, uworld 70 percent completed
i really need help to move forward
advice appreciated
i cannot move back my exam due to my uni exams 😭 i have to take it in june
Post 251: What should I do?
https://www.reddit.com/r/step1/comments/1kiaur8/what_should_i_do/
09/05/2025, 02:22:37
3 comments
Hey my exam's date is one month away in June.
I gave NBME 21 as basine 2 months ago and got 56%.Then, two weeks back gave NBME 26 with 67% correct with full confidence that I'll get 70% in my next practice test.
Few days back gave NBME 28 and unfortunately got 62% (5% decrease)
I don't know what went wrong but it was really disappointing. Some questions I couldn't comprehend what they were trying to ask as the writing style of NBME is way different and vague from Uworld.I want to increase my score to 70+ and having a hard time to do so.
I find it hard to passively read first aid and right now reviewing the NBME I gave both corrects and in corrects.
The systems/subjects I got wrong mostly were MSK, Respo, Repro, Biochem and Biostat.
Can someone give ways to improve the score?
Like will ever I be able to get high score.
I haven't completed Uworld yet, only 60% completed.
With 30 days to go, torn between completing Uworld or revising first aid somehow and giving rest of the nbmes.(Don't want to waste nbmes as well with low scores)
Post 252: 55% and 32nd percentile
https://www.reddit.com/r/step1/comments/1ki9ti2/55_and_32nd_percentile/
09/05/2025, 01:18:01
2 comments
Hello, I just began my preparation for step 1, I am an IMG and I am planning to take step 1 in a year (I am currently in my research intern year before obtaining my oficial title, Mexicans will get it)
I want to know if this is a good start taking into account that the education I received is far away from the kind of questions step 1 asks (education was more clinical rather than molecular)
I began with UWorld on May 1st
Post 253: Reviewing UWORLD MCQS
https://www.reddit.com/r/step1/comments/1ki9set/reviewing_uworld_mcqs/
09/05/2025, 01:16:14
3 comments
TOO STUFFED !
Its taking me 4-5 dedicated hours to review a block , where I think I can be wrong..
- Im memorising (not just conceptualising) everything , tables and stuff.
- I even look into mentioned stuff of other system eg full pathogenesis of schistosomiasis even when Im doing GIT.
The thing is I really need to manage time along with uni exams and this is a burn out >>
Post 254: PASS
https://www.reddit.com/r/step1/comments/1ki9ojq/pass/
09/05/2025, 01:09:55
19 comments
Non US IMG, Studied along with full time MO duty.
Focus on the concepts and their application part rather than just mugging up things. Tested heavily on application part. Tested on 23/4/25. Trust your NBME Scores. Give one or two full length assessments. Pray & Pray. Happy to help. Shoot your questions in replies below.
NBMEs
25/10: 25: 58% 84 incorrect
16/11: 26: 60.5% 79 incorrect
17/02: 27: 66% 68 incorrect
19/03: 28: 69% 62 incorrect
08/04: 29: 66.5% 67 incorrect
13/04: 30: 71.5% 57 incorrect
14/04: Free 120 2021: 75% 90 correct
14/04: Free 120 2024 ONLINE: 72% 86 correct
15/04: 31: 74% 52 incorrect
Post 255: 5/8/25 takers...
https://www.reddit.com/r/step1/comments/1ki8i6v/5825_takers/
09/05/2025, 00:02:26
16 comments
What were your thoughts?
It felt harder than i thought it would be. Felt confident going in with decent uworld/free 120 scores but that test kinda slapped
Post 256: Am I in a good spot?
https://www.reddit.com/r/step1/comments/1ki7ocf/am_i_in_a_good_spot/
08/05/2025, 23:17:30
3 comments
Testing 6/9
UWSA1 on 4/30: 69% (239)
NBME 25 on 5/4: 73%
UWSA2 on 5/8 : 71% (232)
Planning on taking the rest of the NBMEs, free 120 over the next month
Post 257: Do I take step 1 end of June...??
https://www.reddit.com/r/step1/comments/1ki727f/do_i_take_step_1_end_of_june/
08/05/2025, 22:44:31
4 comments
Hi everyone, I’m an IMG currently preparing for Step 1 and I need help deciding the right timing for my exam.
I originally planned to take Step 1 at the end of June, which would give me around a month to prepare for my FMGE (India licensing exam) scheduled on July 26. But now, I’m realizing that I’m not ready for Step 1 by June — my NBME 25 score was below 50%, and I haven’t built the consistent study momentum I need. And I finished about 56% in uworld. The consistency in preparation has gotten to an all time low because I recently feel sick and I recovered well but I completely lost all the momentum I had from before and I'm finding it very difficult to do my questions and stay on track.
If I take Step 1 at the end of June, I’ll barely have time (or energy) to prepare for FMGE, which covers 19 subjects and six years of medical school. That would mean underperforming in both. But just to let you know I am taking FMGE just for the sake of taking it. I thought taking step in June would give me atleast a month to cram for FMGE but turns out I'm not at all ready for step and basically my family is the main reason why I feel so pressued in taking it at the end of June because I've postponed it for a long time now. But of course I can convince them because it is better to make them upset and take the exam when I am ready and pass it than take it in a rush and fail it.
So I’m thinking of:
Taking NBME 26 on May 15 as a checkpoint.
If I score 60%+, I might push through and take Step 1 at the end of June.
If I score <60%, I’ll reschedule Step 1 to end of August so I can focus briefly on FMGE in July and then get back to USMLE prep with a clearer head.
I’m under pressure from my family to take Step 1 soon, but I don’t want to waste an attempt. Has anyone been in a similar situation juggling FMGE and Step 1? Would love to hear how you balanced both or what you’d suggest. Thank you!
Post 258: Tip for remembering EKG/coronary artery STEMI localizations:
https://www.reddit.com/r/step1/comments/1ki6yox/tip_for_remembering_ekgcoronary_artery_stemi/
08/05/2025, 22:39:19
16 comments
I came up with this trick to remember what leads and vessels go together for STEMI localization and thought it might be helpful for anyone who struggles to remember these.
-II, III, aVF: box around them looks like side view of a Foot, F = Foot = inFerior = RAD (think descending=down=inferior)
-I, aVL: draw a diagonaL line to connect them, L = Lateral = Left circumfLex (has 2 Ls in the name)
-V1-V4: draw an upside down A to connect them, A = Anterior = LAD (anterior is in the name)
*to remember if it is LAD vs RAD: left is anterior because the left ventricle is closest to the anterior chest wall and the right ventricle is tucked under it (ie “inferior”)
Post 259: Need Advice on what to change
https://www.reddit.com/r/step1/comments/1ki5cjo/need_advice_on_what_to_change/
08/05/2025, 21:14:24
5 comments
So I'm lost on what to do. I realize this is going to sound helpless and probably annoying since there are so many posts about how to study.
I'm scheduled to take comlex on 6/3 and step 1 6/13. This week I took comsae and the CBSE and only got a 426 and a 50%. I got a 50% on UWSA1 as well :( . So i have to retake both the CBSE and COMSAE in two weeks to get approved to take the exams. I'm not sure if I should just switch to doing only questions using Uworld or if I should also continue content reviewing. Up until now I have been watching sketchy and bootcamp videos pretty much every day and hardly doing questions. And while the videos are helpful I find that when I do questions I get concepts I watched videos on wrong a lot, and a lot of the time its because I don't remember a detail. I don't use anki and I've tried but I can't get through thousands of cards a day. Filtering the anking deck to the bootcamp videos still ends up being like 400+ cards. Honestly I'm not sure if trying it again would be helpful now since I'm so close to my exam date. The other issue I'm having is that reviewing questions takes me way too long, I literally have no idea on how to review faster. I read every single explanation and make sure I understand the concept, and a lot of time will spend time googling or looking up points in the explanation that I didn't understand. I don't have a lot of Uworld done (literally 15%) since our school makes us do Trulearn which cuts into my time to do UW questions. And we need to get a certain percent correct on trulearn (57%) which I'm not meeting currently either. I also don't really have a dedicated which I think contributed to the problem, being on campus all day made it hard to study at home. My class schedule is lighter for the next few weeks so I'm hoping to be way more productive I just don't know what to do.
I have to move and start orientation for 3rd year 6/24 so I really dont think I can even push it back.